Sei sulla pagina 1di 143

Instituto Universitario Eclesistico Santo Toms de Aquino Palmira Edo.

Tchira Venezuela

Introduccin a la lgica formal


ERNST Bruno, Adventures with impossible figures, Tarkin Publications, Norfolk 1986.

El valor de razonar

fr. Fernando Ruiz Valero, O. de M. http://fernandoruiz.wikispaces.com


Curso 2007/2008 v. 2.12 Ad Usum Privatum Creative Commons

A los que quieren pensar, y desean conocer la verdad, porque la verdad los har libres... (Jn 8,32)

...

Instituto Universitario Eclesistico Santo Toms de Aquino San Cristbal Edo. Tchira Venezuela

El valor de razonar. Introduccin a la lgica formal.


CONTENIDO

DEDICATORIA NDICE PRESENTACIN DE LA MATERIA Y BIBLIOGRAFA PARTE I: INTRODUCCIN 1 2 3 4 PENSAR Y ESTUDIAR LGICA LA FILOSOFA Y LA LGICA LENGUAJE, LGICA Y VERDAD HISTORIA DE LA LGICA PARTE II: LGICA CLSICA 5 6 7 8 9 EL TRMINO I: LA SIMPLE APREHENSIN Y EL TRMINO EL TRMINO II: LA DEFINICIN EL JUICIO I: LA PROPOSICIN EL RACIOCINIO I: LA ARGUMENTACIN EL RACIOCINIO II: EL SILOGISMO PARTE III: LGICA MODERNA Y APLICACIONES 10 LGICA PROPOSICIONAL 11 LGEBRA DE CONECTIVAS 12 OTRAS LGICAS: LGICA DE PREDICADOS, DE PROBABILIDAD Y LGICAS BORROSAS 13 ARGUMENTACIN E INVESTIGACIONES LGICAS

iii

iv

Instituto Universitario Eclesistico Santo Toms de Aquino Palmira Edo. Tchira Venezuela

Versin: 2.12

El valor de razonar. Introduccin a la lgica formal Presentacin de la materia de Lgica


1. MATERIA Profesor: Alumnos: Temporizacin: fr. Fernando Ruiz Valero, O. de M. frmerced@yahoo.com Cf.: http://fernandoruiz.wikispaces.com . Estudiantes del IUESTA CBS: 4 h / semana; 17 semanas (68 horas lectivas) EMT y EMF: 2 h / semana; 17 semanas (34 horas lectivas) 2. MOTIVACIN Y OBJETIVOS El estudio de la lgica responde a una doble motivacin. Ante todo, la lgica investiga y establece las reglas del pensamiento correcto, permitindonos encontrar y estar seguros de verdades no inmediatas; es necesaria por tanto para poder avanzar en el camino del conocimiento. En este sentido, su finalidad es instrumental y metodolgica. Al mismo tiempo, estudiamos la lgica con actitud filosfica, preguntndonos porqu es as y cul es la verdadera realidad del pensamiento, su esencia y cmo se relaciona la seguridad del razonamiento con los hechos fsicos. En este sentido, es una filosofa del pensar correcto. La capacidad para mostrar la verdad mediante razonamientos correctos nos habla de la realidad y su relacin con el pensamiento (abriendo el camino a la fsica, a la metafsica y a la epistemologa).

Los objetivos de nuestro estudio son: Asumir una aproximacin racional a la realidad, afrontando los problemas por medio de la reflexin, el anlisis y la argumentacin. Conocer y comprender la situacin de la lgica en el conjunto del saber filosfico y prctico, as como su evolucin histrica. Comprender y manejar las reglas de la argumentacin correcta en el sistema aristotlico, y su desarrollo en las lgicas modernas. 3. BIBLIOGRAFA BSICA

COPI Irving M., Introduccin a la lgica, Ed. Universitaria de Buenos Aires, EUDEBA, 18 1977 [ed. or. Inglesa 1953-1972], 614 pgs. VERNEAUX Roger, Introduccin general y lgica, = Curso de filosofa tomista 1, Herder, Barcelona 31978 [ed. or. 1964], 182 pgs. SANGUINETI Juan Jos, Lgica, Ediciones Universidad de Navarra, Pamplona 1982, 211 pgs [con bibliografa]. BURGOS Alfonso, Iniciacin a la lgica matemticaEdiciones Vega, Caracas 101983, 91 pgs.
NOTA: Ms adelante se indica una Bibliografa selecta de libros y de recursos en Internet.

4. TEMARIO PARTE I: INTRODUCCIN 1 2 3 4 5 6 7 8 9 PENSAR Y ESTUDIAR LGICA LA FILOSOFA Y LA LGICA LENGUAJE, LGICA Y VERDAD HISTORIA DE LA LGICA PARTE II: LGICA CLSICA EL TRMINO I: LA SIMPLE APREHENSIN Y EL TRMINO EL TRMINO II: LA DEFINICIN EL JUICIO I: LA PROPOSICIN EL RACIOCINIO I: LA ARGUMENTACIN EL RACIOCINIO II: EL SILOGISMO PARTE III: LGICA MODERNA Y APLICACIONES 10 LGICA PROPOSICIONAL 11 LGEBRA DE CONECTIVAS 12 OTRAS LGICAS: LGICA DE PREDICADOS, DE PROBABILIDAD Y LGICAS BORROSAS 13 ARGUMENTACIN E INVESTIGACIONES LGICAS 5. EVALUACIN - Evaluacin de la primera parte - Evaluacin de la segunda parte - Evaluacin de la tercera parte - Evaluacin final - Nota valorativa 20% (examen) 20% (examen) 20% (trabajos de clase) 30% (las partes aprobadas con 18 o ms liberan) 10% (por trabajos finales)

6. INDICACIONES a) Estudiar la lgica como si fuera una materia informativa (fechas, acontecimientos, reglas memorizadas) es una soberana estupidez en s misma. Debemos aprender a pensar, no memorizar lo que otros han pensado. b) Es sin embargo imprescindible conocer lo que otros han pensado, porque intentar iniciar de nuevo la aventura del pensamiento es ingenuo y estril, ya que el conocimiento es dialogante e histrico (en dilogo con los que nos han precedido). c) Se espera de los alumnos que, en trabajos, exmenes y tareas en clase se expresen con: precisin correccin concisin pertinencia Los escritos que no consigan estos requisitos y adolezcan de claridad y coherencia lgica sern rechazados sin posibilidad de interpretacin a posteriori (dejemos la exgesis a otros). d) tica del estudiante. Se espera del estudiante un comportamiento tico y responsable para aprender la materia, de modo que la nota sea una evaluacin correcta de su aprovechamiento. Todo material citado o referenciado debe sealarse segn la metodologa. Consecuentemente, toda copia en trabajos y exmenes supone la descalificacin del alumno y la punicin ms alta que contemple el reglamento.

vi

7. BIBLIOGRAFA DESARROLLADA a) Bibliografa bsica: COPI Irving M., Introduccin a la lgica, Ed. Universitaria de Buenos Aires, EUDEBA, 18 1977 [ed. or. Inglesa 1953-1972], 614 pgs. [Es una obra magnfica, de un noteamericano que hace accesible la lgica silogstica mediante numerosos ejemplos y ejercicios]. VERNEAUX Roger, Introduccin general y lgica, = Curso de filosofa tomista 1, Herder, Barcelona 31978 [ed. or. 1964], pp 182. SANGUINETI Juan Jos, Lgica, Ediciones Universidad de Navarra, Pamplona 1982, 211 pgs [con bibliografa]. BURGOS Alfonso, Iniciacin a la lgica matemticaEdiciones Vega, Caracas 101983, pp. 91. b) Bibliografa general BALMES Jaime, El Criterio, Garnier hermanos, Pars 19??. CARROLL Lewis, El juego de la lgica, = H 4439, Alianza Editorial, Madrid 2002, 181 pgs. CRYAN Dan SHATIL Sharron MAYBLIN Bill, Lgica para todos, Paidos, Barcelona 2005 [es una magnfica historia ilustrada de la lgica; ed. or. en ingls, 2001], 176 pgs. ESCOBAR Gustavo, Lgica. Nociones y aplicaciones, Ed. McGraw-Hill, Mxico 1999, 312 pgs. GARRIDO Manuel, Lgica simblica., Cuarta edicin, Tecnos, Madrid 42001, 540 pgs [con bibliografa; es una de las grandes obras sobre lgica simblica, referencia obligada pero de nivel bastante alto]. LPEZ-DRIGA Enrique, Piensa ms y acertars, Ediciones Paulinas, Lima 1992, 192 pgs. MARITAIN Jacques, El orden de los conceptos. I: Lgica menor, Club de Lectores, Buenos Aires 1978, 389 pgs. SANGUINETI Juan Jos, Lgica, Ediciones Universidad de Navarra, Pamplona 1982, 211 pgs [con bibliografa]. WITTGENSTEIN Ludwig, Tractatus Logico-Philosophicus, Versin espaola de Enrique Tierno Galvn, = AU 50 (edicin bilinge), Alianza Editorial Madrid 1975, 203 pags. YORIS Corina, Introduccin a la lgica. Problemario. Apndice: Notas sobre lgica proposicional, UCAB, Caracas 21995. c) Bibliografa en Internet Recursos generales: Arvo.net, Mundo, hombre, Dios; En Internet [2006]: http://www.arvo.net/index.asp .

vii

BLATT Ron, Java categorical syllogism program, en Internet [2006]: http://www.roninabox.com/venndiagram.html [programa que presenta formas de silogismos y permite hacer el anlisis del mismo mediante diagramas de Venn; en ingls]. CURTIS Gary N., Fallacy Files, en Internet [2006; ed. or. 2001]: http://www.fallacyfiles.org/ . DUNIHO Fergus, Sylly Syllogisms, en Internet [2006; ed. or. 2002]: http://www.duniho.com/fergus/sillysyllogisms.html [En una pgina con un programa en Java que genera silogismos aleatoriamente y permite chequear la respuesta; es magnfico para hacer muchos ejercicios en poco tiempo; en ingls]. GARCA DAMBORENEA Ricardo, Uso de razn. El arte de razonar, persuadir, refutar. Un programa integral de iniciacin a la lgica, el debate y la dialctica, con el diccionario de falacias ms completo. en Internet [2007]: http://www.usoderazon.com/ . Lgica. Apuntes de lgica clsica, Colegio Diocesano M.A. Alemn, Ushuaia, Tierra de Fuego (Argentina), publicado slo en Internet [agosto 2002]: http://orbita.starmedia.com/buhofueguino/logica.htm . ROMERA ngel, Retrica. Manual de retrica y recursos estilsticos, en Libro de notas, en Internet[2007]: http://retorica.librodenotas.com/ . SION Avi, Future Logic. Categorical and Condicional Deduction and Induction of the Natural, Temporal, Extensional and Logical Modalities Avi Sion editor, Vancouver-Geneva (Switzerland), 21996; en Internet [2006]: http://www.thelogician.net/2_future_logic/2_fl_frame.htm . d) Biblografa en Internet - Artculos: Artculos de Lgica en Internet [2006]: http://www.fciencias.unam.mx/lytc/articulos/lm/ . CIIDET, Lgica Matemtica, en Internet [2006]: http://www.divulcat.com/monografias/matematicas/logica_matematica.html (se encuentra tambin en monografas.com, pero mal transcrito : http://www.monografias.com/trabajos4/matematica/matematica.shtml ). ESCUDER J., Problemas de Lgica, en Internet [2005]: http://platea.pntic.mec.es/~jescuder/logica.htm Lenguaje y argumentacin, Colegio Diocesano M.A. Alemn, Ushuaia, tierra de Fuego (Argentina); publicado en Internet [2006; agosto 2002]: http://orbita.starmedia.com/buhofueguino/unidad3.htm The Stanford Encyclopedia of Philosophy (Winter 2004 Edition), Edward N. Zalta (ed.), URL = http://plato.stanford.edu/ . ( Tiene numerosos artculos sobre lgica, firmados por autores competentes en el tema. Ver: Aristotle's Logic "Relativism" ; "The Analysis of Knowledge". Informal Logic).

viii

6. DESARROLLO DEL TEMARIO


NOTA: El temario es slo aproximativo y el autor se reserva el derecho de introducir cambios a medida que se desarrolle la asignatura. PARTE I: INTRODUCCIN 1 PENSAR Y ESTUDIAR LGICA (4H)
Presentacin de la disciplina acadmica La filosofa y su novedad El Logos - La actitud racional y otras actitudes Los dos aspectos de la lgica. Apndice: criterios metodolgicos Comentario a la bibliografa Metodologa investigativa en esta materia.

2 LA FILOSOFA Y LA LGICA

(4H)

Qu es filosofa? Qu es lgica? Correccin formal y verdad real (verdad lgica) Definiciones y sus contextos Filosofa como sabidura y como ciencia Divisin del saber.

3 LENGUAJE, LGICA Y VERDAD

(4H)

Las funciones bsicas del lenguaje Las cuatro formas del discurso Emotividad y objetividad El campo de la lgica Valoracin de la verdad Falacias.

4 HISTORIA DE LA LGICA

(4H)

Los sofistas y sus enredos Aristteles: el Organon Tras Aristteles: Crisipo de Soli, Porfirio, Boecio La escolstica aristotlica Duns Scoto y Ramon Llull Leibniz El despliegue de las lgicas modernas: de Gottlob Frege y Russell, Carnal y Wittgenstein, Hempel y Popper, Hilbert y Gdel Lgicas no clsicas: intuicionismo, Lukasiewicz, lgica borrosa y lgica cuntica Panorama actual terico y prctico.

PARTE II: LGICA CLSICA 5 EL TRMINO I: LA SIMPLE APREHENSIN Y EL TRMINO (4H)


En el origen de la informacin: el trmino o concepto Propiedad de los trminos Clasificacin de los trminos Predicables y predicamentos Relaciones entre trminos.

6 EL TRMINO II: LA DEFINICIN

(4H)

Definicin: arte y ciencia Formas de definir y tipos de definicin El Diccionario como aventura Divisin: arte y ciencia Monumental divisin: el rbol de Porfirio.

7 EL JUICIO

(4H)

Qu es el juicio o proposicin Tipos de juicio: caractersticas del juicio categrico Propiedades de los juicios categricos Relaciones entre juicios: el cuadro de oposicin y otras relaciones.

8 EL RACIOCINIO I: LA ARGUMENTACIN

(4H)

Qu es pensar y qu es razonar? La estructura de la argumentacin Leyes de la correcta argumentacin Razonamiento inductivo y razonamiento deductivo Fundamento del razonamiento.

9 EL RACIOCINIO II: EL SILOGISMO

(12H)

I. Teora: Qu es un silogismo? Anlisis de la composicin de un silogismo categrico tpico Principios y reglas del silogismo Figuras y modos Otros tipos de Silogismos.

ix

II. Prctica: Identificacin de silogismos correctos e incorrectos Evaluacin de argumentos en lenguaje corriente Normalizacin y organizacin Silogismos encadenados Entimemas Casos prcticos.

PARTE III: LGICA MODERNA Y APLICACIONES 10 LGICA PROPOSICIONAL (4H)


Origen histrico de la lgica proposicional Proposiciones simples y complejas Conectivas: estudio de las seis conectivas Tablas de verdad Analogas elctricas y electrnicas Tautologas y Contradicciones Listado de tautologas como teoremas.

11 ALGEBRA DE CONECTIVAS

(4H)

Principios del clculo proposicional Verificacin de casos con tablas de verdad Uso simple de las tautologas Uso deductivo del clculo Uso demostrativo del clculo Aplicaciones: buscadores en Internet, sistemas electrnicos, Sudoku

12 OTRAS LGICAS: LGICA DE PREDICADOS, DE PROBABILIDAD Y LGICAS BORROSA

(4H)

Explorando las fronteras de la lgica moderna Introduccin a la lgica de predicados: clases, modos, relaciones Expandiendo los valores de verdad: lo probable y lo difuso - Dnde queda la verdad?

13 INVESTIGACIONES LGICAS

(4H)

Dialctica clsica Retrica clsica El dilogo como bsqueda de verdad y de acuerdo Aplicacin de la lgica a las decisiones de la vida diaria La opcin: racionalidad o irracionalidad Investigacin de la lgica-logos subyacente a un problema Palabras de despedida. --------------ooooo+ooooo-----------IMPORTANTE Esta obra est bajo una licencia Attribution-NonCommercial-ShareAlike 2.5 de Creative Commons. Para ver una copia de esta licencia, visite http://creativecommons.org/licenses/ Licencia: fr. Fernando Ruiz Valero, O. de M. Creative Commons 2006. Tipo: AttributionNonCommercial-ShareAlike (http://creativecommons.org/licenses/by-nc-sa/2.5/ ). Eso significa que puedes libremente reproducir, copiar, e incluso modificar como mejor le parezca, de una parte o de la obra entera, siempre que: Reconocimiento. Debe reconocer los crditos de la obra de la manera especificada por el autor (en este caso: cita completa de la obra, con autor, ttulo, Institucin educativa y edicin). No comercial. No puede utilizar esta obra, ni cualquiera en la que use parte de ella o un derivado de ella, para fines comerciales (en ese caso ponerse en contacto con el autor :-). Compartir bajo la misma licencia. Si altera o transforma esta obra, o genera una obra derivada, slo puede distribuir la obra generada bajo una licencia idntica a esta.

Lgica - 1

LGICA - PARTE I: INTRODUCCIN


1 PENSAR Y ESTUDIAR LGICA 2 LA FILOSOFA Y LA LGICA 3 LENGUAJE, LGICA Y VERDAD 4 HISTORIA DE LA LGICA

Lgica - 2

La Lgica estudia la razn misma en cuanto que es instrumento de la ciencia o medio de adquirir y poseer lo verdadero. Jacques MARITAIN, El orden de los conceptos, 13.

1 Pensar y estudiar Lgica

V. 2.05

Presentacin de la disciplina acadmica La filosofa y su novedad El Logos - La actitud racional y otras actitudes Los dos aspectos de la lgica Apndice: criterios metodolgicos Comentario a la bibliografa Metodologa investigativa en esta materia. Bibliografa especfica: VERNEAUX, Introduccin general y Lgica. Trabajo: valoracin y resolucin de problemas lgicos.

Presentacin de la disciplina acadmica El estudio de la lgica se asocia con complicados problemas formulados en un lenguaje oscuro y anticuado. O bien, con sutiles juegos de razonamiento que algn avispado interlocutor nos presenta para dejarnos cavilosamente en ridculo1. Sin embargo, la lgica es un elemento habitual de nuestra vida diaria, si es que es una vida inteligente. Cualquier ser humano, sin haber estudiado esta materia, la utiliza constantemente en todo tipo de razonamientos para comprender lo que ocurre y anticipar las consecuencias de determinadas decisiones. Estudiar lgica es, en principio, muy sencillo. Se trata, simplemente, de saber cundo la conclusin de un razonamiento es con toda seguridad verdadera, y por tanto nos podamos fiar de esa conclusin (creo que el carro aguantar nueve sacos de cemento y dos personas; cambiar armas por dinero disminuir la violencia en las calles ). Dos son las condiciones para tener absoluta seguridad: una, que la informacin de la que se parte sea verdadera; la otra, que el razonamiento sea correcto. De certificar esta segunda condicin se encarga la disciplina que nos ocupar durante este semestre.. Este curso de lgica est dirigido a alumnos del IUESTA, un instituto universitario que ofrece estudios de filosofa, teologa y educacin. Debido a esta orientacin humanstica de las materias que los alumnos han de cursar, hemos decidido centrarnos en la exposicin de la lgica clsica o aristotlica. No podemos con todo dejar de exponer, aunque sea superficialmente, los conceptos bsicos de la lgica proposicional o lgica matemtica, con la que abordaremos especficamente los razonamientos condicionales. Al mismo tiempo que se exponen las cuestiones lgicas relativas al trmino, el juicio y el razonamiento, se pretende que el lector adquiera la capacidad de usar con precisin estas herramientas universales del pensamiento mediante tcnicas que sern de gran utilidad en cualquier debate o estudio que se aborde. As se crear un hbito de racionalidad que lleva a buscar la fundamentacin y justificacin de las convicciones en cualquier actividad de la inteligencia.
1 En apndice hemos reunido algunos problemas de lgica que usaremos para ilustrar distintas formas de razonamiento. Por poner un ejemplo divertido: El caballo de Mac es ms oscuro que el de Smith, pero ms rpido y ms viejo que el de Jack, que es an ms lento que el de Willy, que es ms joven que el de Mac, que es ms viejo que el de Smith, que es ms claro que el de Willy, aunque el de Jack es ms lento y ms oscuro que el de Smith. Cul es el ms viejo, cul el ms lento y cul el ms claro?. Cf. Infra.

Lgica - 3

Antes de iniciar el estudio especfico de la lgica, se expondrn algunas cuestiones introductorias de filosofa, muy elementales, que orienten al alumno en sus primeros pasos por esta disciplina tan olvidada en la vida y el curriculum de los estudiantes venezolanos. Finalmente, a lo largo de la materia sealaremos las cuestiones filosficas que la lgica nos plantea, por ejemplo a travs de las paradojas y aporas, de la pretensin de totalidad de los sistemas axiomticos, o la gran cuestin sobre el origen de la necesidad lgica. Cmo estudiar esta materia? Me permito dar algunas sugerencias que quizs la mayora de alumnos no necesitan. Ante todo, es muy importante despertar en uno mismo la curiosidad y el inters, ya que el conocimiento est guiado por el inters 2. Lea la motivacin y los objetivos de la presentacin curricular. Pregntese qu es lo que le puede resultar interesante del temario (las preguntas son la encarnacin del inters). Decida cuales van a ser sus objetivos particulares ante esta asignatura. Rtese a s mismo con algunos de los acertijos del apndice. En segundo lugar, hay que afrontar la teora. No es mucha, pero es muy importante. Y exige reflexin, lectura, consulta de la bibliografa y hacer ejercicios para afianzar las ideas. Cada alumno debe hacer su propia investigacin en la teora, segn sus intereses, haciendo interiormente un dilogo: dialogar con la exposicin del profesor (la mayora de las preguntas quedan para uno, para el estudio posterior); dialogar con los libros que uno usa para afianzar los conocimientos. Sin esta reflexin interior es imposible aprender una materia filosfica. En tercer lugar, hay que hacer mucha prctica. Ejercicios o problemas de cada tema, usando los ejercicios propuestos y los de los libros de apoyo. No basta comprender cmo se hacen las cosas, hay que adquirir claridad y seguridad. Eso slo se logra con muchas horas de prcticas. Dudas: si uno no tiene dudas al final de una clase, no ha aprovechado la materia (o bien el alumno y el profesor son ambos unos genios). La duda, como mostr Descartes, es uno de los mejores instrumentos para forzarnos a obtener un conocimiento seguro. Escuche sus dudas y afrntelas con artillera pesada: buenos libros y reflexin. Evaluacin: es a base de exmenes con teora y prctica, con respuestas breves; adems, haremos trabajos que contarn para una de las notas parciales. No estudie para los exmenes, pero tenga en cuenta cmo se le va a evaluar. Atencin a: las palabras clave (hay que comprender bien cada concepto, y aprenderse los propios de la materia; de cada tema debera tener un glosario de las palabras ms importantes); las relaciones (tener los conceptos ordenados); los diversos tipos de razonamiento y sus leyes. Contrastar este material con los ejercicios en cuestiones prcticas y no dejar de buscar nuevos casos de ejemplo.

La filosofa y su novedad Nos preguntamos desde una perspectiva histrica y conceptual qu es lo que puede aportar la filosofa a la vida del ser humano. Eso nos llevar a comprender qu tiene de nuevo la filosofa y para qu puede servir. La filosofa podemos entenderla, en primera aproximacin, como una reflexin sistemtica de la persona humana, de un tipo determinado: una reflexin racional. Tiene por tanto dos componentes.
Este principio de Teora del Conocimiento o Epistemologa es a mi modo de ver la causa ms frecuente del xito o el fracaso en el estudio universitario. En los estudios de filosofa no hay personas inteligentes o torpes, hay personas motivadas y desmotivadas.
2

Lgica - 4

El primer elemento es la pregunta, la cuestin sobre el porqu. El preguntar, el estar siempre con el Por qu?, buscar el sentido que tienen las cosas y lo que ocurre en la vida es inherente al ser humano. Cuando el beb empieza a dominar el lenguaje, empieza a usar las preguntas para obtener informacin.. En conclusin, preguntarse el porqu de las cosas se ha dado siempre (hasta esta generacin ) y es natural en el ser humano. Pero hay diversos modos de respuesta. El primer grupo de respuestas es el de las respuestas prcticas. Son las que se dan en el entorno conocido de la persona, y suponen el inicio de la tcnica. Ante la necesidad de mejorar el filo del hacha o conservar el fuego encendido, el ser humano comprende de un vistazo o con una ligera reflexin cules son los elementos determinantes (estableciendo un primer esquema de causa-efecto) y acta sobre el entorno para conseguir el efecto deseado (busca mejores piedras de silex y las afila con cuidado, o acumula buena cantidad de lea y se asegura de que est seca). Desgraciadamente, hay otras muchas cosas que el ser humano no comprende ni controla: porqu se me infectan los ojos? porqu la trampa de mi vecino est cada maana llena de chigires y la ma no? porqu no llegan las lluvias que tanto necesitamos?. Ante la incapacidad de comprender las causas, o ante la imposibilidad de controlar estos procesos que nos afectan, el ser humano tiende a desarrollar respuestas mticas. Son aquellas respuestas a la razn de las cosas que dan una solucin externa a la cosa misma. Su causa puede estar en el mundo humano: la tradicin o la autoridad desptica justifican el que el jefe se lleve nuestros mejores corderos... o bien puede referirse a fuerzas sobrenaturales que el ser humano intuye y da forma: la infeccin se debe a un mal de ojo, las pobres cosechas a un dios disgustado por las pocas ofrendas. Profundizando en esta manera de responder mticamente a la pregunta natural por la razn de las cosas se pueden enumerar respuestas:

por la supersticin y magia (es as porque una fuerza caprichosa quiso que as fuera) por mitos (las estaciones obedecen a una lucha eterna entre las fuerzas de la vida y las de la muerte) por la va gnstica: hacer de la razn una va a la iluminacin mstica ( porque esta es la estructura oculta del cosmos que slo los iniciados conocen). por la autoridad absoluta del que manda (porque s, porque lo mando yo) por opiniones respetadas en el pasado: tradicin (siempre se ha hecho as) por la va religiosa o mstica (obedece a un designio de Dios).

Todava hoy en da muchas personas responden a sus propios interrogantes con respuestas de este tipo3. Y al inicio de la historia de la humanidad eran las respuestas tpicas a la mayora de los problemas ltimos. Slo poco a poco se difundieron las explicaciones de otro tipo: las explicaciones racionales. La explicacin racional se basa en dos presupuestos: uno, que las ideas que formulamos sobre el mundo pueden ser verdad, es decir, que pueden adecuarse a la realidad de lo que

Pregntese cuantos comportamientos supersticiosos tiene Ud. todava: amuletos, hilos o cruces contra el mal de ojo? evitar cruzarse con un gato negro o pasar bajo una escalera? consultar mi horscopo para la semana? creer que se puede predecir el futuro y que est escrito en mi mano o en un poso de caf?. Esperamos sinceramente que al final de la filosofa haya adquirido una actitud racional frente al mundo y sus incertidumbres. Un sano escepticismo contra estas tonteras y contra muchas teoras de conspiracin ayuda a tener una vida abierta y sin prejuicios. Para ampliar: El escptico digital, en Internet [2006]: http://digital.el-esceptico.org/ .
3

Lgica - 5

existe4. La respuesta racional supone una apuesta por respetar la realidad, por encima de la autoridad, la tradicin o las propias creencias. El segundo elemento es la apuesta y la capacidad de comprender las causas internas de los propios procesos, las que se derivan del modo de ser de las cosas mismas y de las relaciones que tienen entre ellas. Si mi nio tiene mal de ojo o diarrea, examino el ojo o las comidas que ha ingerido, no lo que se le ocurre al chams de turno, o al seminarista que trae agua bendita. Es una apuesta arriesgada, por que las causas son muchas veces difciles de conocer, y es ms cmodo confiar en respuestas rpidas aunque sean estpidas5. , La actitud filosfica es una actitud racional extrema. consiste en preguntarse el porqu de las cosas... y responder desde la racionalidad de las cosas mismas. Por eso la filosofa no es una actitud o modo de pensar automtico en el ser humano. El proceso de expansin de la racionalidad. A un nivel prctico, la racionalidad se impuso desde el comienzo de la humanidad por sus efectos directos sobre la calidad de vida. La inteligencia humana tiene un origen y una finalidad muy humilde: sobrevivir, y sobrevivir mejor. Ser racional tiene muchas ventajas en las decisiones del da a da (en vez de invocar a la divinidad del fuego, lo importante era tener la provisin adecuada de lea para pasar la noche; en vez de pagar a un chamn para que le quite el mal de barriga al nio fumando y cantando sobre l, darle una infusin de guayabo amargo). Los humanos que resolvan mejor sus problemas (mejor tcnica) vivan ms y mejor. Un gran ejemplo de opcin por la tcnica es el Canal de riego de Toledo que deseaba construir Felipe II. A otros niveles la opcin por la racionalidad fue mucho ms difcil. En lo poltico, o sea, en gobierno y organizacin social, se impuso la ambicin y la autoridad de la fuerza. es racional la ley del ms fuerte (la ley de la jungla)? Pues no, a menos que uno viva en la jungla, y sea un animal. Slo poco a poco, a lo largo de muchos siglos y gracias a seres humanos y pueblos excepcionales, el bien comn o el dilogo tomaron la delantera. Pinsese que la democracia ateniense fue una rara y breve excepcin, al igual que la repblica romana. En el nivel ltimo, en la pregunta por el sentido global de las cosas y el origen del mundo, de las razas, de los animales, de las propias costumbres, la respuesta general al inicio de la humanidad fue el mito y la religin6. Solo poco a poco, y de manera definitiva en Grecia en los siglos X IV aC, la racionalidad asume su papel de intrprete principal de la realidad, teniendo en cuenta la diversidad de planos o niveles de lo real (es decir, sin quitar que existan simultneamente otras dimensiones en cada objeto). No se suprimi el culto a los dioses ni el gusto por la mitologa, pero esto no obstaculiz la bsqueda del sentido propio de las cosas. En qu consiste la actitud racional? Es la combinacin de dos elementos. El punto de partida es el mismo que en otras actitudes no racionales, es la famosa pregunta:
Introducimos el concepto clsico de verdad, segn lo expona Platn en Cratilo: verdadero es el discurso que dice las cosas como son; falso es el que las dice como no son (Crat. 385b; Cf. Sof. 262 c; Fil. 37 c). La definicin ms conocida es la de Santo Toms, que sigue a Aristteles: Veritas est adaequatio intellectus ad re: La verdad es la adecuacin entre el conocimiento (una afirmacin) y la realidad (un hecho) (Cf. Sto. Toms, S.Th. I, q 16 a2; Contra Gentiles I, 59; De Veritate q1 a1). 5 Un magnfico ejemplo de la dificultad de aplicacin del mtodo cientfico, que es un desarrollo organizado de esta actitud racional, es la historia del Dr. Semmelweis y el desarrollo de las medidas higinicas en los hospitales (ante las fiebres pauperales que provocaban numerosas muertes entre las madres que daban a luz). Ver el relato en :http://recursostic.javeriana.edu.co/multiblogs/filociencia.php? title=la_aplicacion_del_metodo_el_dr_semmelwei&more=1&c=1&tb=1&pb=1 6 Muchos dicen que el mito es una manera potica de afrontar el sentido de las cosas, y que es ya filosofa. Podemos decir que en cierto sentido es as, y que toda mitologa cumple la funcin de explicar los hechos que rodean al ser humano, pero todava no se da el paso definitivo de dar autonoma y libertad a la racionalidad.
4

Lgica - 6

PORQU? La novedad de la actitud racional reside en que, en vez de contentarse con las respuestas anteriores (por magia, porque siempre fue as, porque los dioses nos castigan ), responde con argumentos, es decir, racionalmente, bajo la luz de la razn humana que busca relaciones lgicas. Qu relaciones lgicas se descubren en los fenmenos y en las cosas? Son relaciones de unas cosas con otras, de un fenmeno con otro: fenmenos consecuentes: cuando se da A, despus se dar siempre B.
cuando se da el fenmeno del Nio en las costas de Per, a los pocos meses cambian las lluvias en centroamrica

causaefecto: la causa de B es A, A causar B;


la fiebre es producida por una infeccin de agentes extraos al organismo

de proporcionalidad: cuanto ms A se de, ms B tendremos;


ha subido la gasolina en Ccuta porque se est persiguiendo ms el trfico en la frontera

de simultaneidad: Ay B se dan siempre juntos;


el relmpago se da siempre acompaado del sonido del trueno.

Cada respuesta racional nos lleva a ver la realidad con una nueva luz, con nuevos interrogantes, que piden nuevas respuestas racionales. Si nos atrevemos a iniciar la bsqueda de respuestas racionales a los porqus, iniciamos una reaccin en cadena. La racionalidad tiende a expandirse abarcando cada vez ms campos. El Logos La gran apuesta del pensamiento griego es afirmar con fuerza que TODO est impregnado de sentido. Un sentido dado que proviene de la cosa misma, pero que trasciende cada elemento de la realidad y le da SENTIDO de modo completo. Este sentido de cada cosa es lo que los griegos llaman logos. El mundo, desde esta visin, es un conjunto de seres en relacin. Y su relacin se mide por ellos mismos, con su modo de ser propio que les viene de su logos. As como a nivel prctico hay una lgica prctica de la pesca o del trabajo hacha-madera, hay una lgica general, a lo bestia (a escala planetaria). No solo lo tcnico (la tecn) tiene su lgica (p. ej. 1 dobln de oro pesa 1,2 gramos; 1000 doblones deben pesar 1200gr. exactos), sino que toda realidad (el cosmos, contemplado desde la teora) tiene una lgica en ella. El logos para los griegos es la racionalidad que fundamenta la cosa real, y le da orden, armona y estructura. Este logos est en afinidad con la mente humana, que lo puede captar y valorar (los presocrticos, Platn o Aristteles se ocuparon del asunto y dieron diversas explicaciones). La palabra hablada es la casa de este sentido, por eso hay una ntima relacin entre el lenguaje y la lgica. Buscar el logos es buscar por debajo del caos aparente que dan los sentidos ese orden, armona y estructura que permite captar lo verdaderamente real (pinsese en el impacto que produca descubrir la constancia del nmero , la rotacin perpetua de la bveda celeste o las leyes del movimiento de los planetas). Al descubrir que toda la realidad es racional, tiene un logos, podemos buscar la racionalidad de cualquier realidad: el arte de la guerra, las enfermedades, la vocacin, Esta reflexin parcial es parte de la filosofa, del amor al saber, que puede llegar a formular el

Lgica - 7

sentido general de las cosas. As no slo comprendemos su sentido, sino que al mismo tiempo descubrimos su funcionamiento y las leyes que lo rigen. Esto nos lleva a una constatacin final: las ciencias (que explican cmo suceden las cosas, para poder dirigirlas) y la filosofa (que se preocupa por el porqu suceden para poder entenderlas) nacieron juntas y crecen juntas: se necesitan mutuamente. La actitud racional y otras actitudes La actitud racional es la que abre las puertas de la civilizacin. Sin esta actitud nos olvidamos de los derechos humanos y de las leyes sociales que acordamos respetar. La actitud racional ha construido realidades tan complejas como la edicin de los dilogos de Platn, las centrales elctricas o la posibilidad de que leas estas hojas. La actitud racional, cuya validez nadie discute en lo tcnico, multiplica su poder de resolucin de los problemas diarios cuando se pregunta no slo por las causas inmediatas, sino tambin por las causas ltimas, ya que la racionalidad prctica lleva a la filosofa, y sta permite reformular la tcnica y ampliar su alcance: Tcnica Filosofa Mejor tcnica Por eso, podemos afirmar al mismo tiempo que la filosofa, como expresin mxima de la actitud racional, no sirve para nada (carece de utilidad prctica), pero es el origen de la actitud que sirve para todo:

ante todo, lo que se impone es la supervivencia y la solucin a los problemas prcticos: primum vivere, deinde philosophari: primero vivir, despus filosofar7. La filosofa no sirve para cocinar alimentos o para quitarse el picor de las garrapatas: no tiene una utilidad prctica o directa. Pero cuando se adopta una actitud racional ante la vida, no se pude quedar uno en la mera supervivencia. Se impone el plantearse preguntas por el sentido de las cosas y por el ser ltimo. Rechazarlas es como romper un pacto con nuestra manera de vivir, y se deja el camino abierto a nuevas actitudes no racionales: las dictaduras, las injusticias, la corrupcin, la violencia... Hacer filosofa es situar el resto de las preguntas y abrir caminos a nuevos conocimientos, como se ha sealado antes (Tcnica-Filosofa-Mejor tcnicamejor filosofa).

Como dice el filsofo Jos Antonio Marina, la apuesta humana por una vida inteligente es muy seria: es apostar por salir de la barbarie (de la fuerza o de la supersticin), elevarse por encima de la lucha por la supervivencia8. La filosofa, que no tiene una utilidad prctica, es paradogicamente la actividad ms til para vivir con dignidad humana y alcanzar la felicidad. Concluimos. La actitud racional se sustenta sobre la fuerza de la pregunta (porqu?) y el rigor en el pensar (reflexionar, discurrir). El ambiente propio de la actitud racional es el dilogo. Cuando se entra en dilogo verdadero se asume que el interlocutor puede tener razn, y que no hay ms rbitro de las discrepancias que la fuerza de los argumentos. La racionalidad autoriza a cuestionar cualquier cosa desde el punto de vista racional, con un solo objetivo: la verdad. Quizs por esas razones se le teme y reprime en los regmenes autoritarios (pensar en los ejemplos de estos ltimos cien aos). La racionalidad exige y promueve la libertad de pensamiento y de palabra, y sta promueve la libertad de accin.
7 8

Cf. http://es.wikipedia.org/wiki/Categor%C3%ADa:Frases_y_citas_latinas Cf. MARINA Jos Antonio, El vuelo de la inteligencia, Plaza y Jans, Barcelona 52001, 24-26.

Lgica - 8

Los dos aspectos de la lgica Con todo lo que hemos explicado hasta ahora se comprende que la actitud racional (razonar sobre las cosas para hacer vida diaria, ciencia y filosofa), exige un estudio cuidadoso del razonamiento mismo y de sus leyes, para poder asegurar, a partir de algunas premisas que tenemos por ciertas, conclusiones necesariamente verdaderas. La lgica que hemos presentado as tiene una doble vertiente: una es la vertiente tcnica, que consiste en descubrir y aplicar las leyes del razonamiento correcto. Hacemos clculos lgicos, descubrimos enigmas, identificamos falacias y contradicciones, probamos argumentos, todo esto hace la lgica como ciencia que estudia el razonamiento correcto (lgica menor). La lgica es as una disciplina instrumental de la filosofa y de las dems ciencias ( organon). Gran parte de lo que trabajaremos ser razonamiento lgico, en esta lnea. La otra vertiente es la filosfica. El trabajo sobre las leyes de la argumentacin nos enfrenta ante la realidad y la razn de ser de este comportamiento segn las leyes lgicas: porqu se da el principio de no contradiccin? porqu todo parece tener una racionalidad y sentido? todo tiene racionalidad y sentido? Es el campo en el que se estudian las paradojas, las contradicciones y aporas de los sistemas lgicos. La lgica aparece entonces abriendo camino a las otras ramas de la ciencia (matemtica, astronoma, nutica,) y de la filosofa (sobre todo metafsica y gnoseologa).

Lgica - 9

Apndice al tema 1 1. Ejemplo de indagacin sobre un logos QU ES INVESTIGAR?


Xavier Zubiri Ya, 19 de octubre de 1982
La entrega del premio Ramn y Caja1 a dos grandes investigadores espaoles pone de actualidad el sentido profundo de la investigacin, tema que fue abordado ayer magistralmente por Xavier Zubiri en su discurso y que englobadesde una perspectiva profundano slo su propia actividad y la de Severo Ochoa, sino la de todos los investigadores. Por ese motivo reproducimos a continuacin, ntegro, el discurso de Zubiri. Estamos reunidos con motivo del premio Santiago Ramn y Cajal a la Investigacin, cuya significacin ha sido ya glosada aqu. Es un premio que nos lo concede, por mediacin vuestra, la sociedad espaola. Y no encuentro mejor manera de expresar mi gratitud a esta concesin que comentar en dos palabras qu es esta investigacin que tan generosamente premiis. Qu es lo que se investiga? Evidentemente investigamos la verdad, pero no una verdad de nuestras afirmaciones, sino la verdad de la realidad misma. Es la verdad por la que llamamos a lo real, ralidad verdadera. Es una verdad de muchos rdenes: fsico, matemtico, biolgico, astronmico, mental, social, histrico, filosfico, etctera. Pero, cmo se investiga esta realidad verdadera? La investigacin de la realidad verdadera no consiste en una mera ocupacin con ella. Ciertamente es una ocupacin, pero no es mera ocupacin. Es mucho ms: es una dedicacin. Investigar es dedicarse a la realidad verdadera. Dedicar significa mostrar algo, deik, con una fuerza especial de. Y tratndose de la dedicacin intelectual, esta fuerza consiste en configurar o conformar nuestra mente segn la mostracin de la realidad, y ofrecer lo que as se nos muestra a la consideracin de los dems. Dedicacin es hacer que la realidad verdadera configure nuestras mentes. Vivir intelectivamente, segn esta configuracin, es aquello en que consiste lo que se llama profesin. El investigador profesa la realidad verdadera. Esta profesin es algo peculiar. El que no hace sino ocuparse de estas realidades, no investiga: posee la realidad verdadera o trozos diversos de ella. Pero el que se dedica a la realidad verdadera tiene una cualidad en cierto modo opuesta: no posee verdades, sino que, por el contrario, est posedo por ellas. En la investigacin vamos de la mano de la realidad verdadera, estamos arrastrados por ella, y este arrastre es justo el movimiento de la investigacin. Esta condicin de arrastre impone a la investigacin misma unos caracteres propios: son caracteres de la realidad que nos arrastra. Ante todo, todo lo real es lo que es slo respectivamente a otras realidades. Nada es real si no es respecto a otras realidades. Lo cual significa que toda cosa real es desde s misma constitutivamente abierta. Slo entendida desde otras cosas que habr que buscar, habremos entendido lo que es la cosa que queremos comprender. Lo que as entendemos es lo que la cosa es en la realidad. El arrastre con que nos arrastra la realidad hace, pues, de su inteleccin un movimiento de bsqueda. Y como esto mismo sucede con aquellas otras cosas desde las que entendemos lo que queremos entender, resulta que al estar arrastrados por la realidad nos encontramos envueltos en un movimiento inacabable no slo porque el hombre no puede agotar la riqueza de la realidad, sino que es inacabable radicalmente, a saber, porque la realidad en cuanto tal es desde s misma constitutivamente abierta. Es, a mi modo de ver, el fundamento de la clebre frase de San Agustn: Busquemos como buscan los que an no han encontrado, y encontremos como encuentran los que an han de buscar. Investigar lo que algo es en la realidad es faena inacabable, porque lo real mismo nunca est acabado. La realidad es abierta y mltiple. Pero adems de abierta, la realidad es mltiple. Y lo es por lo menos en dos aspectos.

Lgica - 10

En primer lugar, porque hay muchas cosas reales, cada una con sus caracteres propios. Investigar las notas o caracteres propios de cada orden de cosas reales es justo lo que constituye la investigacin cientfica, lo que constituye las distintas ciencias. Ciencia es investigacin de lo que las cosas son en la realidad. Pero, en segundo lugar, lo real es mltiple, no slo porque las cosas tienen muchas propiedades distintas, sino tambin por una razn a mi modo de ver ms honda: porque lo que es abierto es su propio carcter de realidad. Y esto arrastra a la investigacin no de las propiedades de lo real, sino a la investigacin del carcter mismo de la realidad. Esta investigacin es un saber de tipo distinto: es justo lo que pienso que es la filosofa. Es la investigacin de en qu consiste ser real. Mientras las ciencias investigan cmo son y cmo acontecen las cosas reales, la filosofa investiga qu es ser real. Ciencia y filosofa, aunque distintas, no son independientes. Es menester no olvidarlo. Toda filosofa necesita de las ciencias; toda ciencia necesita una filosofa. Son dos momentos unitarios de la investigacin. Pero como momentos no son idnticos. Esta cuestin de qu es ser real es, ante todo, una autntica cuestin por s misma. Porque las cosas no son tan slo el riqusimo elenco de sus propiedades y de sus leyes, sino que cada cosa real y cada propiedad suya es un modo de ser real , es un modo de realidad, Las cosas no difieren tan slo en sus propiedades, sino que pueden diferir en su propio modo de ser reales. La diferencia, por ejemplo, entre una cosa y una persona es radicalmente una diferencia de modo de realidad. Persona es un modo propio de ser real. Es necesario conceptuar, pues, lo que es ser persona, es decir, hay que investigar que es ser real. Porque hay modos de realidad distintos del de cosa y persona. Cada cosa nos impone una manera de estar. Pero, adems, este concepto y esta diferencia de modos de realidad es cuestin grave. As, las personas estamos ciertamente viviendo con cosas. Pero sea cualquiera la variedad y riqueza de estas cosas, aquello en lo que estamos situados con ellas es en la realidad. Cada cosa con que estamos nos impone una manera de estar en la realidad. Y esto es lo decisivo. Del concepto que tengamos de lo que es realidad y de sus modos, pende nuestra manera de ser persona, nuestra manera de estar entre las cosas y entre las dems personas, pende nuestra organizacin social y su historia. De ah la gravedad de la investigacin de lo que es ser real. Es una investigacin impuesta por las cosas mismas. Lo que en las cosas reales se nos impone as, es justo su realidad. Esta fuerza de imposicin es el poder de lo real: es la realidad misma como tal, y no slo sus propiedades, lo que nos arrastra y domina. Por esto, el poder de lo real constituye la unidad intrnseca de la realidad y de la inteligencia: es justo la marcha misma de la filosofa. Hegel pudo escribir: Tan asombroso como un pueblo para el que se hubieran hecho inservibles su derecho poltico, sus convicciones, sus hbitos morales y sus virtudes, seria el espectculo de un pueblo que hubiera perdido su metafsica. Finalmente, investigar qu es ser real, es una tarea muy difcil. Por esto deca Platn a un joven amigo principiante en filosofa: Es hermoso y divino el mpetu ardiente que te lanza a las razones de las cosas; pero ejerctate y adistrate mientras eres joven en estos esfuerzos filosficos, que en apariencia para nada sirven y que el vulgo llama palabrera intil; de lo contrario, la verdad se te escapar de entre las manos. Platn se dedic a este esfuerzo durante toda su larga vida. Algunas veces se senta desanimado. En cierta ocasin escribi: apeireka ta onta skopon, qued desfallecido escrudiando la realidad. Una de las personas que mejor comprende esta distincin y unidad de ciencia y filosofa es mi admirado y querido amigo Severo Ochoa. Por esto, y por nuestra vieja amistad, su compaa en esta ocasin es para mi un momento esencial de este premio. Al referimos a la investigacin, vosotros habis pensado tambin en la filosofa. Es la primera vez que esto ocurre. Y yo, y conmigo todos los denodados cultivadores de la filosofa, nos sentimos con ello muy legtimamente honrados y satisfechos. Gracias en nuestro nombre.

Lgica - 11

Ejercicios del captulo 1 LGICA Y RAZONAMIENTO -I


Tras haber expuesto en clase qu es el pensamiento racional y la lgica, pasamos a verificar el aprovechamiento realizado.

1. Elabore diez preguntas sobre su entorno cotidiano, sobre las cosas que le interesan, especialmente aquello que no est funcionando. Distinga entonces aquellas que son de nivel tcnico-prctico y las que piden una respuesta ms amplia. 2. Elabore respuestas sencillas a dos de las preguntas anteriores mediante explicaciones racionales. Procure encontrar tambin dos preguntas a las que usualmente se les da respuestas mticas (autoritativas, supersticiosas, tradicionales,...). 3. Existen tambin un tipo de respuestas que no hemos comentado en la teora, aquellas que asumen lo espiritual (Porqu el ser humano tiende a apartarse del bien?) y aquellas que ponen como causa valores personales: el amor, la solidaridad (Porqu tengo que ayudar al que est en la droga?). En qu se diferencian de las respuestas mticas? Se pueden armonizar con lo racional? 4. Defina qu es: i. Logos: ii. iii. iv. Respuesta tcnica: Respuesta mtica: Respuesta racional:

5. Explique cual es la novedad de la respuesta racional 6. Es natural en el ser humano preguntarse el porqu de las cosas? 7. Es natural en el ser humano responder racionalmente a la pregunta sobre las cosas? 8. Reflexione sobre el logos de una actividad simple: la defensa del juego de futbol, el amaestramiento de animales, adelgazar, la conversin, etc. 1. Describa primero de un modo general el objeto de estudio 2. Indique cul es su lgica interna, a modo de ensayo brevsimo (un par de prrafos) 3. Refine su respuesta: indique cuales son los elementos relevantes (variables), las reglas constituyentes (organizacin) y las reglas normativas (estrategias de manejo). 4. Puede culminar con una reflexin sobre el sentido de lo que ha encontrado (PD: no se asuste, pero en ese momento ya est haciendo filosofa)

Lgica - 12

2. La filosofa y la lgica

V. 2.10

Qu es filosofa? Qu es lgica? Correccin formal y verdad real (verdad lgica) Definiciones y sus contextos Filosofa como sabidura y como ciencia Divisin del saber. Bibliografa especfica: VERNEAUX, Introduccin general y Lgica, pp. 11-28. 69-88. Trabajo: valoracin y resolucin de problemas lgicos.

Qu es filosofa? En este primer apartado nos proponemos comprender qu es la filosofa y situarla dentro del proceso de reflexin racional en el que nos hemos introducido en el captulo anterior. Ante esta pregunta, examinaremos tres respuestas. Las dos primeras son aproximaciones por distintas vas al objeto de estudio; la tercera, en clave de investigacin reflexiva, ser la ms importante. Primera aproximacin a lo que es la filosofa: etimolgica (desde el origen de la palabra). El trmino filo-sofa es un trmino griego usado en castellano y en la mayora de las lenguas sin apenas adaptacin. Viene de la unin de dos trminos: filo y sofa, y significa amor a la sabidura.
En griego: ("filos", amigo) y ("sofia", sabidura): (amor a la sabidura)(Ver apndice). Esta definicin etimolgica es un buen punto de partida, y se usa mucho para hablar de qu es la filosofa, pero sin las otras dos investigaciones, no podemos saber en qu consiste ese amor a la sabidura.

Segunda aproximacin: a travs de su origen (historia del trmino filosofa). Sabemos que se empez a usar en GRECIA, en el primer milenio antes de Cristo, como designacin de personas que buscaban la sabidura sin pretender por ello ser superiores a los dems. Llamarse filsofo tena una connotacin de humildad y de compromiso por servir a la verdad, en abierta oposicin a la postura de los sabios, los sofistas, que vivan y se jactaban de sus conocimientos.
El trmino, como se explica en el apndice, ya lo usan Herodoto y Tucdides, pero adquiere su verdadero sentido con Pitgoras y, sobre todo, Platn, en el marco de la controversia de su maestro Scrates con los que se autodenomiaban sofistas, sabios.

La tercera aproximacin o respuesta consiste en analizar cuidadosamente la filosofa tal como se ha ejercido por diversos autores y en diversos tiempos de la historia, buscando qu es lo comn y esencial en ella. Pero por desgracia hay muchos estilos de hacer filosofa, a veces enfrentados entre ellos en sus planteamientos y contenidos. Si queremos abarcar la actividad de los filsofos en general, lo que se puede llamar filosofa, concluimos en algo semejante a esto: La filosofa es una indagacin (investigacin, bsqueda) de la toda la realidad (de cualquier dimensin y de su totalidad) buscando su sentido, el porqu, por medio de la razn, con voluntad de plenitud y sistema

Lgica - 13

Esta primera conclusin, que algunos filsofos no compartiran, puede afinarse todava un poco ms. Intentando llegar a una definicin analtica de filosofa, la definicin tomista nos parece todava vlida (Cf. VERNEAUX, p. 15): La filosofa es el conocimiento de todas las cosas por sus razones ltimas, adquirido por la sola luz de la razn natural ( cognitio rerum omnium per altsimas causas, sola rationis lumine comparata). Estudiando as qu se entiende por filosofa obtenemos su sentido y su estructura autntica: su logos!. Pero hemos de darnos cuenta de un detalle curioso: si procedemos as, estamos estudiando filosficamente la filosofa, cosa perfectamente posible. En esta concepcin clsica de la filosofa destacan diversos elementos: Conocimiento. La filosofa es conocimiento, saber, sabidura; pero un conocimiento elevado, organizado y seguro (no es doxa, opinin). En el sentido clsico de la palabra es ciencia (en el sentido moderno puede ser considerado como tal pero de manera distinta a las ciencias empricas). De todas las cosas. Su campo de estudio abarca la totalidad, no est restringido a un aspecto de la realidad (como lo estn la psicologa o la fsica) (= objeto material). Por su razones ltimas. Al observar una realidad encontramos razones inmediatas que explican de qu se compone, cmo ha llegado a ser as, cules son sus propiedades o su utilidad, A eso se llama, en el sistema aristotlico, el conocimiento de las causas. As se logran ciencias de las causas inmediatas: la fsica, la retrica,; la filosofa investiga las causas ltimas, las que explican las cosas ya investigadas en sus causas inmediatas. Eso lleva a la mxima generalizacin, al nivel del ser (= objeto formal). Por la sola luz de la razn natural . La filosofa surge como hemos dicho de la actitud racional, la que acepta el reto de buscar el logos o racionalidad de las cosas. Eso restringe el conocimiento posible a aquellas realidades y bajo aquellas perspectivas que pueda conocer la razn9. Existen otros saberes que utilizan otros medios de conocimiento de la realidad: la teologa (por la fe), el conocimiento del esposo y la esposa (por connaturalidad del amor). Los diversos pensadores han hecho filosofa desde distintas perspectivas, y por tanto el trmino filosofa tiene en cada filsofo un significado distinto. Con todo, la definicin tomista parece describir adecuadamente el uso comn del concepto de filosofa. Con todo, a estas caractersticas que seala la definicin nosotros aadiramos tres cosas:

la filosofa es un saber: exige y crea una actitud de bsqueda, de humildad, de dilogo sincero; no importa tanto tener las respuestas como estar siempre preguntndose por el sentido de las cosas. la filosofa es una ciencia o (ver ms adelante) metaciencia (un conjunto de conocimientos articulados racionalmente, no unas cuantas frases geniales), la filosofa se basa en el poder de las preguntas, afrontadas racionalmente (ver tema anterior). Lo filosfico reside ante todo en las preguntas que se hace el filsofo, no tanto en las respuestas que elabora.

9 Sera importante entonces distinguir entre racional y razonable. Racional es lo que se da por la lgica interna que lleva de las causas al efecto (un exceso de peso derrumba un edificio; un conductor ebrio pierde el control de su mquina). Razonable es aquello que no contradice la razn pero no se produce de modo necesario a partir de las causas manifiestas (dos personas que congenian, de similares intereses y caracteres complementarios se casan: es razonable, pero no es racional, ya que se casan no por estos datos, sino porque se aman).

Lgica - 14

Qu es lgica? Con Aristteles nace la lgica formal o el estudio de las reglas que aseguran el razonamiento correcto. Para l y para todos los que le siguieron era una parte de la filosofa, con una utilidad propedutica o preparatoria. La lgica naci como el estudio especializado del razonamiento correcto, que es el que se va a emplear para desarrollar luego el resto de la filosofa (que en aquel tiempo inclua la totalidad de conocimiento organizado y seguro, o cientfico). La definicin del DRAE (Diccionario de la Real Academia, ed. 22) es: ciencia que expone las leyes, modos y formas del conocimiento cientfico (ver apndice). Esta definicin nos valdra con tal de eliminar el trmino cientfico, pues tambin se emplea la lgica en el conocimiento comn. La definicin clsica de Santo Toms es: El arte directivo del acto mismo de la razn para que el hombre, en el acto mismo de la razn proceda ordenadamente, con facilidad y sin error (Ars directiva ipsius actus rationis per quam sciliet homo in ipso actu rationis ordinate et faciliter et sine errore procedat)10 Sus elementos: Un arte. Un arte es una disciplina que, apoyada ciertamente en una tcnica, con sus reglas y mtodos, los supera cuando el que la realiza domina perfectamente las tcnicas y las pone al servicio de su proyecto en modo excelente. A la lgica se la denomina arte porque su objeto no es establecer las leyes del pensamiento correcto y ya est (eso sera una mera tcnica), sino que cada persona ha de emplearla para analizar sus razonamientos y crear otros nuevos (no es slo simple ciencia, aunque es tambin ciencia).

Del acto de la razn. Su objeto material es la razn, o el razonamiento, que es una parte pequea del pensamiento). Para que proceda ordenadamente y sin error. El objetivo del estudio del pensamiento no es su contenido, qu es lo que se piensa, sino la forma, en donde reposa la fuerza de la argumentacin. La lgica proporciona las leyes que descubren cundo podemos estar seguros de la correccin o validez de una argumentacin (y por tanto de que las conclusiones a las que hemos llegado son verdaderas).

Santo Toms, In Analtic. Post., lib. I, lect. 1. Citado en MARITAIN Jacques, El orden de los conceptos. I: Lgica menor, Club de Lectores, Buenos Aires 1978, 13.
10

Lgica - 15

La lgica entendida as es un estudio metdico de las reglas que aseguran la correccin de un argumento, y tiene por tanto un valor de ciencia auxiliar o propedutica para todas las dems, y tambin para la filosofa. Al mismo tiempo, empleada para verificar la correccin de los argumentos, o para descubrir las falacias, se muestra como un arte que, como todas ellas, requiere disciplina, concentracin, paciencia, preocupacin suprema por dominar el arte y prctica asidua11. Correccin formal y verdad real (verdad lgica) El lenguaje es una herramienta que nos permite comunicarnos acerca de las cosas reales que nos rodean (entre otras cosas). Cuando hacemos una afirmacin sobre algo del mundo, su valor depender de lo acertada que sea, y en ltimo trmino es verdad o mentira. La verdad es una propiedad esencial de este tipo de lenguaje afirmativo, esencial para la superviviencia del ser humano y para que logre su felicidad. Qu es la verdad? Es la adecuada correspondencia entre una afirmacin y la realidad a la que se refiere12. Si decimos: Simn Bolvar muri en Santa Marta en 1830, la afirmacin ser verdadera o cierta si una persona de ese nombre muri en esa ciudad y en ese ao. Si algn dato es incorrecto, la frase entera es mentira, no es verdad, no es cierta. Los elementos del lenguaje que podemos estudiar desde la lgica son de tres tipos: trminos (casa, blanco, hombre, mortal), juicios (esta casa es blanca, todo hombre es mortal) y argumento o razonamiento (todo hombre es mortal; Scrates es hombre; luego, Scrates es mortal). Cada tipo tiene su propia forma de ejercer el reflejo exacto de la realidad: los trminos son ms o menos Un juicio (afirmacin formada por dos : o ms trminos, en relacin) Un razonamiento (dos o ms juicios puestos en relacin, --llamados premisas de los que se obtiene otro juicio, llamado conclusin): --> precisos o vagos. --> verdadero o falso

Y los trminos de que se componen, precisos o no.

--> correcto o incorrecto

Y los juicios que componen el razonamiento son verdaderos o falsos.

Lo ms importante es que los razonamientos correctos cumplen esta ley bsica que es la que da origen a la lgica: Un argumento correcto, empleado sobre proposiciones verdaderas, da conclusin verdadera. Siempre. (un argumento incorrecto con premisas verdaderas puede dar conclusin falsa o verdadera) (un argumento correcto con premisas falsas puede dar conclusin falsa o verdadera) (un argumento incorrecto con premisas falsas puede dar conclusin falsa o verdadera)
Cf. FROMM Erich, El arte de amar, Paidos, Buenos Aires 1959, 106-108. Nuevamente es una afirmacin de Santo Toms: Veritas logica est adaequatio intellectus et rei (Summa, I :21:2).
11 12

Lgica - 16

Esto sucede porque la propiedad fundamental del argumento es deducir conclusiones de las premisas de modo acertado y necesario, debido a la lgica interna que liga las cosas y las relaciones entre ellas. Si el argumento no refleja la lgica interna de la realidad, no hay seguridad de la veracidad del resultado.
Ello nos lleva a la siguiente aplicacin prctica: si uno encuentra, en un dilogo o un debate, que le estn dando unas premisas verdaderas y la conclusin es tambin verdadera, no por eso puede estar seguro de que el razonamiento es correcto. Slo cuando nos de una conclusin falsa, si las premisas eran verdaderas, podremos estar seguros de que, en ese caso, el razonamiento era incorrecto. Por lo tanto, a lo largo del proceso de conocimiento se detectan los argumentos (y las teoras) incorrectas, pero nunca tenemos la seguridad prctica de estar usando teoras correctas, a no ser que lo demostremos con las herramientas de la lgica formal.

Definiciones y sus contextos En lo anterior hemos combinado la investigacin histrica con la filosofa tomista. Las definiciones que hemos estudiado son las clsicas del tomismo. Y debemos ser conscientes de ello para que sean de utilidad. En principio, toda afirmacin tiene un marco de referencia (todo texto requiere un contexto). Fuera de su contexto, la frase o la palabra puede malinterpretarse o resultar incomprensible. Las definiciones (y cualquier afirmacin) son relativas a su contexto. Por ello, en filosofa tenemos siempre en cuenta el contexto, en muchos sentidos: como textos y gneros literarios de los que se tom el fragmento, definicin o afirmacin que se estudia Como conjunto de saberes que se consideran aceptados y sabidos por la mayora de la comunidad como ideas generales y valores principales (creencias) que forman la base de nuestra cultura y nuestra tica Referido a este ltimo e importantsimo marco de referencia, hemos de comprender que nosotros pertenecemos a tres contextos que hemos de comprender y a los que hemos de ser fieles: un primer contexto que es el vital, en el que me he criado, el que absorb a travs de la vida de mi familia y de mis gentes un segundo contexto es el de mi opcin de vida, la vida a la que me he incorporado, con sus nuevos valores y creencias (la consagracin, la vocacin presbiteral, el estudio de la educacin, etc) un tercer contexto es el de la filosofa y cultura cristiana que yo estudio, muchas veces diversa de la cultura actual (que me llega por el contexto vital, el primero). Filosofa como sabidura y como ciencia Ver Verneaux, pp. 17-39. En sentido moderno, caracterizar la filosofa como sabidura es subrayar su carcter dinmico, de bsqueda, enfrentada a lo

Lgica - 17

inabarcable y en perpetua construccin. Pero adems pensar la filosofa como sabidura nos hace destacar el conjunto de actitudes que exige: humildad, amor a la verdad, sacrificio personal La filosofa es ciencia en cuanto saber ordenado, sistemtico y racional. Pero en sentido moderno, la filosofa slo cabe analgicamente entre las ciencias del espritu y en un papel singular, ya que no es un saber basado en los principios de verificacin o falsacin de modo sistemtico. Por eso se la suele situar ms bien sobre todas ellas, como una metaciencia. Caracterizar la filosofa como ciencia es subrayar su carcter racional, su vocacin de construccin de representaciones verdaderas y tiles de la realidad.
Hacia el final del siglo XX se ha ido subrayando cada vez ms el valor de la actitud propia de la filosofa (filosofa como sabidura). Pero los estudiantes de los cursos de filosofa tienden a estudiarla aprendiendo las ideas que desarroll cada filsofo, es decir, fijndose sobre todo en el contenido. Por eso es bueno recordar las claves de donde surge la filosofa segn Julin Maras: Piensen ustedes: si las filosofas se consideran como repertorios de doctrinas, como teoras, como sistemas de conceptos, se tiene una visin que deja fuera los estilos, que deja fuera lo que son las actitudes desde las cuales cada filsofo filosofa. Qu es problema para ellos?, qu es verdad para ellos?, qu es saber a qu atenerse? qu quiere decir entender?, con qu se cuenta?, qu es lo que importa?, qu es lo que se busca? A parte de las doctrinas, que como estamos vendo se encadenan, de cierto modo proceden unas de otras, se corrigen, se superan, hay algo fundamental que est ligado a cada persona, en su poca, en su lengua, en su pas, en sus angustias personales, en lo que realmente necesita saber para vivir. 13

Divisin del saber. Ver los diversos libros que tratan el tema (no hay una clasificacin nica, se pueden seguir varios criterios. La ms comn hoy en da es la divisin entre ciencias de la naturaleza y las ciencias del espritu.

MARAS Julin, Aristteles, en Los estilos de la filosofa. Curso de conferencias, Madrid 1999-2000 , en Internet [2006]: http://www.hottopos.com/mirand11/jmariast.htm .
13

Lgica - 18

Apndice al tema 2
Informaciones complementarias sobre lgica y filosofa 1. Filosofa en DRAE 2. En el Diccionario de la Real Academia Espaola, al registrar los significados comunes del trmino, dice:
filosofa. (Del lat. philosopha, y este del gr. ). 1. f. Conjunto de saberes que busca establecer, de manera racional, los principios ms generales que organizan y orientan el conocimiento de la realidad, as como el sentido del obrar humano. 2. f. Doctrina filosfica. La filosofa de Kant. 3. f. Conjunto de doctrinas que con este nombre se aprenden en los institutos, colegios y seminarios. 4. f. Facultad dedicada en las universidades a la ampliacin de estos conocimientos. 5. f. Fortaleza o serenidad de nimo para soportar las vicisitudes de la vida. 6. f. Manera de pensar o de ver las cosas. Su filosofa era aquella de vivir y dejar vivir.14

2. El origen de la filosofa (Wikipedia) Sobre su origen y significado, dice en Wikipedia (sept 2005)15:
La filosofa es una disciplina o campo de estudio que se encarga de la investigacin, anlisis y creacin de ideas en general, abstractas o en un nivel fundamental. La palabra "filosofa" procede del griego, estando compuesta de ("filos", amigo) y ("sofia", sabidura): (amor a la sabidura) La filosofa no puede entenderse slo como una reaccin espontnea o natural de los hombres. Por naturaleza, el hombre tiene la curiosidad de saber. Esta curiosidad por saber no es por s misma filosfica, como tampoco lo son las preguntas infantiles, ni lo era la curiosidad que pudiera experimentar un Australopithecus cuando sala de la cueva. Los orgenes de la filosofa occidental se remontan a la Grecia clsica. Etimolgicamente, la palabra filosofa, acuada por los griegos, significa amor a la sabidura. De acuerdo con Nicola Abbagnano, la palabra filosofa aparece como nombre de una indagacin. No se encuentra todava en Homero ni en Hesodo. A los sietes sabios se les llama sofistas, y sofista se llamaba a Pitgoras. Herodoto usa la palabra en su significado etimolgico cuando hace decir al rey Creso dirigindose a Soln: "He odo hablar de los viajes que filosofando has emprendido para ver muchos pases". Y Tucdides hace decir a Pericles de s mismo y de los atenienses: "Amamos lo bello con sencillez y filosofamos sin timidez". Aqu el vocablo no se refiere a una disciplina especfica. (Abbagnano, Nicola: Historia de la Filosofa., Barcelona, Montaner y Simon, 1955, pg. 4.) Segn Cicern, quien us por primera vez la palabra filosofa fue Pitgoras. l comparaba la vida con los festejos de Olimpia en los que algunos eran negociantes, otros slo iban para concursar, otros por divertirse y otros por curiosidad. A estos ltimos los llam filsofos. Otra fuente del origen de la palabra filsofo indica que Herclito, al referirse a los indagadores, los llamaba filsofos. Es necesario que los hombres filsofos sean buenos indagadores de muchas cosas (Ibid., pg. 5).
Real Academia Espaola, Diccionario de la Lengua Espaola. Vigsima segunda edicin, Madrid, Espasa Calpe - Real Academia Espaola 222001, 2 vols, ad. loc. cit. 15 Wikipedia es una Enciclopedia libre que se produce en Internet entre todos los usuarios. Su contenido se distribuye bajo licencia GNU (libre distribucin) y se distribuye en muchos idiomas: http://www.wikipedia.org/
14

Lgica - 19

Como indagacin, fue el primer significado de filosofa, que haca parte de un saber encaminado a un aprender a ser ms all de las apariencias. Por su parte, Platn le daba la denominacin de filosofa al conjunto de disciplinas tales como la Geometra, la msica, y otras que se encaminaban a su funcin propedutica. La filosofa se contrapona a la sofa muy propia de las deidades, y tambin a la dxa, vale decir, a la mera opinin del vulgo.

3. Lgica en el DRAE En el Diccionario de la Real Academia Espaola, al registrar los significados comunes del trmino, dice:
lgica. (Del lat. logca, y este del gr. ). 1. f. Ciencia que expone las leyes, modos y formas del conocimiento cientfico. 2. f. Tratado de esta ciencia. Escribi una lgica que fue muy comentada.

4. Logos en el Diccionario de Filosofa Herder logos GEN.


(del griego logos, que proviene del verbo legw<, legein, que originariamente significaba hablar, decir, narrar, dar sentido, recoger o reunir) Se traduce habitualmente como razn, aunque tambin significa discurso, verbo, palabra. En cierta forma, pues, significa razn discursiva que muestra su sentido a travs de la palabra.De los diferentes significados originarios del trmino logos -en cuanto que hablar, narrar, y reunir (como se renen las palabras para formar un discurso ordenado)-, han surgido el significado de logos como razn, entendida tanto como:a) razn matemtica: orden, proporcin, medida o b) como razn discursiva: razn que se manifiesta en el discurso ordenado de las palabras. As ha pasado a formar parte de otros vocablos para designar el orden de lo que se trata, o estudio de. Por ejemplo: bio-loga, estudio de la vida; psico-loga o estudio de la psique. Tambin da lugar al trmino lgica que designa la ciencia del saber demostrativo. Todo cuanto est regido por las leyes del logos o de la racionalidad es lgico, mientras que el discurso que las infringe es ilgico o irracional. En Herclito, el logos aparece por primera vez entendido como razn comn a todas las cosas; razn o principio csmico que expresa tanto la ley universal que rige el mundo y hace posible el orden (cosmos / kosmos) y la justicia, como expresa tambin el propio pensamiento humano. De esta manera establece una unin entre el ser y el logos, como reunin de lo ente y el ser. Dicha unin se manifiesta tambin en el pensamiento humano que, en la episteme, concibe lo ente en relacin a su ser y lo revela a travs del lenguaje. De esta manera, para Herclito, el logos unifica un orden metafsico y un orden epistmico, y preside la conducta del hombre y del cosmos, y es a la vez el mismo fuego o GDPZ ( arkh) del mundo. A pesar de que todo viene a la existencia segn el logos, ste no se manifiesta de manera evidente. Slo el sabio puede darse cuenta de que, gracias al logos, todas las cosas son una unidad. De esta manera, el logos aparece como el ser mismo que se manifiesta a travs del lenguaje, como la misma physis ( fusis) que sale a la luz (ver texto ). En la poca de los sofistas, el logos designaba ms bien el contenido del pensamiento que seala la razn de algo y las razones de una accin, y designaba tambin al conjunto del mbito de lo pensado. En este sentido, mientras el ethos designaba el mbito de lo moral, el logos designaba el conjunto de las actividades del espritu. Ahora bien, en la medida en que el logos expresaba tanto un decir como un discurrir, se engendr una cierta ambigedad y confusin entre el acto de decir y lo dicho, que fue fuente de paradojas sofsticas. Platn, basndose en la concepcin de los sofistas y de Scrates que conceban el logos como argumento, basa su filosofa en el dilogo, que lo entiende como remisin de todo lo dicho al tribunal del logos (ver dilogos platnicos). Aristteles tiende, ms bien, a entender el logos como contenido semntico, es decir, como el sentido de una palabra tal como puede

Lgica - 20

aprehenderse en su definicin. Por ello el logos nos remite plenamente a la lgica, ya que solamente tiene sentido si adopta la forma de enunciados predicativos susceptibles de ser verdaderos o falsos. De esta manera, aparece claramente la contraposicin entre el mito y el logos. Aunque el mito tambin es un discurso, se basa simplemente en la transmisin de la tradicin y su nico valor estriba en la capacidad de expresar las pasiones humanas. Pero no es fuente de verdad. No obstante, Aristteles todava confiere un cierto valor a los mitos o, mejor dicho, a las motivaciones (la admiracin) que los generaron (ver texto ).Los estoicos se remitieron nuevamente a Herclito y consideraron el logos como comn al hombre y al cosmos; razn universal, principio de orden de todo el universo, que es entendido como una unidad viviente. Por ello hablan tambin de un lgos spermatiks o razn seminal que es origen de vida y que contiene dentro de s mltiples logoi spermatikoi ( logoi spermatikoi) o formas inteligibles. No obstante, tambin siguen conservando la concepcin aristotlica del logos como anlisis semntico del discurso, y por ello distinguen entre un logos interior y un logos proferido al hablar (ver lektn). Con Filn de Alejandra el Logos pasar a ser considerado como ley moral y principio unificador de lo inteligible, intermediario entre el Creador y lo creado. Tambin Plotino y los neoplatnicos lo entendern como ser inteligible intermediario entre Dios y el mundo (ver alma del mundo). Recogiendo estas concepciones, el cristianismo acabar identificando el logos con el Verbo divino, el cual, al hacerse carne (evangelio de San Juan), se identifica con la segunda persona de la Trinidad. As, Clemente de Alejandra distingui en el logos un principio creador del mundo, un principio de sabidura (la de los profetas y filsofos), y un principio de salvacin (el Logos encarnado). De esta manera, de principio abstracto inmanente al mundo, tal como era entendido en la filosofa griega, pasa a ser considerado como realidad trascendente creadora. En la poca contempornea, Heidegger ha insistido en el significado griego originario de logos como aquello que permite desvelar el ser. El logos permite el dejar ver lo manifestado en el enunciado y, a la vez, es la razn de ser de lo que este enunciado enuncia. En cuanto que lo que se manifiesta a travs del logos lo hace por relacin a algo, el logos es tambin relacin, y de ah surge su sentido como proporcin (ver texto ). En cuanto permite el desvelamiento del ser, es lo que permite edificar una ontologa y no una mera ontoteologa (ver tambin altheia). 16

MORAT MARTNEZ, Diccionario de filosofa en CD-ROM . Copyright 1996. Empresa Editorial Herder S.A., Barcelona. Todos los derechos reservados. ISBN 84-254-1991-3.

16

Lgica - 21

Ejercicios del captulo 2 LGICA Y RAZONAMIENTO -I


Tras haber expuesto en clase qu es el pensamiento racional y la lgica, pasamos a verificar el aprovechamiento de este tema introductorio. Teora 1. Haga un mapa conceptual del captulo, situando los conceptos claves y la relacin entre ellos. 2. Explique con sus propias palabras la relacin entre la filosofa y el conocimiento cientfico. 3. Estudie los materiales complementarios del apndice y elabore su propia definicin de lgica y de filosofa, recogiendo los elementos esenciales de ambas. 4. En qu sentido se puede afirmar que la filosofa es propiamente una ciencia? 5. En qu sentido se puede afirmar que la filosofa no es propiamente una ciencia? Aplicaciones 6. Consciente de la ley fundamental de la lgica, diga qu se puede asegurar de un razonamiento que: a) Es correcto y tiene una premisa verdadera y otra falsa: b) Es incorrecto pero absolutamente todas sus premisas son verdaderas: 7. Consciente de la ley fundamental de la lgica, diga qu se puede asegurar en las siguientes situaciones: a) Si un razonamiento, en el que las premisas de partida son verdaderas, nos da una conclusin falsa podemos asegurar que: b) Si un razonamiento, en el que las premisas de partida son verdaderas, nos da una conclusin verdadera podemos asegurar que:

Lgica - 22

3 Lenguaje, Lgica y Verdad

v.2.10

Las funciones bsicas del lenguaje Las cuatro formas del discurso Emotividad y objetividad El campo de la lgica Valoracin de la verdad Falacias. Bibliografa: COPI Irving M., Introduccin a la lgica, Ed. Universitaria de Buenos Aires, EUDEBA, 181977, 34ss (seguimos totalmente el texto). CURTIS Gary N., Fallacy Files, en Internet [2007; de. or. 2001]: http://www.fallacyfiles.org/ DOWNES Stephen, Stephen Guide to the Logical Fallacies, en Internet [2007; ed. or. 1996 ]: http://onegoodmove.org/fallacy/ . Igualmente, en castellano: GARCA DAMBORENEA Ricardo, Uso de razn. Diccionario de falacias, en Internet [2007]: http://www.usoderazon.com/ ; Wikipedia: http://es.wikipedia.org/wiki/Falacia

El lenguaje es una herramienta muy compleja: la estudiamos distinguiendo funcin y forma (no mezclar ambos conceptos!). Nuestro objetivo es poder situar el estudio lgico de aquellas expresiones que nos transmiten informacin y sobre las que podemos argumentar racionalmente. Las funciones bsicas del lenguaje Paras qu se puede usar el lenguaje? Cierto, para casi todo. Pero el conjunto de oraciones que podemos construir tienen de alguna manera u otra tres funciones principales: Una buena parte de nuestro lenguaje tiene una funcin informativa: afirma (o niega) algo de la realidad. Se usa para describir el mundo y para razonar acerca de l (estos apuntes tienen una funcin fundamentalmente informativa)

La segunda funcin del lenguaje es la expresiva. Aqu entra el lenguaje potico y el lenguaje exclamativo, como las interjecciones, y su uso pretende describir o despertar vivencias en el oyente similares al parlante (los himnos de Laudes y Vsperas tienen una funcin expresiva, que nos prepara para recitar el salterio con el corazn y la mente abiertos a Dios). La tercera es la funcin directiva, que se da cuando se usa con propsito de originar o impedir una accin (las informaciones del primer tema sobre el modo de hacer los trabajos son un discurso de tipo directivo, su objetivo es que ustedes hagan sus tareas de un determinado modo).

Normalmente, el lenguaje comn combina varias de estas funciones en la misma frase (y difcilmente se encuentran textos con funcin informativa pura en las conversaciones comunes). Nuestro campo de trabajo en lgica ser el discurso con funcin informativa: porque de l podemos decir que sea verdadero o falso (esto es muy importante). De un texto expresivo o directivo no se puede decir que sea verdadero o falso; ms an, se espera que gran parte del lenguaje de los enamorados no sea informativo: te quiero tanto que te comera! me muero de amor por ti!17. Las cuatro formas del discurso Los textos de gramtica habitualmente definen la oracin como la unidad del lenguaje que expresa un pensamiento completo, y dividen las oraciones en cuatro categoras:
Intentar interpretar literalmente este tipo de expresiones no informativas es un error, que a pesar de dar lugar a enredos divertidos, es metodolgicamente incorrecto.
17

Lgica - 23

declarativas (Ayer me mordi un perro) interrogativas (No es verdad que ayer te mordi un perro?) imperativas (Tobi, murdele!) exclamativas (Ay!) No hay que confundirse con las formas de las oraciones: afirmaciones (negaciones), preguntas, rdenes y exclamaciones. Pero atencin, en qu tipo de oraciones se da el lenguaje con funcin informativa? Principalmente en oraciones de tipo declarativo. Pero a veces se da en oraciones interrogativas (como la del ejemplo anterior: No es verdad que ayer te mordi un perro?: lenguaje con funcin informativa y forma interrogativa). Eso s, no toda oracin declarativa tiene funcin informativa (puede ser una queja o un insulto). Por lo tanto, forma declarativa y funcin informativa no estn ligadas automticamente. La razn est en que la categora gramatical de la oracin no obliga a emplearla para una determinada funcin del lenguaje, ya que los seres humanos hemos creado una herramienta extraordinariamente flexible y compleja.

Emotividad y objetividad El texto de COPI insiste en que la emotividad es un componente bsico del lenguaje, que encontramos en casi cualquier expresin lingstica. Se debe a que las emociones no son un modo diverso de pensar, sino una dimensin esencial de la inteligencia humana, presente en todos los niveles de la experiencia. Las emociones juegan adems un papel fundamental en la comprensin de los otros seres humanos y en la interpretacin de la experiencia (cuando nos interesa una persona recordamos fcilmente su nmero de telfono o su historia personal; si no nos interesa, se hace casi imposible de recordar). Por otro lado, la valoracin emocional que se nos cuela en el lenguaje lleva a desacuerdos graves sobre temas en los que, con una actitud distinta, los dos oyentes estaran tranquilamente de acuerdo. Es habitual que la emotividad impida ser objetivo (valoracin de un penalti por un fan del equipo defensor y por uno del equipo atacante). El ser humano ha aprendido que para desarrollar el conocimiento cientfico (seguro, objetivo y racional), debe despojar a sus expresiones de toda carga emotiva, buscando nicamente aquellas declaraciones informativas que son pertinentes a la verdad que se busca. En lgica, por tanto, trataremos de aislar las informaciones de la valoracin emotiva . Nos concentraremos en lo que se dice sobre la realidad, y con un nico inters: es verdad? es falso?18. El campo de la lgica El campo de la lgica formal lo constituye el razonamiento. ste se realiza con algn tipo de argumentacin, deductiva o probatoria. Una argumentacin es un discurso o fragmento de lenguaje que deja claro, para el que conoce el tema, que de los datos o argumentos presentados se deriva un nuevo dato, que debe ser aceptado como verdadero por la fuerza del argumento lgicamente correcto. Para poder asegurar que un razonamiento es correcto, se debe estudiar cmo se han estructurado los elementos que lo componen; por eso aislaremos cada frase u oracin, que es el componente ms pequeo del que podemos afirmar si es verdad o mentira (que llamamos en
Atencin, en algunas lgicas modernas se difumina esta dualidad verdadero/falso, y se trabaja con otros valores (probable, % de seguridad,). Veremos algo al final del curso.
18

Lgica - 24

la tradicin aristotlica juicios); cada juicio tiene un contenido con una forma lgica: sta, la forma, es la manera en la que se combinan los elementos del juicio para reflejar una realidad; esa organizacin es la que debemos reconocer e interpretar para examinar el proceso de la argumentacin. A su vez, todo juicio contiene ideas elementales o complejas, que son los conceptos con los que describimos la realidad y sus propiedades (que en la lgica aristotlica llamamos trminos). Tambin dedicaremos un tiempo a conocer y manejar apropiadamente sus tipos y sus propiedades ms importantes, as como la tcnica de la definicin. Valoracin de la verdad Debemos clarificar que la argumentacin es correcta o incorrecta segn la forma de combinar los juicios propuestos (premisas) para obtener o probar una afirmacin final (conclusin o tesis). La correccin de la argumentacin es independiente de que cada premisa sea verdadera, falsa, o todava imposible de conocer como cierta o no. El elemento del lenguaje informativo que nos da una descripcin de la realidad es el juicio (que corresponde en el lenguaje a la oracin). El juicio es una oracin con funcin informativa, verificable. Qu es la verdad? La verdad es, segn la tradicional definicin escolstica, la adecuacin de la mente a la realidad (adecuatio mentis et re). Es una realidad dual: por una parte, una representacin de la realidad (la frase esta casa es blanca) y por otro, la realidad (la casa en medio de la colina, con sus paredes blancas y su terraza o tejado). El juicio es verdad si la realidad tiene la forma o la propiedad que se describe en la mente por medio del lenguaje. Si no hay adecuacin entre ambos, por lo que sea, el juicio es falso. Se puede llegar a la conviccin sobre la veracidad o falsedad de un juicio por dos maneras: El primero es por verificacin directa: experimentar o probar por uno mismo o por otro fiable que las cosas son as. Aquellas realidades que aparecen ya claras ante nosotros las llamamos evidentes19, de las que hemos hecho una experiencia indubitable, que es un tipo importante de certeza. El otro camino es la argumentacin. Se parte de premisas que ya se han verificado (por anteriores indagaciones), y se deduce o comprueba que sea cierto. Lo fascinante es que as podemos llegar a estar seguros de cosas de las que no hemos hecho experiencia ( las drogas son malas; no comer lleva a la muerte), o de las que la experiencia parece decir lo contrario (la Tierra es redonda; giramos alrededor del Sol) con el mayor grado de certeza posible. Este es el camino que estudia la lgica. Falacias no formales. En este apartado veremos en primer lugar qu son las falacias y porqu se usan, despus los tipos de falacias, y finalmente entraremos en la descripcin de las principales falacias no formales. Bibliografa. La fuente ms importante a nuestro alcance para estudiar las falacias es el libro de COPI, al cual seguiremos para facilitar la consulta de sus magnficas explicaciones. Por nuestra parte, hemos tomado mucho material de la web Fallacy Files20. El tema de las
Sobre lo que es evidente y lo que por el contrario debe ser probado, hay un enorme debate filosfico. COPI, Primera parte, Cap. 3. Sobre Falacias, existe un magnfico sitio web: CURTIS Gary N., Fallacy Files, en Internet [2006; de. or. 2001]: http://www.fallacyfiles.org/ . Tambin: DOWNES Stephen, Stephen Guide to the Logical Fallacies, en Internet [2007; ed. or. 1996 ]: http://onegoodmove.org/fallacy/ . Igualmente,
19 20

Lgica - 25

falacias se engloba hoy en da dentro de la disciplina denominada lgica informal, con mucha aceptacin en el mundo anglosajn21. Qu son las falacias? Se llama falacia a un razonamiento o argumentacin que no es correcta, pero que lo parece. Ese es el sentido de falaz en castellano: engaoso, mentiroso22. Al no ser correcta, la conclusin no queda probada por las premisas que se han aportado, sea cual sea el fallo en el razonamiento. Lo que las hace interesantes es que se usan con frecuencia en los debates porque, aunque no pueden probar su conclusin, son persuasivas, y bien usadas son muy convincentes (a veces ms que los argumentos probatorios). Se estudian con detalle desde tiempos de Aristteles porque es fcil caer en la tentacin de usarlas o, ms fcil an, dejarse convencer por ellas. En ingls se suele reservar el nombre de falacia ( fallacy) para el tipo de argumento falaz. Nosotros no nos preocuparemos tanto, y llamaremos falacia a cualquier argumento incorrecto que tiene cierto poder de conviccin. Aadiremos siempre qu tipo de falacia es, segn la clasificacin que presentamos, que respeta en lo posible los nombres tradicionales latinos. Para combatir una falacia, especialmente cuando se emplea en un debate, se pueden emplear diversas tcnicas: Por explicacin. Contrastar el argumento con la lista de falacias e identificar su tipo; all encontraremos informacin sobre su mecanismo de aparente prueba, y podremos convencer a otros de su falsedad. Por contraejemplo. Es la tcnica ms til. Se toma la falacia y sin modificar su forma y su estilo se aplica a otra situacin en la que quede en evidencia la falsa conclusin que provoca. Tiene un efecto devastador, y muchas veces es la nica manera de convencer a los dems de la invalidez de la argumentacin. Por denuncia. Cuando se utilizan argumentos no racionales, como el Ad hominem o ad populum, a veces se puede utilizar como herramienta para desprestigiar al oponente que ha querido huir de la argumentacin racional cayendo tan bajo como para decirnos eso o aquello que es improcedente a nuestro tema. Tipos de falacias La clasificacin de la falacia se hace teniendo en cuenta dos cosas: porqu no es correcta? Y porqu es convincente para algunos?. Desde muy antiguo se distinguen dos grupos de falacias: falacias no formales o lingsticas: se basan en caractersticas o trucos del lenguaje. falacias formales (no lingsticas): se deben a errores en la construccin del argumento, y se refutan usando las leyes que indican qu forma es correcta y cul no. El tratamiento del segundo grupo requiere conocer bien cules son las formas correctas y las leyes que las regulan. El primer grupo, sin embargo, se pueden estudiar ya con los elementos de anlisis del lenguaje que ya dominamos. Copi divide a su vez las falacias no formales en dos subgrupos:
en castellano: GARCA DAMBORENEA Ricardo, Uso de razn. Diccionario de falacias, en Internet [2007]: http://www.usoderazon.com/ ; Wikipedia: http://es.wikipedia.org/wiki/Falacia 21 Cf. GRAKE Leo, Informal Logic,en Stanford Encyclopedia of Philosophy, en Internet [2007; reeditada el 21 mar 2007]: http://plato.stanford.edu/entries/logic-informal/ . En el apartado 3 da una buena bibliografa. 22 En el DRAE: Falaz. (Del lat. fallax, -?cis).1. adj. Embustero, falso. 2. adj. Que halaga y atrae con falsas apariencias. DRAE, Ad. Loc. Cit.

Lgica - 26

falacias de inatingencia. falacias de ambigedad. El significado de cada grupo lo explicaremos en la descripcin de cada tipo23.

FALACIAS NO FORMALES (descripcin de las principales falacias no formales) a) Falacias de inatingencia Son las falacias en las que las premisas no tienen que ver con la conclusin. Hay una falta de atingencia24, de concordancia, entre lo que se aduce y la conclusin que se quiere probar. Pero la falacia obtiene su poder de conviccin de otros nexos de relacin de tipo psicolgico y emotivo, que en el fragor del debate pueden ser verdaderamente convincentes, ocultando la inatingencia lgica. 1) Argumento Ad Baculum Significado: Recurso a la fuerza. Es el argumento que pretende convencer de una conclusin (o inducir a una decisin) por la amenaza. Funcionamiento: En el debate, el interlocutor incurre en esta falacia cuando recuerda, muchas veces indirectamente, las consecuencias de una decisin, o indica como argumento las fuerzas que le apoyan. Una de las variantes es la de prender el ventilador, que consiste en la amenaza de desvelar un secreto que perjudicar a todos (viene de la muy clara expresin inglesa, when the shit hits the fan) Ejemplo: La mejor manera de trazar la calle es sin duda hacerla pasar por aquella colina (donde est mi casa); recuerden que fue mi familia la que aportar el 30% de todo el presupuesto... 2) Argumento Ad Hominem Significado: Contra el hombre. Funcionamiento: En vez de aportar argumentos relacionados con el tema que se examina, se lanzan acusaciones contra el oponente para descalificarlo. Las acusaciones pueden ser improcedentes, o referirse a errores del interlocutor en el rea que nos ocupa (pero que no debe invalidar la validez del argumento actual) Es muy corriente y, cuando se alude a problemas relacionados con el tema, eficaz en sumo grado. Ejemplo: Cmo te atreves a explicarme que es malo fumar marihuana? no me dijiste una vez que t la probaste en la universidad. La propuesta del sr. J. es muy interesante, pero me gustara que antes nos explicase la foto que le hicieron con peluca y traje de mujer el jueves en el conocido bar La Jaula de Locas. Nos dice Mr. A. que hay que evacuar la ciudad por el peligro de inundaciones, pero no vamos a aceptar sus razonamientos porque... en 1983 pronostic el desbordamiento del ro Apure y se equivoc miserablemente! 3) Argumento Ad Hominem circunstancial Significado: Contra el hombre basado en sus circunstancias Funcionamiento: Es el recurso a las cualidades positivas que se suponen en una persona por su trabajo, por su afiliacin poltica, etc, para que no se oponga o apruebe
Una fuente sencilla de definiciones y ejemplos, adems de Fallacy Files que ya hemos mencionado, la tenemos tambin en Wikipedia: http://es.wikipedia.org/wiki/Categor%C3%ADa:Falacias_l%C3%B3gicas . La versin en ingls tiene ms tipos de falacias y ejemplos. 24 DRAE: relacin, conexin, correspondencia. Ad Loc. Cit.
23

Lgica - 27

algo. La diferencia con el anterior es que en vez de desprestigiar al interlocutor ante el pblico, se le obliga a un comportamiento acorde con sus funciones o ideales. Ejemplo: Seor alcalde, estoy seguro de que no castigar nuestros fallos en los libros de cuentas de la cooperativa porque todos sabemos que su partido es totalmente favorable a ellas. 4) Argumento Ad Ignorantiam Significado: Por ignorancia Funcionamiento: Es una falacia difcil de usar y fcil de detectar. Se cae en ella cuando se defiende que algo no es X alegando que todava no se conoce con certeza que lo sea. El caso tpico sucede en los ambientes jurdicos, donde se sigue el principio de que toda persona es inocente mientras no de demuestre lo contrario (y el juez aplica en consecuencia el aforismo in dubio, pro reo25). Se aprovecha esta situacin para argumentar que, puesto que existe la duda (ignoracia de la culpabilidad), queda probada la inocencia. UN caso, segn Copi, en el que el recurso a la ignorancia puede probar la inocencia, es cuando la investigacin se ha realizado por expertos (tipo CSI). Si en un juicio afirman que no encontraron nada, se puede admitir que all no haba nada que incriminase. Ejemplo: Quieren acusarme de envenenar los perros con mis alimentos DoggyPuppy y no saben exactamente lo que les caus la muerte: qu vergenza! Me acusan siendo inocente!. 5) Argumento Ad Misericordiam Significado: Por la misericordia. Funcionamiento: Se invoca la piedad o misericordia de la persona que escucha como argumento para tomar una decisin (en vez de atender a argumentos pertinentes). Se suele aludir a las dificultades personales o cualquier situacin de sufrimiento que, siendo parte del problema, se quiere usar como escudo para que no se tenga en cuenta el anlisis racional del hecho. Ejemplo: No me ponga la multa por estacionar mal!: tengo ya muchas deudas este mes!. Miren al acusado; vean su sufrimiento, su dolor por tantas penalidades que pas de joven, por la muerte reciente de su madre y la paraplejia de sus dos hermanos menores. No merece ser declarado inocente? 6) Argumento Ad Populum Significado: Recurso al Pueblo Funcionamiento: Se apela a la sentimientos o ideas populares, para solicitar una decisin movidos simplemente por esas razones (inatingentes). El ejemplo ms tpico se observa en muchos mtines polticos, en los que se apela a las grandes ideas, defensa de la patria y su soberana, dignidad frente a la injerencia extranjera, rechazo de los malvados, para obtener un voto. Pero tambin se puede caer en esta falacia cuando se razona que hacer 'A' es popular, luego probablemente, hacer 'A' es lo correcto (es lo que se llama en medios norteamericanos Bandwagon Fallacy, sbete al vagn de la banda). Mucha gente tiende a apoyar una causa simplemente porque es la ms popular y es la que ganar.
El aforismo significa: en caso de duda (cuando no se ha llegado a probar el delito) se falla a favor del reo. Eso llevar en muchos casos a no condenar al culpable porque faltan pruebas, pero eso es mejor que legar a condenar sin plena conviccin, pues en ese caso se podra condenar a inocentes. Cf. Wikipedia, art. In dubio pro reo, en Internet [2007]: http://es.wikipedia.org/wiki/In_dubio_pro_reo .
25

Lgica - 28

Los linchamientos populares y la incitacin al desorden pblico suelen seguir esta falacia buscando la exaltacin y el sentimiento de pertenencia a la masa. Ejemplo: Llevamos demasiado tiempo callados! Han pisoteado nuestra dignidad! Nos han ignorado! Por eso... a saquear el depsito de alimentos! Me culpan por saquear el camin accidentado de la polar? Si todos haran lo mismo que yo hice! 7) Argumento Ad Verecundiam Significado: Dirigido al respeto. Invocacin de la autoridad Funcionamiento: Se invoca como razn para justificar la decisin el apoyo de una autoridad respetada. La forma tpica es apoyar la conclusin en el hecho de que tal persona, popular y sabia, as lo pensaba (La evolucin de la humanidad es un hecho natural sin intervencin de Dios, as lo pensaba Darwin ). Hay que tener en cuenta que el recurso a una autoridad en la materia no siempre es falacia, si aporta una premisa pertinente y es un experto en la materia. Citar a Bolivar para apoyar el principio de mxima suma de felicidad, en un debate sobre estilos de gobierno, es apropiado; citar una opinin suya para que se admita que el socialismo marxista es lo mejor no lo es, pues Bolivar no puede ser autoridad en un tema que no exista en su tiempo. Una variante, como un reverso del argumento ad hominem, es el de pedir la solucin 'A' porque el que la apoya es popular y exitoso (y al que apoya la solucin 'B' no lo conocen ni en su casa) Ejemplo: Dios existe y la ciencia lo admite! Fjese que Einstein deca: 'Dios no juega a los dados con el universo'!. Compre interiores B.B., sea como yo! (es un anuncio del picher grandes ligas Pepito San). 8) Non causa pro causa post hoc ergo propter hoc Significado: No hay causa para esta causa; tras esto, luego a causa de esto. Puede ser entendida como causa falsa. En ambientes anglosajones se usa el tipo general de la falacia Arenque rojo, o Red Herring26. Funcionamiento: El tipo ms sencillo, el post hoc ergo propter hoc, establece precipitadamente la causalidad de un hecho 'A' sobre otro 'B', simplemente porque tras 'A' sucedi 'B' (ver primer ejemplo). A veces se citan dos o tres repeticiones de la secuencia A-->B para demostrar errneamente la relacin causa-efecto. Otras veces se presentan hechos que tienen una relacin casual, fortuita, con la consecuencia. El poder de convencimiento viene de la supersticin, de errores de comprensin, o de paralelismos con otros fenmenos donde si hay relacin causaafecto. Es importante no ampararse gratuitamente en este tipo de falacia. En sentido general, toda falacia da una premisa improcedente. La falacia se aplica cuando se afirma expresamente la relacin causal de 'A' sobre 'B'. Ejemplo: En cuanto se aprob la ley de tierras se produjo un terremoto El terremoto fue un castigo de Dios por aprobar esa ley injusta. Te mordiste accidentamente la lengua porque antes insultaste a tu hermana. ves? Llovi porque cantaste mal!

26

Cf. Red Herring's fallacy, en Internet[2007]: http://www.fallacyfiles.org/redherrf.html

Lgica - 29

9) Pregunta compleja Significado: Evidente de por s Funcionamiento: La pregunta compleja no es propiamente una falacia retrica, sino una forma de inducir a decir algo que no est en la mente del interlocutor (debera estar en la lista de los timos o engaos). Se realiza en un dilgo acostumbrando al oponente (ms bien la vctima) a responder una serie de preguntas sencillas (el gancho) hasta llevarlo a una pregunta compleja que presupone un hecho anterior. En ese momento, responder 'S' conlleva aceptar una responsabilidad; decir 'NO' significa admitir el antecedente. Ejemplo:Edad?; es usted venezolano? tiene Cdula? es ud. un buen ciudadano? dej ya de fumar marihuana? b) Falacias de ambigedad Son las falacias en las que las premisas juegan con la variacin de los trminos (ambigedad en el sentido, variacin de su extensin). Son muy frecuentes y algunas veces difciles de detectar. 1) El equvoco Significado: Por trmino equvoco. Funcionamiento: La falacia se apoya en el uso de dos significados diferentes de un trmino, poniendo entonces en relacin dos situaciones distintas que no tendran nada que ver entre s. Usado en los chistes, insultos y en las insinuaciones. Ejemplo: Cmo no va a oler mal en casa de los X, si viven al pie de la montaa, y todos sabemos los pies huelen y mucho (pie de persona y pie de montaa son trminos relacionados pero tratados aqu equvocamente: el pie de una montaa es su base, y no tiene nada que ver con que los pies humanos suelan tener mal olor). 2) La anfibologa Significado:dualidad de significado (de la frase). Funcionamiento: Es un hecho corriente en el lenguaje escrito, pero poco usada como falacia. La anfibologa es la diversidad de significados que puede tener una frase al estar estructurada de manera ambigua. Ejemplo: No me extraa que te mordiera el perro de Juan: siempre fue un mal bicho (no se puede deducir de la construccin gramatical si se refiere al animal que posee Juan o al propietario mismo) 3) El nfasis Significado: Insistencia en una parte del discurso Funcionamiento: Es una falacia muy comn en la prensa escrita y en ciertos discursos populares. Se pretende equivocar al oyente mediante el refuerzo de una parte del discurso, que al quedar sin la otra parte cambia de significado. Es el tpico caso del ttulo y subttulo. Los contratos que trasladan informacin importante a la letra pequea son igualmente falaces. Ejemplo: gran titular de un peridico NOS INVADE ESTADOS UNIDOS / con sus alegres modas deportivas 4) La composicin Significado: Unin de ejemplos (para hacer una ley).

Lgica - 30

Funcionamiento: Es la falacia que se comete cuando una persona presenta un ejemplo y de ah pasa a afirmar una ley general. En algunos casos es posible que todo el grupo responda al comportamiento que se ha recordado, pero con mucha frecuencia, se quiere hacer olvidar la diversidad, o incluso las excepciones. Ejemplo: No te puedes fiar de los vendedores de repuestos a domicilio: a m me han estafado dos veces!. Es usted racista, como todos los espaoles: no vio en la TV como un espaol le pegaba a una niita sudamericana en el metro de Madrid? 5) La divisin Significado: Divisin (y aplicacin) de una propiedad del conjunto a cada una de sus partes. Funcionamiento:Si 'A' tiene la propiedad 'P', entonces, se afirma que cada parte de 'A' tiene esa propiedad, siendo as que la propiedad slo se aplica estrictamente al conjunto, no a todos y cada uno de sus individuos. Ejemplo: El universo tiene quince mil millones de aos; el universo est compuesto de molculas; luego, cada molcula del universo tiene quince mil millones de aos; mtalo! no te puedes fiar de los lobos, tienen instinto asesino (tomado de fallacyfiles; el truco est en que la propiedad de edad o ferocidad posible no est distribuida a todas las molculas o todos los lobos).

Existen otras muchas falacias no formales, en las que se pretende engaar al oyente con argmentos incorrectos pero sugerentes. Como ejemplo de falacias de otro tipo, vanse, por poner un par de ejemplos, la falacia Monte Carlo o de la apuesta27, o la Red Herring.

Glambler's fallacy, en Internet [2006]: http://www.fallacyfiles.org/gamblers.html . Consiste en seguir la idea comn de que, en un juego de azar verdadero, cuando se da un resultado ( sali rojo tres veces seguidas), la probabilidad de que vuelva a salir otra vez ms disminuye considerablemente. La teora de la probabilidad y los experimentos prcticos nos demuestran que no es as, que cada jugada es independiente de las anteriores. Existe su versin opuesta, la de los que creen en similares circunstancias que tras una cadena de resultados (salieron tres ases seguidos), seguir la racha de buena suerte.
27

Lgica - 31

Apndice al tema 3:
Lenguaje y argumentacin28 Las falacias no formales Hasta ahora hemos asimilado el estudio de la lgica a la investigacin de las condiciones formales que debe cumplir el razonamiento para ser considerado correcto. La nocin de validez es una nocin formal que es decisiva en el estudio de la lgica e identificar lgica y lgica formal es aceptable. Sin embargo, la lgica clsica estudi tambin razonamientos que son incorrectos, pero que pueden ser persuasivos, es decir, convencer, cuya falla no reside en cuestiones que hacen a la forma de los mismos, sino que se origina en su contenido o materia. A estos razonamientos incorrectos, que pueden llegar a ser persuasivos y cuya incorreccin no puede detectarse por reglas formales, se los llama falacias no formales. Por ejemplo, consideremos el siguiente pasaje de la Apologia de Scrates, de Platn, en el cual Scrates se defiende de una acusacin que se le haba hecho y trata de probar su inocencia.

... lo que yo tendra que decir para defenderme se reduce ms o menos a estas cosas y quiz otras del mismo tenor. Es posible que alguno de vosotros sienta indignacin al acordarse de su propia conducta si, al sufrir un proceso, y mucho menos grave que ste, rog e implor a sus jueces con abundantes lgrimas, hizo comparecer a sus hijos pequeos para inspirar la mayor compasin posible, y a otros de sus familiares, y a muchos amigos, y en cambio yo no har nada de eso, por lo que se ve, y eso que estoy corriendo, como le parecera a l, el postrero y mximo peligro. Es posible que alguno, al pensar estas cosas, ms se encone contra m y que, irritado por esos mismos motivos, deposite con ira su voto. Y digo, si alguno de vosotros experimentara tales sentimientos, pues por mi parte no lo creo; pero si no obstante hubiera alguno, me parecera hablarle como es debido dicindole: 'Yo, buen hombre, tambin tengo, por supuesto algunos familiares [... ] y tambin hijos, tres, uno ya jovencito, los dos restantes nios todava. Sin embargo, no he hecho comparecer a ninguno de ellos para rogaros me absolvis. Apologa de Scrates, Platn, S. IV a.C., EUDEBA, Bs. As., 1966. Podemos considerar que en este pasaje Scrates trata de argumentar en favor de su inocencia, pero que lo hace a partir de lo que se llama apelacin a la piedad o la misericordia ; se incurre en esta falacia cuando se trata de conmover los sentimientos de alguien para que acepte una proposicin, en este caso, la que afirma la inocencia de Scrates. Lo particular de este caso es que en el texto se incurre en la falacia al mismo tiempo que se afirma que no se va a acudir a ella ("... yo no har nada de eso..."). En otros casos, se comete la falacia de apelacin a la autoridad, como en el siguiente texto:

Ren Favaloro es un gran cirujano y dice que la poltica sanitaria del gobierno est equivocada. As que esta poltica es errnea. Se comete esta falacia cuando se acude a la autoridad de una persona que cuenta con prestigio para establecer la verdad de una proposicin. En publicidad, muchas veces se acude a esta falacia cuando un deportista popular dice: " Este es el whisky que tomo yo" o "Este es el modelo de auto que yo uso ",etc., para sugerir que se trata de buenos productos. Un tercer tipo de falacia no formal es la apelacin a la fuerza, que puede ser ms o menos sutil o desembocada. El siguiente es un ejemplo de esta falacia: SANTIAGO, Chile, 13 (AFP). - El general Augusto Pinochet, comandante en jefe del Ejrcito, dijo que 'de los derechos humanos me preocupo yo. Tengo 80 000 hombres armados' para enfrentar posibles juicios contra militares por violaciones a los derechos humanos, inform la revista Hoy. Los juicios deberan enfrentarse con argumentos y pruebas y no con 80 000 hombres armados. Este texto est tomado de Internet, de la excelente pgina El buho fueguino, del Liceo de Tierra de Fuego (extremo sur de Argentina). Desgraciadamente, a partir del 2007 la pgina est fuera de circulacin. Su direccin original era [2006]: http://www.orbita.starmedia.com/~buhofueguino
28

Lgica - 32 El argumento ad hominem (literalmente contra el hombre") es aquella falacia en la cual, en lugar de dar razones contra una determinada afirmacin, lo que se hace es atacar a la persona que la sostiene. Por ejemplo: El jefe del estado mayor dice que el pas est indefenso y reclama un aumento del presupuesto militar. Pero, claro, l es militar. Aqu se descalifica la afirmacin "el pas est indefenso" por la circunstancia de que quien la hace es militar y supuestamente se beneficiara con el aumento del presupuesto. El argumento ad hominem es la contracara de la falacia de apelacin a la autoridad, pues en la segunda se intenta persuadir acerca de la verdad de una proposicin apelando al valor de la fuente, mientras que en el primero se trata de persuadir sobre la falsedad de una proposicin sealando algo negativo en la fuente de la misma. En ambos casos, en lugar de presentar elementos para discutir la verdad o falsedad de una proposicin, se busca inferir su verdad o falsedad a partir de considerar la fuente de la que procede la proposicin. Otra falacia no formal en la que se incurre muchas veces es la llamada apelacin al pueblo. En la misma se pretende sostener la verdad de una proposicin tratando de despertar las pasiones o emociones de un grupo de personas o se sostiene que "todos" estn de acuerdo con la misma. Es la falacia de los demagogos. Por ejemplo:

El seor ministro ha dicho que los nuevos impuestos servirn para terminar con el dficit cuasi fiscal y con el circulante inflacionario. Pero nosotros somos gente sencilla, qu sabemos del dficit cuasi fiscal? Qu sabemos del circulante inflacionario? Lo que s sabemos es que otra vez nos quieren meter la mano en el bolsillo. As que esos nuevos impuestos no deben ser aprobados. En este caso, se apela a la ignorancia ("qu sabemos ... ?") y a expresiones fuertemente emotivas ("meter la mano en el bolsillo"), para descalificar la afirmacin "... los nuevos impuestos servirn..." Otra falacia no formal, que puede ser considerada una variedad de la ad hominem, se conoce con el nombre de t tambin y consiste en impugnar una afirmacin sobre la base de sealar que quien la sostiene ahora no lo ha hecho en el pasado o en otras circunstancias. Por ejemplo:

Ahora que estn en el gobierno, los conservadores dicen que los nuevos impuestos ayudarn a disminuir el dficit fiscal, pero cuando eran minora se oponan sistemticamente a cualquier nuevo impuesto. As que estos impuestos no deben ser votados por nuestra bancada. En lugar de discutir la proposicin "los nuevos impuestos ayudarn a disminuir el dficit fiscal", se dice a quienes la sostienen, que en el pasado no lo hacan. Este hecho puede servir para mostrar cierta inconsecuencia en las posiciones del partido, pero no demuestra la falsedad de la proposicin. Se llama apelacin a la ignorancia a la falacia que consiste en sostener que una proposicin es verdadera porque no se ha probado que es falsa o que es falsa porque no se ha probado que es verdadera. Por ejemplo:

Los extraterrestres no existen dado que nadie ha podido probar que los haya. Hay, sin embargo, un mbito, el de la justicia, en el cual se parte de la presuncin de inocencia, es decir, de que nadie es culpable hasta que se haya demostrado lo contrario. La falacia de causa falsa se la comete cuando se toma por causa de un efecto algo que no lo es o que lo es slo parcialmente. Por ejemplo:

Hoy su 'esperanza de vida' ha crecido significativamente en nuestro pas, gracias al constante esfuerzo de nuestros mdicos y al aporte de la actualizada gama de medicamentos que fabrica la industria farmacutica. El aporte de los mdicos y los medicamentos al crecimiento de la "esperanza de vida", slo puede considerarse un factor entre otros. Se comete la falacia de peticin de principio o razonamiento circular cuando se toma como premisa en un razonamiento la misma proposicin que se pretende demostrar. Por ejemplo:

Lgica - 33

Mi partido es hoy la alternativa de gobierno ya que el mismo plantea una poltica distinta del partido oficialista. Hay una gran cantidad de falacias no formales que han sido descriptas desde Aristteles en adelante. Las enumeradas slo constituyen una muestra de las ms frecuentes. Lo comn a todas ellas es que aunque psicolgicamente pueden ser persuasivas, las premisas no son lgicamente atingentes a la conclusin. Debe observarse que a veces no es sencillo determinar si un cierto texto constituye o no una falacia, ni tampoco identificar con claridad a cul de los tipos enumerados pertenece. Esto es as porque, a diferencia de las falacias formales que tienen un tratamiento casi matemtico, en las falacias no formales debe realizarse un anlisis de tipo semntica y pragmtico de los trminos y expresiones involucradas. Actividades Explicar las siguientes falacias no formales y sealar al grupo que con mayor probabilidad pertenece cada una. 1 . Hoy me toca a m patear los penales. A fin de cuentas, es mi pelota. 2. Tenemos que aceptar que hay fenmenos telepticos porque hasta ahora nadie ha podido probar que no existen. 3. Aristteles, Santo Toms, Descartes, Hegel y gran parte de los grandes filsofos afirmaron la existencia de Dios. As que Dios existe. 4. Mi prueba merece una mejor calificacin. La semana anterior a la misma estuve enfermo y ni pude asistir a clase ni tampoco estudiar, por eso mi rendimiento aunque bajo merece una mejor calificacin. 5. Dios existe, porque la Biblia as lo dice, y sabemos que lo que la Biblia nos dice debe ser verdadero porque es la palabra revelada de Dios. 6. Buenos Aires, 23 (Tlam). La Polica Federal Argentina difundi ayer un comunicado con los antecedentes del oficial inspector retirado R.P.F., autor de un testimonio sobre 'la estructura de la represin ilegtima en la Argentina' presentado ante la Comisin de Derechos Humanos. El comunicado, que lleva la firma del jefe de Polica, acusa a R.P.F. del "delito de estafa y "abandono del hogar", entre otros ilcitos. 7. Buenos Aires, 14 (DYN). En un comunicado que firma el vicepresidente del PJ se destaca que el ex senador radical no puede insinuar supuestos acuerdos con la UCEDE, ya que representa a un partido que ha hecho del negocio su prctica poltica permanente. 8. Buenos Aires, 16 (DYN). La prisin preventiva por "desacato agravado' del ex presidente Ral Alfonsn fue resuelta ayer por un juez de Ro Gallegos. Al tomar conocimiento de la medida judicial el ex jefe de Estado reaccion anoche con indignacin: 'En la Argentina de Menem, dijo, Firmenich y Videla estn libres y parece que a Alfonsn lo quieren encarcelar. Ms adelante Alfonsn agreg: "Ustedes saben la filiacin poltica del juez de Ro Gallegos". 9. El ex lder de la Repblica Democrtica Alemana acusado por la muerte de 200, personas no debe ser juzgado pues es un hombre anciano que pas diez aos de su vida en una crcel nazi. 10. Debe concederse esta excepcin a nuestra empresa pues la misma siempre ha colaborado con su partido y no nos gustara tener que suspender dicha colaboracin. Buscar en diarios y revistas ejemplos de falacias no formales y explicarlas.

Lgica - 34

Ejercicios del captulo 3 Funciones del Lenguaje - I


1. Cules son las tres funciones principales del lenguaje? 2. Cul es la propiedad bsica de las oraciones que pertenecen a cada una de esas funciones? 3. Aplique su razonamiento indicando y explicando las funciones de las siguientes frases (y entre parntesis su forma) a) Nunca ms comer carne de animales sacrificados! b) Mi abuelo emigr cuando era joven y all consigui reunir el suficiente dinero para montar un pequeo negocio y casarse con mi abuela. c) Vulvete ahora mismo por donde has venido! d) No es verdad que este ao hay muchsimos huracanes, ms de lo normal? e) Dme donde est el tesoro del pirata Jack Sparrow! f) Mi abuela naci en Canarias pero tuvo que criarse con unos parientes de Aragn, muy lejos de donde naci.

g) Puah! Qu asco! h) Me puedes hacer el favor de pasarme el queso, por favor? i) j) Tanto nos am Dios que entreg a su Hijo por nuestra salvacin! Mi abuela tuvo un noviazgo muy romntico: se carte con mi abuelo mientras l estuvo en Mxico, y slo al final, cuando regres con algo de dinero, le pidi matrimonio.

k) Todo lo hice por amor. l) Me puedes pasar por favor la bandeja de las arepas?

Lgica - 35

Funciones del Lenguaje - II


El siguiente texto de La vida es sueo del gran Caldern de la Barca (acto primero) est centrado en la desesperacin de Segismundo, condenado a la mazmorra por un orculo que atemoriz a su padre Basilio. Determine cuales son las oraciones y su funcin (informativa / expresiva / directiva
) Dentro SEGISMUNDO SEGISMUNDO:Ay, msero de m, y ay infelice! ROSAURA: Qu triste vos escucho! Con nuevas penas y tormentos lucho. CLARN: Yo con nuevos temores. ROSAURA: Clarn... CLARN: Seora...? ROSAURA: Huyamos los rigores de esta encantada torre. CLARN: Yo an no tengo nimo de hur, cuando a eso vengo. ROSAURA: No es breve luz aquella caduca exhalacin, plida estrella, que en trmulos desmayos pulsando ardores y latiendo rayos, hace ms tenebrosa la obscura habitacin con luz dudosa? S, pues a sus reflejos puedo determinar, aunque de lejos, una prisin obscura; que es de un vivo cadver sepultura; y porque ms me asombre, en el traje de fiera yace un hombre de prisiones cargado y slo de la luz acompaado. Pues hur no podemos, desde aqu sus desdichas escuchemos. Sepamos lo que dice. Descbrese SEGISMUNDO con una cadena y la luz vestido de pieles SEGISMUNDO:Ay msero de m, y ay infelice! Apurar, cielos, pretendo, ya que me tratis as, qu delito comet contra vosotros naciendo. Aunque si nac, ya entiendo qu delito he cometido; bastante causa ha tenido vuestra justicia y rigor, pues el delito mayor del hombre es haber nacido. Slo quisiera saber para apurar mis desvelos --dejando a una parte, cielos, el delito del nacer--, qu ms os pude ofender, para castigarme ms? No nacieron los dems? Pues si los dems nacieron, qu privilegios tuvieron que no yo goc jams? Nace el ave, y con las galas que le dan belleza suma, apenas es flor de pluma, o ramillete con alas, cuando las etreas salas corta con velocidad, negndose a la piedad del nido que dejan en calma; y teniendo yo ms alma, tengo menos libertad? Nace el bruto, y con la piel que dibujan manchas bellas, apenas signo es de estrellas --gracias al docto pincel--, cuando, atrevido y cruel, la humana necesidad le ensea a tener crueldad, monstruo de su laberinto; y yo, con mejor instinto, tengo menos libertad? Nace el pez, que no respira, aborto de ovas y lamas, y apenas bajel de escamas sobre las ondas se mira, cuando a todas partes gira, midiendo la inmensidad de tanta capacidad como le da el centro fro; y yo, con ms albedro, tengo menos libertad? Nace el arroyo, culebra que entre flores se desata, y apenas sierpe de plata, entre las flores se quiebra, cuando msico celebra de las flores la piedad que le dan la majestad del campo abierto a su huda; y teniendo yo ms vida, tengo menos libertad? En llegando a esta pasin, un volcn, un Etna hecho, quisiera sacar del pecho pedazos del corazn. Qu ley, justicia o razn negar a los hombres sabe privilegios tan suave excepcin tan principal, que Dios le ha dado a un cristal, a un pez, a un bruto y a un ave?

Lgica - 36

LGICA IUESTA CBI Semestre I Ejercicio N 2 07/11/07

FALACIAS
Tras haber estudiado los distintos tipos de falacias y haber comprendido la naturaleza del engao al que inducen como falsos argumentos, explique de cada uno de estos casos: a) Si es una falacia, de qu tipo es? b) Porqu? (En qu se basa para intentar probar una conclusin?) NOTAS: Recordar las exigencias de todo trabajo en la materia (ver guin inicial) Extensin mxima: una hoja dos pginas (unas tres-cinco lneas mximo por falacia) Se valorar especialmente el razonamiento con el que el alumno seala y demuestra la falacia, no tanto la clasificacin. a) Aunque el doctor Ramrez te dijo que la enfermedad es grave y te aconsej que te operases, dado que es un cirujano y l te operara, tiene claros intereses en la materia (gana mucho dinero con cada operacin). Por lo tanto, su opinin puede ser tranquilamente ignorada

b) Ver, ya s que usted opina que la iglesia se debe pintar de blanco y tiene buenos argumentos, Padre, pero yo sigo opinando que el color crema le sentara mucho mejor, y, como principal donante en la campaa de financiacin de las obras, seguro que comprender cul es la decisin verdadera en este asunto

c) Ya, usted dice que debo dejar de beber porque es malo para m salud, pero se da
cuenta de que cualquier ser vivo que deja de beber se muere?

d) Las Nuevas Tribus no deben ser expulsadas, puesto que los que estn a favor de esta medida son personas muy interesadas en atacar la religin, con una manifiesta militancia en ideas contrarias a la moral y de vida privada muy dudosa

e) Yo conoc un cura en el pueblo de al lado que tena una doble vida, y que dej un grave escndalo despus de varios aos de vivir all; por tanto, comprender que no me fe de ningn sacerdote

f)

Las Nuevas Tribus deben ser expulsadas, puesto que hasta el mismsimo Simn Bolivar, con todas sus ideas, se manifest catlico y obediente a la Iglesia Catlica

g) Cada vez que hemos tomado una foto del curso alguno ha abandonado el seminario: luego, la mejor forma de evitar que hay abandonos es no hacer fotos de curso

Lgica - 37 h) Se ha podido probar absolutamente que los miembros de las Nuevas Tribus practiquen el espionaje y difundan doctrinas dainas para los indios?, Luego los expulsan injustamente, porque son totalmente inocentes de esa acusacin.

Lgica - 38

FALACIAS -III
NOMBRE:________________________________________ Fecha: _______ Seguimos con las falacias. Este ejercicio est destinado a convertirle a usted en un maestro en la deteccin y denuncia de argumentos falaces. Se le pide que desarrolle su propio cuadro de falacias recopilando aquellas que va descubriendo en la vida ordinaria. Elaborar su propia definicin

FALACIAS NO FORMALES a) Falacias de inatingencia 1) Argumento Ad Baculum Definicin: Ejemplo: 2) Argumento Ad Hominem Definicin: Ejemplo: 3) Argumento Ad Hominem circunstancial Definicin: Ejemplo: 4) Argumento Ad Ignorantiam Definicin: Ejemplo: 5) Argumento Ad Misericordiam Definicin: Ejemplo: 6) Argumento Ad Populum Definicin: Ejemplo: 7) Argumento Ad Verecundiam Definicin: Ejemplo: 8) Non causa pro causa post hoc ergo propter hoc Definicin: Ejemplo: 9) Pregunta compleja Definicin: Ejemplo: b) Falacias de ambigedad

Lgica - 39

1) El equvoco Definicin: Ejemplo: 2) La anfibologa Definicin: Ejemplo: 3) El nfasis Definicin: Ejemplo: 4) La composicin Definicin: Ejemplo: 5) La divisin Definicin: Ejemplo: c) Falacias de otro tipo (las que usted encuentre en sus investigaciones)

Lgica - 40

La Lgica estudia la razn misma en cuanto que es instrumento de la ciencia o medio de adquirir y poseer lo verdadero. Jacques MARITAIN, El orden de los conceptos, 13.

4 Historia de la Lgica

V. 01

A) La Lgica clsica: Los sofistas y sus enredos Aristteles: el Organon Tras Aristteles: Crisipo de Soli, Porfirio, Boecio La escolstica aristotlica Duns Scoto y Ramon Llull La tentacin nominalista B) El despliegue de las lgicas modernas: Leibniz de Gottlob Frege y Russell, Carnal y Wittgenstein, Hempel y Popper, Hilbert y Gdel Lgicas no clsicas: intuicionismo, Lukasiewicz, lgica borrosa y lgica cuntica Panorama actual terico y prctico. Bibliografa especfica: CRYAN Dan SHATIL Sharron MAYBLIN Bill, Lgica para todos, Paidos, Barcelona 2005 ; Wikipedia, artculo Lgica (se han hecho contribuciones desde esta fuente, no al revs); GARRIDO Manuel, Lgica simblica., Cuarta edicin, Tecnos, Madrid 42001, pags. 501-535.

AVISO: CAPITULO EN REDACCIN

En este captulo pretendemos ofrecer una panormica rpida y necesariamente superficial de las personas y grupos que pe3nsaron y desarrollaron el arte de la lgica. La visin histrica se suele obviar pensando que lo importante es aprender reglas y resolver casos. Sin embargo, olvidar la dimensin histrica de una ciencia, y mucho ms de un arte, es perder una parte importante de su esencia. El que la aprende as, como un conocimiento atemporal se condena a encerrarse en lo poco que aprendi y a repetirse en formas y contenidos. La visin histrica de esta disciplina quiere despertar en el estudiante dos cosas: una, la empata con las personas que, como l, se han preguntado antes cmo asegurar la validez de ciertos razonamientos y la incorreccin de otros. En segundo lugar, al situar histricamente los desarrollos de la lgica que vamos a estudiar, podremos adquirir la verdadera perspectiva de cada avance: su contexto vital, el dilogo de pensamiento al que responde, y la utilidad que ofreca. Dividimos la lgica en dos grandes perodos: la lgica clsica, centrada en los silogismos, y la lgica moderna, a partir de Leibniz y la preponderancia de la lgica proposicional (otra divisin ampla el perodo de la lgica clsica hasta inicios del siglo XX y reserva la denominacin de lgica moderna para las lgica axiomticas y las que van ms all del valor dual verdadero/falso ver ms adelante). Por ltimo hay que decir que la historia de la lgica se desarrolla conjuntamente con la historia de la filosofa y la de la matemtica.

Lgica - 41

A) La Lgica clsica Los sofistas y sus enredos

El ser humano ha empleado desde el inicio ciertos razonamientos racionales en cuestiones tcnicas e inmediatas, aplicando patrones de causa efecto y de proporcionalidad (si con cinco trampas cazo dos conejos al da, preparando otras cinco ms conseguir hasta cuatro cada da). El principio de no contradiccin tiene un amplio uso en deducciones inmediatas. El estudio de los principios de los razonamientos, lo que nosotros conocemos como lgica, se desarrolla solamente a partir del siglo IV a. C., y en tres culturas distintas: China, India y Grecia. Cada una hace un desarrollo diverso. La tradicin china, que se inicia con Confucio (+479 aC) y Mozi (+390 aC), desarroll interesantes estudios de la argumentacin, por ejemplo en la Escuela de los nombres o Lgicos. Sin embargo, tras la unificacin del imperio chino al inicio de la dinasta Qin, el consejero Li Si convenci al emperador Qin Shi Huang para que eliminara toda oposicin intelectual a la corriente que le apoyaba, el legalismo, y as, entre 213 y 206 aC se desarroll una feroz persecucin en la que se quemaron todas las obras de las cien escuelas de pensamiento29 y fueron enterrados vivos entre 400 y 700 estudiosos. Es posible que esta represin acelerara la cada esta dinasta (hacia 206 aC) y su reemplazo por la dinasta Han, oficialmente confucionista. La lgica en China se perdi hasta la llegada de la lgica india con la expansin del Budismo. En la India dos de las escuelas oficiales del hinduismo tuvieron que ver con la lgica. El libro Rig Veda contiene ya especulaciones sobre el principio de identidad y no contradiccin (s. X aC) y en el s. VI surge ya la escuela anviksiki, citada en el Mahabharata. La escuela Nyaya desarroll una fuerte bsqueda del conocimiento verdadero por motivos filosficos, que plasm en su mayor obra, las Nyaya Sutras (hacia el s. II aC). En ella explican la inferencia (anumana) en distintas situaciones y plantean un proceso de inferencia en cinco pasos que tocan lo particular, lo general y la aplicacin concreta. Eso les permiti desarrollar un estudio del error y de la las falacias. En el siglo II dC el filsofo budista Nagarjuna desarroll el Tetralema, una forma de razonamiento similar al cuadro de oposiciones con dos trminos. Posteriormente se desarrollaron otras escuelas de lgica budista (s. V s. XIII) y de nueva escuela Nyaya (s. XIII s. XIX), que ayudaron a comprender la necesidad de superar los esquemas de lgica aristotlica. En Grecia el inters por la lgica formal se origina al mismo tiempo que la palabra, el dilogo, adquiere un papel preponderante en el gobierno y en la toma de decisiones en general. Es el tiempo en el que nace la filosofa, ofreciendo mltiples interpretaciones de la realidad que dan primaca a los descubrimientos de la reflexin ms all de lo que presentan los sentidos (de ah el concepto de verdad como aletheia, desvelamiento). La escuela pitagrica (Pitgoras, 582-496 aC), Herclito (+ 475 aC), Parmnides (h. 440 aC), y la escuela elata son algunos de los filsofos que hicieron reflexiones filosficas sobre cuestiones lgicas. Sin embargo, el gran impulso para el desarrollo de la lgica formal fue la creciente popularidad de los sabios griegos. provenientes de diversas escuelas, tenan en comn su inters por las cuestiones humanas, a las que con frecuencia daban soluciones de conveniencia, a veces contradictorias entre s. Un claro ejemplo son las aporas de Zenn, que opona al sentido comn el razonamiento que mostraba las contradicciones del movimiento y el cambio en general.
Cf. para este interesante ejemplo de represin intelectual, las informaciones que aporta la versin inglesa de wikipedia: Wikipedia, art. Burning_of_books_and_burying_of_scholars, en Internet [12 nov 2007]: http://en.wikipedia.org/wiki/Burning_of_books_and_burying_of_scholars
29

Lgica - 42

Estos sabios, segn los relatos de la escuela socrtica, se dejaron llevar por el orgullo, y cayeron en el relativismo, el escepticismo o las opiniones interesadas. Protgoras (+411), Gorgias, Hipias, Prdico, Trasmaco, Critias, y Calicles son las figuras ms importantes, quedando para el conjunto de ellos la denominacin despectiva de sofistas (ver Las nubes de Aristfanes). Lo ms importante para nosotros es que sus razonamientos, con los que pretendan convencer al pueblo, eran con frecuencia falaces. Contra ellos reaccionan fuertemente Scrates (+399) y su discpulo Platn (~428 347 aC) . En la obra de ambos se concreta la necesidad de fijar el sentido nico de la verdad, y la rigurosidad del proceso de reflexin, basado en dilogos con los que se razona de manera inductiva-deductiva (el famoso mtodo mayutico).
Aristteles: el Organon

Sin embargo, slo con Aristteles despliega la lgica su verdadero valor.

El Organon Estructura del Organon (en griego instrumento) segn la compilacin de Andrnico de Rodas: Isagoge (o Categoras) un libro. Peri Hermeneias /De Interpretatione (Sobre la interpretacin o en torno a la hermenutica) un libro. Analytica Priora o Primeros Analticos 2 libros dedicados principalmente a los silogismos Analytica Posteriora o Segundos Analticos 2 libros dedicados principalmente a las demostraciones Tpica 8 libros, en gran medida dedicados a la dialctica. Protrptico o Exortacin a la Filosofa Elenco Sofstico o Refutacin a los sofistas un libro.

Tras Aristteles: Crisipo de Soli, Porfirio,

Lgica - 43

La Lgica en la Alta Edad Media

Boecio

Pedro Hispano

La edad media escolstica: aristotlicos, Duns Scoto y Ramon Llull

Lgica - 44

B) El despliegue de las lgicas modernas Leibniz

La lgica sistematizada: Gottlob Frege y Russell, Carnap y Wittgenstein, Hempel y Popper, Hilbert y Gdel

Lgicas no clsicas: intuicionismo,

Lukasiewicz, lgica borrosa y lgica cuntica

Lgica - 45

Panorama actual terico y prctico

AVISO: Este captulo, para mantener la compatibilidad de pginas, debe terminar en la pgina 44 (54 del contador absoluto. Son 6 pginas dedicadas a la historia de la lgica.

Lgica - 46

Ejercicios del captulo 5 Historia de la Lgica


Tras haber recorrido la historia de esta apasionante herramienta del pensamiento, la lgica formal, le invitamos a hacerse algunas preguntas para asimilar los contenidos. Teora 1. Haga un mapa conceptual del captulo, situando los conceptos claves y la relacin entre ellos. 2. Ha evolucionado la lgica? Explique con sus propias palabras cmo ha sido esta evolucin. 3. Pregntese. 3.1.Si ha habido un gran cambio en la lgica, eso significa que una razonamiento que ahora consideramos lgico (correcto) puede no serlo dentro de un tiempo? 3.2.Los descubrimientos en la lgica... significa que se ha encontrado algo que antes no estaba all, una novedad? 4. En qu sentido se puede afirmar que la lgica es propiamente una disciplina histrica? 5. Qu sistema lgico le parece a usted mejor, la lgica clsica o la lgica moderna con todos sus planteamientos modernos? Aplicaciones

Lgica - 47

LGICA - PARTE II: LGICA

CLSICA

5 EL TRMINO I: LA SIMPLE APREHENSIN Y EL TRMINO 6 EL TRMINO II: LA DEFINICIN 7 EL JUICIO I: LA PROPOSICIN 8 EL RACIOCINIO I: LA ARGUMENTACIN 9 EL RACIOCINIO II: EL SILOGISMO

Lgica - 48

Es preciso comenzar por decir lo que es el nombre, lo que es el verbo, y despus lo que son la negacin y la afirmacin, la enunciacin y el juicio. ARISTTELES, Peri Hermeneias, c. 1

5 El trmino (I): la simple aprehensin y el trmino

V. 2.01

En el origen de la informacin: el trmino o concepto Propiedad de los trminos Clasificacin de los trminos predicables y predicamentos Relaciones entre trminos Bibliografa: en este captulo seguimos el texto de VERNEAUX, op. cit., 89-103.

En el origen de la informacin: el trmino o concepto El elemento fundamental del lenguaje es la palabra. Nos comunicamos con palabras, que combinamos en frases de diverso tipo. La informacin se trasmite con palabras, porque cada una lleva un significado, estn ligadas a conceptos o, como deca Aristteles, modificaciones del alma, de las que aquellas (las palabras) son expresiones30. El anlisis de los conceptos lo haremos siguiendo a Aristteles, con las precisiones que aport despus la escolstica. Si seguimos su modo de proceder, veremos su utilidad y cmo aportar una gran claridad al anlisis lgico de los juicios. Tengamos, pues, paciencia e inters para asimilar un tipo de reflexin que nos parecer extrao. La actividad bsica y elemental de la mente humana es la percepcin: darse cuenta de ese color, de que ah hay un conejo, o que por el aroma el almuerzo ya debe estar a punto. Saber cmo sucede esto y cules es la estructura sentiente de la inteligencia son temas de epistemologa (gnoseologa), nosotros simplemente nos quedamos con la capacidad de percibir y la obra que la inteligencia humana hace con la aprehensin de la realidad: su abstraccin en conceptos. Los conceptos son, para nosotros, un significado de algo real o imaginado que existe en la mente humana (casa, agua, alegre, blanco, correr, conejo, apasionante). Se dice tradicionalmente que son representaciones de las cosas o de sus propiedades tal como existen en la realidad Las palabras son el signo fontico de estos significados que el hombre fabrica mediante un proceso de abstraccin (Aristteles) o reconoce (Platn). En lgica empleamos la palabra tcnica trmino (como lo que queda al final de la divisin de los juicios en sus unidades mnimas con significado). Un trmino, como veremos, puede ser simple (casa, seminarista, carro), o su especificacin mediante un trmino complejo: esta casa, algn seminarista, carro americano no muy grande pero muy viejo,

30

ARISTTELES, Peri Hermeneias, c. 1,2-3.

Lgica - 49

Propiedad de los trminos Un trmino o concepto es una esencia (lo que hace que la cosa sea para nosotros esa cosa31). Tiene dos aspectos o propiedades: la comprehensin es el conjunto de notas o caractersticas que lo constituyen. En el anlisis de una clase de realidades (los seres humanos, por ejemplo) descubrimos como inseparables, inherentes, a ese trmino unas carctersticas determinadas (hombre {vivo, sensible, inteligente, libre,}). Las otras pueden variar ampliamente (edad, color de la piel, lengua, habilidades, convicciones y valores). Las caractersticas esenciales son las que abarca o comprende el trmino. la extensin es el conjunto de sujetos, o cosas, a las que es aplicable, sean individuos como especies de individuos (hombre Juan, Pedro, Martn,; latinoamericano, oriental, europeo,) (los sujetos de los que se puede decir tal trmino). La propiedad fundamental es la comprehensin; la extensin es resultado de la primera. La regla fundamental de su relacin es: la comprehensin y extensin de los conceptos estn en razn inversa la una respecto a la otra (ms comprehensin, menos individuos en ella). Hay entre ambos una proporcin inversa. Esto permite relacionar conceptos entre s, restringiendo o generalizando su comprehensin:
Ser vivo > (ms general, e incluye a) Animal > Mamfero > Mamfero terrestre > Primate > Humano

A mayor comprehensin, menor extensin, y viceversa. Clasificacin de los trminos a) Desde el punto de vista de la comprehensin: Desde el punto de vista de la comprehensin, se dan dos relaciones: simples complejos, y concretos abstractos. En cuanto a su contenido, el trmino puede ser: Simple: solo comprende una esencia (hombre, carro) Complejo: comprende varias esencias (hombre venezolano, carro viejo) En cuanto a su relacin con los sujetos a los que se aplica, el trmino es: Concreto: aunque son siempre abstractos (como esencias), se denomina concreto cuando hace referencia a un sujeto (viejo). Abstracto: hace referencia a una forma o cualidad desprendida de todo sujeto (ancianidad). b) Desde la extensin. Puede ser de tres tipos segn lo que abarque su extensin. Se suele marcar con artculos: Singular: su extensin se reduce a un individuo: un perro, ese perro Particular: su extensin es indeterminada: algn perro Universal: su extensin es total: todo perro, ningn perro. Un concepto abstracto es siempre universal. Pero puede particularizarse con palabras cuantificadoras o designadoras: este hombre, algn hombre. A veces se hace sitio para otra divisin: indefinidos y colectivos (no es bueno incluirlos en la divisin).
Cf. Diccionario de Filosofa Herder, ad. loc. cit. Estas esencias tienen una existencia real? Esta es la polmica de los universales que hizo correr ros de tinta en la edad media.
31

Lgica - 50

NOTA 1: Particular puede ser expresado con artculos indefinidos: unos chamos, unas arepas. Al usar el plural indicamos que existen dos o ms sujetos. Pero en lgica, usando la palabra cuantificadora algn (algn chamo es caraqueo, alguna arepa est dura) con el trmino en singular, nos referimos a una cantidad indeterminada que puede ser uno o ms. Con encontrar un nico caso sabemos ya que el juicio en el que est incorporado es verdadero (si fueran todos caraqueos, o todas las arepas duras, el juicio particular es igualmente verdadero). Particular por tanto abarca al menos a un individuo. NOTA 2: El trmino universal se refiere a la totalidad de individuos. Parecera que todos los hombres y todo hombre es lo mismo. Pero no es as, puesto que no nos referimos a la clase colectiva, sino a un trmino aplicado en toda su extensin: TODO hombre. Al usar el trmino universal en un silogismo lo haremos siempre que sea posible en singular, evitando as la paradoja de: Los chinos son numerosos; Confucio es chino; luego, Confucio es numeroso.

Predicables y predicamentos El anlisis de Aristteles dej en evidencia que la complejidad de los juicios se debe a los trminos que aparecen como predicados de la oracin. A ellos dedic sus ms profundos estudios. c) Cuando pasan a integrarse en una proposicin o juicio. Al formar parte de un a proposicin, como sujeto o predicado, quedan en nueva relacin. Ser sujeto es sencillo; pero como predicado surgen tres cuestiones: v. Cmo se relaciona con el sujeto? Son los significados: o Unvoco: Se predica de distintos sujetos siempre del mismo modo, con el mismo significado o Equvoco: en situaciones diversas puede tener significados distintos o Anlogo: se aplica a diversos sujetos en un sentido que no es idntico, pero tampoco totalmente diferente. Hay una relacin entre los significados: la analoga.
Atencin: El concepto de analoga es uno de los ms importantes en filosofa, ya que nos da una herramienta de comprensin de unas propiedades o configuraciones a partir de las otras. Cuando paso de una cualidad (el pastel es muy bueno) a otra (t eres muy buena) estoy trasponiendo por la semejanza de funcionamiento en ambos casos. Dicha relacin puede ser de diverso tipo. Santo Toms, el genio de la analoga, estableca dos tipos, de proporcin o atribucin (la del ejemplo anterior), y otra ms simple de proporcionalidad ( pie de la montaa y pie de un animal)32

Por qu, a ttulo de qu se relaciona con l? Son los predicables (las diversas maneras de atribuir un concepto -unvoco- a un sujeto). Se examina a qu parte de la esencia (comprehensin esencial del trmino) se est refiriendo ese trmino o Gnero --> Esa herramientas de dibujo es un pincel o Diferencia especfica --> Dame slo los que tienen pelitos en la punta, los pinceles o Especie --> los instrumentos del pintor son el pincel y la
Cf. VERNEAUX, p. 93-94; Para un estudio detalladsimo del tema, ASHWORTH, E. Jennifer, "Medieval Theories of Analogy", The Stanford Encyclopedia of Philosophy (Winter 2004 Edition) , Edward N. Zalta (ed.), en Internet [2007]: <http://plato.stanford.edu/archives/win2004/entries/analogy-medieval/>.
32

vi.

Lgica - 51

o Lo propio o El accidente vii.

esptula --> todo pincel, fino o grueso, tiene una mata de pelo en la punta para depositar la pintura sobre la superficie --> este pincel es de pelo de marta cibelina extrafino, y calibre num. 8

Qu dice de ese sujeto? Son los predicamentos, y reciben el nombre de categoras, de las cuales tenemos 1+9: El primer predicamento es la categora substancial, donde reside la esencia: o Substancia (aquello por lo que una cosa es lo que es y le da unidad e identidad a pesar del cambio accidental) Y todos los dems son accidentes (que mudan con el tiempo sin quedar afectada la identidad del objeto): o Cantidad (J. pesa quince kilos; la mapanare tiene mucho veneno) o Cualidad (el rbol es viejo; la casa es blanca; el jardn es pequeo) o Relacin (J. es hijo de A.; Andrs es mayor que P.; lo encontrars bajo el pino) o Lugar (los esquimales viven en el rtico; est en la cocina; la herida es en el pi) o Tiempo (costar diez horas; tard veinte minutos en cocer; vendr maana) o Posicin (fue arriba; est bajo la cama; dispara a la cabeza; ) o Posesin (es mo; el Colibr es de J.; no es mo; ) o Accin (el martillo rompi la mesa; J. reparti cartas; ) o Pasin (la tierra se moj; le dieron cuarenta azotes; sufri un accidente) Relaciones entre trminos a) Segn la extensin Hay familias de conceptos comparables, en relacin de mayor a menor (trminos superiores e inferiores). As se construyen las divisiones, como el rbol de Porfirio, del gnero ms extenso a la especie ms restringida. Al trmino con mayor extensin se le llama superior y forma un todo lgico; al trmino de la misma familia con una extensin menor se le llama inferior, y es una parte lgica del trmino superior. Cuando se clasifica, se buscan aquellas relaciones de extensin que forman el gnero y la especie. Excepto el gnero ms alto y la especie ms baja, todas las dems son al mismo tiempo gnero (de la siguiente) y especie (de la anterior). Ejemplo ya indicado:
Ser vivo > (ms general, e incluye a) Animal > Mamfero > Mamfero terrestre > Primate > Humano

b) Segn la comprehensin La relacin fundamental entre trminos es la oposicin. Dos trminos estn en oposicin cuando se excluyen: no pueden coexistir o aplicarse al mismo tiempo y bajo el mismo aspecto en un mismo sujeto. La razn de la oposicin parece estar en la constitucin metafsica de la realidad, que sigue el principio de no contradiccin, su complementario, el principio de identidad, y la consecuencia en algunos casos, el principio de tertio excluso. Como seal Leibniz, estudiando la negacin de un concepto encontramos relaciones que pueden ayudarnos a clarificar la realidad (y a resolver argumentos por la prueba de reduccin al absurdo). Hay cuatro tipos de oposicin, que debemos distinguir claramente:

Lgica - 52

o Contrarios: dos conceptos positivos que se excluyen por ser del mismo gnero pero diversa especie: blanco/negro, negro/amarillo. Negar uno no lleva directamente a afirmar el otro. o Contradictorios: uno es la negacin pura del otro, y viceversa. No hay punto medio (sentado/ no sentado). Un trmino y su negacin tienen este tipo de oposicin, el ms fuerte y el que se usa en lgicas de verdad/falso. o Relativos: un tipo especial de oposicin en la que dos trminos positivos se excluyen pero se reclaman y dependen el uno del otro: padre/hijo; noche/da. o De Privacin (hbitus): cualidad natural de un sujeto y su negacin (ciego/vidente; sano/enfermo). Difcil de manejar sin recurrir a fsica y metafsica. Grficamente:

La relacin de oposicin es muy til porque: (1) Si asumimos o aceptamos un trmino (como predicado de un sujeto), entonces podemos llegar a la conclusin (inferir) de que no se puede afirmar al mismo tiempo ninguno de sus opuestos (Juan es europeo luego no es americano, no es asitico, no es venzolano, no es ...); (2) Si aseguramos la negacin de un trmino, podemos afirmar que debe ser alguno de los opuestos (Lucas, que es venezolano andino, no es tachirense luego es merideo o trujillano). Si adems, la relacin es de tipo contradictorio, la negacin de uno significa la afirmacin del otro (ya que no hay ninguno ms) En una investigacin de trminos, investigamos siempre aquellos que se presentan en oposicin. Clasificamos qu tipo de relacin de oposicin tienen (contraria / contradictoria; relativos / de privacin). Nos interesa averiguar sobre todo si dos opuestos estn en relacin excluyente (afirmar uno lleva a negar el otro y viceversa). Eso ocurre siempre en los contradictorios (mentiroso no sincero), y a veces en los relativos y los de privacin. En ese caso se dice que los trminos son complementarios.

Lgica - 53

Ejercicios del captulo 5 EL Trmino


El presente captulo es una aproximacin muy terica a la cdula bsica de la lgica, el trmino. Los siguientes ejercicios pretender ayudar a asimilar este instrumental tcnico cuya utilidad se ver en el captulo siguiente y, sobre todo, en la lgica silogstica. Teora 1. Haga un mapa conceptual del captulo, agrupando las diversas clasificaciones y caracterizaciones de los trminos.. 2. Explique qu es la comprehensin de un trmino y su extensin. Aplquelo a los trminos 'zapato deportivo' y 'bota de goma'. 3. Explique porqu el trmino 'bebida alcohlica' tiene menor comprehensin y mayor extensin que el trmino 'cerveza'. Relacione ambos con el trmino 'Big Cola'. 4. Explique por favor porqu el trmino herramienta tiene (1) menor comprehensin y (2) mayor extensin que el trmino martillo. Relacione ambos con el trmino destornillador. Aplicaciones 5. Clasifique los siguientes trminos de acuerdo a su extensin y comprehensin TRMINO casa la casa ancha de la colina ancho anchura (anchor) el CD de Juanes todo CD de Juanes al menos un CD de Juanes ... 6. Oposicin de trminos. Indique qu tipo de relacin de oposicin existe entre las siguientes parejas de trminos: 1. Asesino / vctima 2. Caniche / Pitbull 3. Sano / enfermo 4. Alumno / profesor 5. Verdadero / falso. __________________________________ __________________________________ __________________________________ __________________________________ __________________________________ Simple Complejo Concreto Abstracto Singular Particular Universal ----------

Lgica - 54

7. Oposicin de trminos. Indique qu tipo de relacin de oposicin existe entre las siguientes parejas de trminos: 1. Rey / Sbdito 2. Europeo / Asitico 4. Crudo / Cocinado 5. Terrestre / Extraterrestre __________________________________ __________________________________ __________________________________ __________________________________

3. Entero / Castrado (en caballos)______________________________

8. Indique brevsimamente qu significado tienen los trminos como predicado y cmo funciona el trmino (unvoco, equvoco, anlogo) en las dos oraciones, explicando porqu: 1. Marea: Fue una gran marea del mar atlntico / fue una gran marea humana roja rojita

2. Pureza: Ella tiene una gran pureza del corazn / esta cocana tiene una pureza del 90%.

3. Pie: Vive al pi de la montaa / me duele el pi derecho.

4. Extinguido: El mamut se extingui hace muchos aos / el fuego se extingui rpido.

Lgica - 55

Es preciso comenzar por decir lo que es el nombre, lo que es el verbo, y despus lo que son la negacin y la afirmacin, la enunciacin y el juicio. ARISTTELES, Peri Hermeneias, c. 1

6 El trmino (II): la definicin

V. 2.05

Definicin: arte y ciencia Formas de definir y tipos de definicin El Diccionario como aventura Divisin: arte y ciencia Monumental divisin: el rbol de Porfirio. Bibliografa: en este captulo seguimos el texto de VERNEAUX, Op. Cit., 103-108, y COPI, Op. Cit., Parte I, Cap. IV (La definicin). Cf. Prez Francisco Javier, Pensar y hacer el diccionario. Nociones de lexicografa, = Los Libros de El Nacional, de. El Nacional, Caracas 2005. SIGLAS: RAE = Diccionario de la Real Academia Espaola, 222001

Definicin: arte y ciencia Tiene algn inters prctico definir? Pues s. En primer lugar, nos da la herramienta para pensar y para comunicarnos con precisin y sin ambigedades o equvocos. Pero en segundo lugar, muchas de las discusiones y desacuerdos que se dan en la vida diaria y en la accin pastoral se deben a la diversa interpretacin de los conceptos que se manejan: qu entiendes t por parroquia participativa? una oracin bien hecha? en qu me estoy propasando?... Pero a pesar de su gran importancia, mucha gente define mal, sin usar la definicin para aproximarse a la esencia de las cosas. En este tema estudiaremos la definicin, su mecnica, sus reglas y sus formas. Qu definimos? No definimos objetos singulares, que estn ah en la realidad, sino los conceptos o las palabras que las expresan. Mesa es susceptible de definicin; esta mesa en la que estudio no, su problema no es definirla, sino ver qu concepto le conviene: esto es una mesa, o es un pupitre. Definimos por tanto conceptos o smbolos. Para qu definimos? Para conocer. Nuestra dificultad puede venir de varias causas: 1) Desconocimiento (el castellano tiene 40.000 palabras de uso moderno) 2) Ambigedad del uso. Muchas palabras tienen varias acepciones33. Esta variacin del significado segn el contexto se multiplica adems con el uso, y esto hace que discutamos porque estamos en desacuerdo sobre el significado de trminos (yo opino que esto es una buena comida, t piensas que no es una buena comida). Para superar el desacuerdo debemos convenir en lo que asumimos ambos como buena comida34. 3) Vaguedad del trmino. Los trminos tienen fronteras imprecisas. Para usarlos convenientemente, en sentido unvoco, es preciso acotar su significado. 4) Teorizacin, esto es, nueva comprensin de esa realidad (como cuando se usan determinadas palabras en un ambiente nuevo, como una congregacin, un experiencia espiritual, una investigacin filosfica) 5) Retrica, es decir, necesidad de convencer a los oyentes de un nuevo uso del trmino.
Acepcin. Cada uno de los significados de una palabra segn los contextos en que aparece. RAE. En un debate es importante, cuando se percibe que puede haber ambigedad sobre trminos claves, es muy til preguntar algo as como: Qu entiende usted por ..?, y ofrecer a nuestra vez lo que significa para nosotros. Esta manera de proceder ayuda mucho a encontrar soluciones.
34

33

Lgica - 56

Formas de definir y tipos de definicin Definir es explicitar y acotar el significado de un trmino. Las caractersticas de la definicin son, segn el diccionario de la Real Academia de la Lengua, claridad, exactitud y precisin 35. Para conseguir estas caractersticas, vamos a indagar sobre la estructura de la definicin, sobre el significado, los tipos y finalmente las tcnicas de definicin. 1. Estructura de la definicin y funcionamiento bsico. Una definicin consta de dos partes: un trmino o signo a definir, y una explicacin de su significado: SIMBOLO = SIGNIFICADO Palabra = Explicacin (usando muchas palabras) ( concepto ) = ( comprehensin del concepto ) En latn sus nombres son: Definendum = Definens La relacin entre ambas es obvia: la explicacin sirve para aclarar y precisar (o explicitar y acotar) el significado del trmino que se define. Eso lleva a las tres reglas elementales que debe cumplir una definicin para cumplir su propsito (al final veremos las reglas completas de la definicin): - El definens o explicacin debe ser MS CONOCIDO y MS SENCILLO que el definendum o trmino a definir. El definendum y sus trminos afines NO DEBE ENTRAR como parte de la definicin (conflicto de circularidad) El definens debe situar el trmino con la MXIMA PRECISIN, debe atreverse a decir qu tipo de realidad es (es la acccin de, o es un objeto, animal, instrumento, situacin, sentimiento, relacin, concepto, doctrina, oficio, ); por eso no se deben usar vaguedades muy comunes del tipo: 1. 2. 3. 4. es algo que una cosa que es cuando como si (+ ejemplo)

2. El significado. Si pasamos ahora a estudiar el significado del trmino a definir, que es lo que pretende captar y exponer una definicin, debemos empezar por reconocer que es realmente asombroso cmo los seres humanos hemos concedido a las palabras el poder de tener un significado. El sonido moon hace referencia para los ingleses al satlite que orbita alrededor nuestro, mientras que para nosotros es otro sonido, luna. Ms an, reconocer rpidamente el significado de sonidos como achtung danger! o attenzione!, todas ellas indicadoras de peligro, puede salvarte la vida en un momento dado. Recordemos que como signo, la palabra no tiene ningn significado natural (a excepcin de las onomatopeyas: guau boom!, etc). Es un acuerdo convencional entre los hablantes del
Segn el DRAE: Fijar con claridad, exactitud y precisin la significacin de una palabra o la naturaleza de una persona o cosa. Ad. Loc. Cit. Por tanto no es: dar ejemplos, explicar su utilidad, usar la palabra en una frase bonita,
35

Lgica - 57

idioma. Cmo hace un grupo de personas para aceptar que una palabra tiene cierto significado? No vamos a entrar en el origen de la significacin del lenguaje, tema apasionante pero que requiere un estudio propio. Aceptemos que en general las palabras se han cargado de significado, y nos encontramos con que proviene del siguiente campo de fuerzas:
Trmino = Significado da EXPLICACIN de lo que significa acota la COMPREHENSIN USO depende del MBITO dan la CONNOTACIN

La connotacin es por tanto la explicitacin de la comprehensin del trmino pero situado en el contexto en el que se usa; por eso une la funcin informativa y la expresiva. Y existen tres tipos de connotacin36: 1) Connotacin subjetiva. Es el significado que tiene un trmino para nosotros (para m, pobreza, limpieza suficiente, amistad o lealtad tienen un significado diverso del que tienen esas palabras para mi hermano de sangre o de comunidad). Lo que creemos que significa un trmino o una palabra depende de nuestra experiencia vital, especialmente situaciones fuertes que hemos vivido sobre ese tema, de nuestro inters sobre el mismo, lo que conozcamos ya, el estado anmico que tengamos ese da, Connotacin objetiva. Es el conjunto de caractersticas comunes a todos los objetos que la comunidad de hablantes admite en la extensin del trmino. Pero en la vida real casi nunca podemos acordar al 100% los objetos que pertenecen a esa clase y los que no. Con todo, es el punto de referencia que nos permite superar desacuerdos subjetivos y asumir significados objetivos, propios de un colectivo. Connotacin convencional. Dado que es imposible la mayor parte de las veces conocer totalmente la connotacin objetiva, las comunidades establecen, en su uso comunicativo diario, el significado que tiene para ellos los trminos que usan. Este significado es comn y objetivo para cada grupo de hablantes, como si hubiera un acuerdo o convencin entre ellos (de ah el nombre). Es el significado que recogen los diccionarios de cada lengua.

2)

3)

3. Tipos de definicin. Los tipos de definicin que podemos encontrar se refieren ante todo al uso o al mbito en el que se desea definir una palabra. Son cuatro tipos de definiciones principales: 1) Definicin estipulativa (o nominal). Es la definicin de un trmino nuevo. Se da cuando queremos referirnos a una nueva realidad (por ejemplo, un nueva funcin en la parroquia, que hemos llamado los servidores de la fraternidad). Debe seguir la forma y reglas elementales de la definicin, con el fin de dejar a los destinatarios una idea clara y precisa de lo que se quiere decir con el definendum que hemos inventado. No se puede decir que sea acertada o verdadera, porque antes de ella no haba nada. Definicin lexicogrfica. Es la informacin del significado de un trmino segn el uso que los hablantes de un determinado mbito o comunidad lingstica le dan.
36

2)

El uso preciso del trmino se refiere a la extensin del trmino (los seres a los que se puede aplicar el trmino). Se llama significado extensional o denotativo. Si por el contrario partimos del significado que conlleva el trmino estamos trabajando con su comprehensin, y nos encontramos ante el significado intencional o connotacin, que es el que estudiamos aqu. Cf. COPI, Op. cit., Cap. Definicin, III (p. 107 de nuestra edicin).

Lgica - 58

Normalmente una palabra tiene varias acepciones, pues cada palabra con el tiempo adquiere nuevos significados en nuevos contextos en los que se usa (que pueden ser analgicos o equvocos). Una definicin lexicogrfica puede ser verdadera o falsa, pues puede adecuarse al uso real o errar. 3) Definicin aclaratoria. Es la definicin que explicita la comprehensin de un trmino que suscita polmica o debe ser usado con precisin. No se puede hablan casi nunca de verdad o mentira porque establecen lmites convencionales: un territorio est libre de analfabetismo cuando el 95% (?) de su poblacin es capaz de leer y comprender un texto de tipo TAL (tantas lneas, tal vocabulario). Definicin tcnica. Es la que surge de una nueva teora que hace comprender el trmino de manera nueva. Por ejemplo, los trabajos de Newton y Einstein hicieron redefinir la comprensin tcnica de fuerza y de energa.

4)

4. Tcnicas de la definicin. Una vez situados en un tipo de definicin, deberemos elegir la tcnica que nos proporcione esa definicin clara, exacta y precisa. Hay dos grupos de tcnicas. Es importante conocerlas todas y poder decir cul de ellas se ha usado en una definicin. En primer lugar las tcnicas espontneas o primitivas, luego las elaboradas o cientficas. Las tcnicas espontneas son las que usamos ordinariamente cuando no hay necesidad de profundizar o precisar demasiado. 1) Definicin por sinnimos. Consiste en dar el significado usando trminos sinnimos que cumplan la regla elemental de ser ms conocidos y ms sencillos 37. Es una tcnica eficiente, especialmente entre varios idiomas (Silly? Significa tonto). El problema es que poqusimas palabras son sinnimos puros, la mayora tienen connotaciones convencionales ligeramente distintas, aaden matices o aspectos distintos (por ejemplo, machete: es exactamente lo mismo que charapo, peinilla, rula?). Usar sinnimos puede ser una buena manera de empezar, pero en el uso acadmico debe ser siempre completada por una definicin ms elaborada. Definicin por enumeracin de casos o por ejemplos. Consiste en indicar el significado mencionando un caso que se adapte al trmino (ejemplo), o enumerando los miembros de la clase. Es muy til y puede ayudar en una definicin (muchos diccionarios modernos son ilustrados, y acompaan definiciones elaboradas con dibujos o fotos de algn miembro de la clase). El problema es que un ejemplo puede confundir y deja sin aclarar si otro caso pertenece o no, ya que casi siempre es imposible enumerar todos los casos. Y cuando lo fuera, enumerar su extensin puede dejar oscura la comprehensin del trmino (p. ej.diciendo que cetceo consiste en las ballenas, los cachalotes, los delfines y las orcas, no aclararemos qu es un cetceo). Definicin ostensiva. Se hace mostrando o indicando el objeto al que se refiere el trmino: una mesa es esto (tocando la mesa). Rpida y grfica, pero con los mismos problemas del caso anterior.

2)

3)

Las tres son tiles pero superficiales. Las tcnicas de definicin elaboradas o cientficas pretenden llegar a la mxima precisin posible, y son necesarias cuando hay que tomar decisiones importantes usando esos trminos (por ejemplo, para decidir si se sacrifica un caballo que tiene fiebre equina, o para hacer filosofa sobre el amor, o redactar un reglamento en el que se defina lo que es una falta leve y una falta grave):
Sinnimo: trmino que tiene la misma significacin (para nosotros: igual comprehensin, y por tanto igual extensin).
37

Lgica - 59

4)

Definicin por gnero y diferencia especfica . Es la reina de las definiciones, la ms perfecta filosficamente, y se realiza indicando simplemente el gnero del trmino y la diferencia especfica respecto a los de las otras especies que componen el gnero. La ms famosa es: hombre = animal racional (gnero: animal; diferencia especfica que solamente tienen los humanos: racional). Hay que procurar usarla siempre que sea posible, aunque a veces merece la pena continuar la definicin en modo analtico (ver ms adelante). El problema est en que frecuentemente no se conoce el gnero y la diferencia, por lo que hay que contentarse con empezar con un gnero muy superior (mesa = objeto que sirve para.), y dar luego caracteres de la connotacin. Definicin etimolgica. Es la que se realiza acudiendo a las races de la palabra (autonoma = del griego, auts nomos, ley propia). El RAE suele traer la etimologa cuando es aclaratoria, y el Corominas (ver apndice) la estudia en profundidad. Suele ser muy interesante para saber porqu esa palabra adquiri determinados significados, pero pocas veces aclara suficientemente el significado del trmino. Definicin por causa o por efecto. Es una definicin aproximativa, en la que se indica la causa que lo origina o su efecto caracterstico. Puede ser parte de una definicin por connotacin, pero por si sola, sin combinarse con otras tcnicas, puede dejarnos sin una idea clara de lo que estudiamos (noche = lo que sigue al da). Definicin por connotacin. Se construye a base de enumerar las caractersticas de la comprehensin que se consideran esenciales y comunes a los miembros de la clase, partiendo a veces del gnero o de un sinnimo cercano (hombre = ser biolgico dotado de inteligencia, memoria y voluntad, con capacidad de decisin, que interacta con sus semejantes en un espacio y tiempo determinado (hbitat o contexto)). Es la ms til a los estudiosos, y la que requiere ms trabajo. Cuando est bien hecha, permite al que la recibe comprender el significado y eliminar la vaguedad o ambigedad. Definicin por anlisis. A veces no es posible una descripcin rpida de la comprehensin del trmino, o se desea una mayor aclaracin. Entonces nos vamos al trabajo propio del intelectual: el anlisis de la realidad, la bsqueda de sus caractersticas, la acotacin de su extensin, la investigacin sobre sus causas, consecuencias, esencia Es una tarea muy satisfactoria, pero exige gran extensin, y se realiza slo en los diccionarios temticos, enciclopdicos o en las enciclopedias38.

5)

6)

7)

8)

5. Reglas de la definicin. Todo lo anterior pide una sntesis que gue el trabajo de realizar o de juzgar definiciones. Estas reglas son muy comunes en mbitos neotomistas39 y es importante conocerlas y seguirlas (y exigirlas en los dems), porque resumen lo que hemos explicado con detalle hasta ahora: 1) la definicin no debe contener lo definido (que no sea circular). 2) Que la definicin sea ms clara que lo definido (no puede usarse lenguaje oscuro, emocional o potico, o lenguaje figurado).

38 En ellas, cada trmino desarrollado se llama artculo, y si es de prestigio puede venir firmado, con bibliografa, y se cita como si fuera un artculo de revista. En los ltimos 80 aos se han elaborado estos diccionarios temticos sobre casi todas las reas del saber, y son el punto de arranque de cualquier trabajo especializado. 39 Cf. VERNEAUX, Op. cit., 105; COPI, Op. cit., 120-124.

Lgica - 60

3) Que abarque o convenga a todo lo definido y slo a l (o sea, que sea precisa, esto es, que no sea demasiado ancha caben otras cosas que no eran-, ni demasiado estrecha que deje fuera algunos miembros). 4) Que no sea negativa, pudiendo ser positiva (algunas slo se pueden definir por su contrario, como nada). 5) Que sea breve (porque busca acotar el significado, no dar una conferencia o hacer una poesa). El Diccionario como aventura Un diccionario, segn el RAE, es Libro en el que se recogen y explican de forma ordenada voces de una o ms lenguas, de una ciencia o de una materia determinada (RAE, ad loc. cit.). Como la connotacin objetiva y la definicin por gnero y diferencia son casi inalcanzables, el autor del diccionario o lexicgrafo se embarca en una investigacin sobre su propio lenguaje, armado tan solo de su propia experiencia y de ese mismo lenguaje. Cada diccionario tiene sus inclinaciones, sus preferencias, su manera diversa de ver y comprender el mundo. Una definicin depende de los conocimientos y de las ideas del autor. Por eso componer o consultar un diccionario es sumergirse en una aventura del lenguaje, una aventura viva que ha tardado siglos en forjarse. Para su historia, ver el artculo de la Espasa o la Encarta. Repasemos algunos diccionarios del castellano40: Tras los primeros diccionarios bilinges (Alonso de Palencia, 1490; Nebrija, 1492-95, etc), el primer diccionario espaol es el de Covarrubias, Tesoro de la Lengua castellana o espaola, 1611. El ms importante ha sido el Diccionario de la Real Academia de la Lengua Espaola, editado por primera vez entre 1726 y 1736 (con el ttulo de Diccionario de Autoridades). Estn ya en la edicin 22, 2001, con 87.000 voces nada menos. Se puede consultar en internet (http://www.rae.es), all avisan que preparan la edicin 23. Consultarlo es un paso obligado, aunque hasta hace poco eran muy reacios a incorporar palabras nuevas, y las definiciones eran tambin muy conservadoras. Otros diccionarios actuales de calidad: o Maria Moliner: Diccionario de uso del espaol, Gredos 1966, 21988 muy reorganizado; es una de las mayores aventuras lexicogrficas de la lengua espaola, 14 aos de trabajo con sus fichas en su casa mientras cuidaba de su hogar. Sus definiciones vienen en lenguaje accesible y de gran claridad o Julio Casares: su Diccionario ideolgico de la lengua espaola, Gustavo Gili 1959, 181994, supuso una verdadera revolucin y es una herramienta importante para todo escritor y creador. La parte fundamental es una organizacin de las palabras en familias ideolgicas, por las que podemos pasar de una palabra a otras asociadas a ella como sinnimos, compaeras de categora, antnimos,... Es una gran fuente de inspiracin. o Joan Corominas: Diccionario crtico etimolgico castellano e hispnico, Gredos 1955, revisado, corregido y actualizado en la edicin en seis volmenes de 1980. Es una gran autoridad sobre el origen de las palabras y su uso a travs de los siglos. Corominas tiene otra obra similar en cataln.

Cf. RUIZ CASANOVA Jos Francisco, Topografa esencial del espaol, en Babelia, suplemento cultural de El Pas (Sbado 13 octubre 2001).
40

Lgica - 61

o Manuel Seco, ed.: Diccionario del espaol actual, Aguilar 1999, muy valorado, que se beneficia de 30 aos de investigacin con modernas tcnicas informticas, y aporta ejemplos de uso modero de los trminos. Casas editoras de reconocido prestigio: Larousse, Espasa, Collins, Webster, Bersync (para el italiano: Zingarelli, Mondadori). Uno puede fiarse de las obras de estas editoriales, pero teniendo en cuenta el nivel al que publican, pues la misma casa publica versiones abreviadas, muy simplificadas, generalmente para uso escolar.

El consejo es: mira bien qu diccionario usas; no hay que fiarse de los diccionarios de bolsillo de autor desconocido. LO mejor es revisar algunas definiciones que conocemos bien. Por ltimo, sealar que, segn orden de los trminos, no todos los presentan por orden alfabtico (que es lo ms usual). Existen algunos que tienen un orden temtico: por familias de palabras o de sentidos; Slo existe un gran diccionario castellano temtico: Julio Casares, Diccionario ideolgico de la lengua espaola (1946,1992). Los buenos diccionarios temticos, como los de Paulinas, tiene al final un ndice analtico y uno ideolgico para poder usar los artculos con soltura. En la actualidad, la informtica permite un sistema de consulta y de relacin de trminos (a travs de hipertextos), que difumina el orden secuencial de los diccionarios escritos. Divisin: arte y ciencia La divisin (en lgica) es una operacin que consiste en analizar la extensin de un trmino y distinguir en este todo las partes que lo componen. Es un proceso paralelo a la definicin, que estudia nicamente la comprehensin. En la divisin distribuimos el todo en sus partes lgicas41. En el estudio de un tema la divisin suele seguir a la definicin. Por ejemplo, si nos han pedido un ensayo sobre los problemas morales de la eutanasia, se comienza definiendo por connotacin objetiva lo que se entiende por eutanasia (citando diccionarios o estudios competentes), y a continuacin se enumeran los tipos de eutanasia (eutanasia activa y pasiva). Un trmino puede dividirse de muchas maneras (Continente americano = [conjunto de pases] / [por zonas geogrficas] / [por zonas climticas] / [por desarrollo econmico]). Eso se debe al criterio de divisin, que nos lleva a buscar un tipo u otro de realidad (en el ejemplo se puede estudiar la organizacin poltica, geogrfica, climatolgica, econmica,del mismo todo). En el ejemplo anterior, hay varios tipos de divisiones de la eutanasia: segn la accin, activa y pasiva; segn la condicin de la persona, eutanasia geritrica, en enfermos terminales y en enfermos con enfermedades incurables (llamada por algunos teraputica). Las divisiones muestran el contenido del trmino que se est dividiendo al enumerar los elementos que la componen. Aqu hay un matiz importante: la enumeracin de las partes no es la enumeracin de los individuos, ni de los especimenes, sino de los grupos ms amplios que se pueden hacer bajo el conjunto que se analiza (decir que los cetceos se dividen en odontocetos y misticetos ver apndice-, da una adecuada informacin; dar las setenta y tantas especies de delfines y ballenas es enredar la cuestin).

Decamos en el captulo anterior que: Al trmino con mayor extensin se le llama superior y forma un todo lgico; al trmino de la misma familia con una extensin menor se le llama inferior, y es una parte lgica del trmino superior.Cf. ut Supra.
41

Lgica - 62

Gnero y especie. Cuando dividimos adecuadamente, los trminos que son divisin del todo se llaman gnero, respecto a sus divisiones, que son las especies. En ese caso, la comprehensin de cada especie abarca la del gnero ms otros elementos; estos elementos extras son los predicables que se llaman diferencia especfica (por ejemplo, los primates es uno de los 18 rdenes en los que se dividen, segn la clasificacin actual, los mamferos placentarios; su diferencia especfica es el desarrollo tctil, la organizacin social, esqueleto no especializado,...). Cada trmino-especie puede, a su vez, convertirse en gnero de otras subespecies; los gneros pueden ser considerados tambin partes de un todo ms general (los mamferos placentarios forman, junto con los marsupiales y los monotremas 42 el grupo de los mamferos43. Cuando preparamos un estudio de divisin, o incluso para dividir un trabajo en apartados, debemos seguir ciertas reglas de una buena divisin: Que sea completa, en el sentido de que las especies abarquen toda la extensin de las partes en cada nivel de anlisis (no hace falta que hagamos todos los niveles de anlisis desde el big bang) Que sea exacta, o sea, que divida en partes separadas e independientes, pero complementarias. Que sea compacta: que resulten un nmero adecuado de especies. Si es demasiado alto, se deben introducir divisiones intermedias. Que tenga siempre el mismo criterio de divisin,
es decir, que si divide los deportes por habilidades (correr, batear, rodar; a su vez, sobre un objeto o sin objeto, etc), no puede dividir la segunda rama o una de las ramas segn sean deportes de naturaleza o de cancha.

Cuando sea posible, es til hacer divisiones dicotmicas: en dos partes (que son complementarias). Monumental divisin: el rbol de Porfirio Porfirio, el comentador del Organon de Aristteles, neoplatnico furibundo (era discpulo de Plotino) populariz en el siglo III una divisin dicotmica de la realidad desde su totalidad hasta el hombre singular. Con ella demostraba el orden que reina en el universo y la fuerza del razonamiento analtico sobre la armona del cosmos. Es el antecesor de las moderna clasificaciones taxonmicas. Es un buen ejemplo de este proceso, pero hoy en da no es una divisin adecuada segn los avances cientficos. Se consideran otras formas ms apropiadas que tiene en cuenta las propiedades uniformes de cada grupo.

El orden que abarca slo dos especies vivas, ambas en Australia, de caractersticas sorprendentes: ornitorrinco y equidna. Cf. http://es.wikipedia.org/wiki/Monotremata . 43 Para estos temas, se ha usado la wikipedia, enciclopedia libre y participativa en Internet: http://es.wikipedia.org/ .
42

Lgica - 63

Apndices al tema 6
a) Divisin de ejemplo: los cetceos
Fuente: http://www.sergiohanquet.com/cetaceos/cetca.htm ; rbol completo en: http://en.wikipedia.org/wiki/Cetacea

CETCEOS = [ MISTICETOS (barbas en vez de dientes) | ODONTOCETOS (dientes) ]

* MISTICETOS (seleccin de especies para la zona atlntica tropical)


Familia Balaenidae Familia Balaenopteridae Eubalaena glacialis Balaenoptera physalus Balaenoptera acutorostrata Balaenoptera edeni Balaenoptera borealis Balaenoptera musculus Megaptera novaeangliae Ballena vasca Rorcual comn Rorcual aliblanco Rorcual tropical Rorcual boreal Ballena azul Yubarta o jorobada

* ODONTOCETOS (68 especies; enumeramos algunas para la


zona atlntica tropical)
Familia Delphinidae Globicephala macrorynchus Globicephala melaena Orcinus orca Pseudorca crassidens Grampus griseus Tursiops truncatus Delphinus delphis Stenella frontalis Stenella coeruleoalba Steno bredanensis Lagenodelphis hosei Physeter macrocephalus Kogia breviceps (kogiidae) Kogia simus (kogiidae) Caldern tropical Caldern comn Orca Falsa orca Delfn gris Delfn mular Delfn comn Delfn moteado Delfn listado D de dientes rugosos Delfn de Fraser Cachalote Cachalote pigmeo Cachalote enano

Familia Physeteridae

Familia Ziphiidae

Ziphius cavirostris Zifio comn Hyperoodon ampullatus Zifio boreal Mesoplodon densirostris Zifio de Blainville Mesoplodon europaeus Zifio de Gervais Mesoplodon mirus Zifio de Trae (Nota: existen entre las otras especies cinco de delfines de agua dulce, como el delfn rosado o tonina, Inia Geoffrensis, que habita las cuencas del Orinoco y el Amazonas) (fotos: delfn rosado y delfn comn)

b) rbol de Porfirio

El rbol de Porfirio que presentamos ahora, en un diagrama clsico, nos muestra la fuerza de la divisin que sita al ser humano en relacin con el cosmos. No es la mejor clasificacin del conjunto del cosmos, pero es una buena prueba de la capacidad de comprender y relacionar que tiene el ser humano.

Lgica - 64

Fuente: FERRATER MORA Jos, Diccionario de filosofa, Buenos Aires: Sudamericana, 1969, Vol. 1, S. 125-126. Cf. Diccionario Filosfico Herder (Diccionario de filosofa en CD-ROM: autores, conceptos, textos) en CD [1991], ad. loc. cit.

Lgica - 65

c) Esquematizar un tema

Una de las aplicaciones ms eficaces de la clasificacin es la divisin de un tema para redactar un ensayo, decidir un reparto de tareas, etc. Eso exige poner en prctica todas las habilidades del anlisis lgico de trminos (que sea completa, exacta, compacta, con criterio uniforme), con atencin sobre todo a la fidelidad a la realidad (no sea que estemos aplicando esquemas y criterios que no le son adecuados) y a la utilidad del resultado. Con mucha frecuencia los estudiantes novatos hacen demasiadas divisiones, porque quieren decir demasiadas cosas a la vez. Por eso recordamos la famosa frase de Antoine de Saint Exupry en Tierra de hombres:
La perfeccin Parece que todo el esfuerzo industrial del hombre, todos sus clculos, todas las noches en vela encima de sus planos, slo conduzcan de modo visible a la sencillez. Parece que se necesite toda la experiencia de varias generaciones para perfilar lentamente la curva de una columna, de un casco de barco, de un fuselaje de avin, para lograr la pureza primigenia de un seno o de un hombro. Parece que el trabajo de los ingenieros, de los delineantes, de los analistas del centro de estudios, consiste, aparentemente, en borrar y pulir, en aligerar aquel empalme, equilibrar esta ala hasta que ya no se la note, hasta que ya no sea un ala incrustada en un fuselaje, sino una sola forma que, perfectamente lograda, se ha desprendido de su ganga, una forma que sea como un conjunto misteriosamente ensamblado, espontneo como un poema. Parece que la perfeccin se alcanza no ya cuando no quede nada por aadir, sino cuando no queda nada por suprimir.44

Es una magnfica invitacin a la simplicidad, que indica que se ha llegado a la comprensin adecuada de lo que estamos viendo o creando, y que normalmente requieren trabajo y disciplina. Como informacin til aadimos en el apndice unos apuntes de Richard B. Ramsay de la universidad FLET45.

SAINT EXUPRY Antoine, Tierra de hombres, ltima pgina. Citado por vendell en Internet [08/02/2003; consulta 2007]: http://verbascum.blogalia.com/historias/5323 45 RAMSAY Richard B., Pautas para un buen bosquejo. Recursos generales Universidad FLET , en Internet [2007]: http://www.universidadflet.org/moodle/file.php/95/PautaBosquejo.htm
44

Lgica - 66

Apndice al tema 6
Richard B. Ramsay Pautas para un buen bosquejo

Estudie las siguientes sugerencias para mejorar el formato de un buen bosquejo. Con estas sugerencias, no quisiera imponer ningn tipo de organizacin en particular para un ensayo. Sin embargo, creo que contienen pautas bastante universales que pueden servir para hacer la comunicacin ms clara para la mayora de lectores. 1. En un buen bosquejo, las divisiones en paralelo son distintas, pero iguales en importancia. Por ejemplo, un posible bosquejo de la Carta a los Romanos sera: La Carta a los Romanos I. Doctrina A. El pecado B. La justificacin por fe C. La santificacin por fe D. La seguridad eterna II. Exhortaciones prcticas A. El amor B. Sumisin a las autoridades etc. Note que las divisiones principales son I. Doctrina, y II. Exhortaciones prcticas. Estos dos conceptos son distintos, pero paralelos. Tambin las subdivisiones son distintas, pero semejantes en importancia. Un bosquejo mal hecho de Romanos sera, por ejemplo: Romanos I. Doctrina A. El pecado B. La justificacin por fe II. La santificacin por fe III. Exhortaciones prcticas Por qu est mal hecho? Porque la santificacin por fe es una doctrina, y cabe mejor como subdivisin del punto I, en paralelo con la justificacin por fe. Es decir, Doctrina es una categora grande que incluye la santificacin. Supongamos que usted es un experto en nutricin, y le han invitado a dar un discurso en un restaurante para los cocineros. Usted quiere explicar las distintas clases de alimentos que se usan, destacando la importancia de cada una. Construya un bosquejo para el discurso en los espacios abajo, utilizando las siguientes palabras: manzana zanahorias verduras arvejas pltano Tema general: I. alimentos bistec frutas naranja carnes filete coliflor carne asada

A____________________

V ______________________

Lgica - 67 A. _________________ B. _________________ C. _________________ II. C _____________________ A. _________________ B. _________________ C. _________________ III. F _____________________ A. _________________ B. _________________ C. _________________ (Ver respuesta abajo) 2. En un buen bosquejo, todas las subdivisiones se relacionan de la misma manera con la divisin mayor a la cual pertenecen. Por ejemplo, en el bosquejo de los alimentos, hay tres tipos de verduras: zanahorias, arvejas, y coliflor. Cada una es una subdivisin de verduras. I. Verduras A. Zanahorias B. Arvejas C. Coliflor El siguiente bosquejo sera mal hecho... I. Verduras A. Zanahorias B. Filete ... porque filete no es una verdura. Esto tambin sera mal hecho... I. Verduras II. Zanahorias III. Arvejas ... porque zanahorias y arvejas son subdivisiones de verduras, no categoras de igual importancia. Otro error sera un bosquejo as... I. Verduras A. Zanahorias B. Arvejas C. Sal D. Limn

Lgica - 68 ... porque la sal y el limn son alios para una ensalada, pero no son verduras. Clasifique estas frases para formar un buen bosquejo: el pato la vaca animales con cuatro patas el caballo ___________________ I. _________________ A. ________________ B. ________________ II. ________________ A. ________________ B. ________________ (Ver respuesta al final de esta seccin.) 3. Un buen bosquejo usa un sistema vlido de enumeracin. Hay dos sistemas de enumeracin que son generalmente aceptados: El sistema tradicional es as: I. A. B. la paloma animales con dos patas el perro animales

1. a. b. (1) (2) (a) (b) (3) (4) c. d. 2.

II.

C. D. A. B. C. etc.

III.

Lgica - 69 A veces en un libro o un ensayo, los ttulos principales no tienen nmero ni letra, pero los subttulos siguen con la enumeracin demostrada arriba. (Ver el ensayo modelo de David Suazo J. al final de este cuaderno.) Por ejemplo, podra ser as: _____________ 1. 2. a. b. 3. 4. _____________ 1. 2. etc. El segundo sistema aceptable es as: 1. 2.

2.1 2.2

2.2.1 2.2.2 2.2.3 2.2.4

2.2.2.1 2.2.2.2

3. 4. etc.

2.3 2.4

Lo importante es elegir uno de estos sistemas y usarlo correctamente y consistentemente. 4. En un buen bosquejo, hay por lo menos dos secciones en cada nivel de clasificacin. Por ejemplo, en el bosquejo de los alimentos, hay tres divisiones mayores, I. Verduras II. Carnes III. Frutas y cada divisin mayor tiene varias subdivisiones: I. Verduras A. Zanahorias B. Arvejas C. Coliflor II. Carnes A. Bistec B. Filete C. Carne asada III. Frutas

Lgica - 70 A. Manzana B. Naranja C. Pltano Un bosquejo mal hecho sera as: I. Verduras A. Zanahorias II. Carnes A. Bistec III. Frutas A. Manzana Si hay una sola subdivisin, en realidad el tema de esa seccin es solamente la subdivisin, y habra que analizar de nuevo todo el esquema y replantear lo que quiere comunicar. Respuestas de los ejercicios de Pautas para un buen bosquejo: 1) Tema general: Alimentos I. Verduras A. Zanahorias B. Arvejas C. Coliflor II. Carnes A. Bistec B. Filete C. Carne asada III. Frutas A. Manzana B. Naranja C. Pltano

Nota: El orden de las subdivisiones se puede variar. 2) Animales I. Animales con dos patas A. El pato B. La paloma II. Animales con cuatro patas A. La vaca B. El caballo C. El perro

(Se puede variar el orden)

Lgica - 71

Ejercicios del captulo 6 TRMINOS II (La definicin y clasificacin)


Tras haber recorrido la historia de esta apasionante herramienta del pensamiento, la lgica formal, le invitamos a hacerse algunas preguntas para asimilar los contenidos. Teora 1. Haga un mapa conceptual del captulo, situando los conceptos claves y la relacin entre ellos. 2. Elija los diez conceptos fundamentales del captulo y haga una definicin cientfica de cada uno de ellos. 3. Haga diversos tipos de definicin (cuatro al menos) de un mismo trmino, por ejemplo, ser humano, o gandola, o indisciplina, o .... Indique en cada caso qu tipo de definicin est intentando. 4. Rellene en sus propios apuntes los ejercicios de clasificacin que propone el texto del apndice. 5. Indique varias tcnicas de definicin elaboradas o cientficas, en qu consiste cada una, y porqu son mejores que las tcnicas de definicin espontneas o primitivas. 6. Defina por gnero y especie: hombre (ser humano), diamante (que es la cristalizacin del carbono bajo altsimas presiones en condiciones de gran pureza qumica), y delfn (el simptico cetceo que todos conocemos por las pelculas). Prctica

7. Elaboracin de un Diccionario de un rea vital de su inters, en el que usted est trabajando o que sea su hobby (de la vocacin, del carisma de la congregacin, de la vida de seminarista, de la defensa en el futbol, del entrenamiento con pesas,...) a) Elegir los 10 trminos principales por su implicacin y contenido filosfico para el tema que nos ocupa (elabore primero una lista con quince o veinte para tener dnde elegir) b) Estudiar la definicin de los mismos en buenos diccionarios, y presentar una de cada uno. (indicar en cada caso de qu diccionario se ha extrado). c) Aadir la propia definicin, segn las normas de la definicin de este captulo, pero dentro de las connotaciones del tema especfico que est usted estudiando. d) Aada al inicio o al final la bibliografa empleada.

Lgica - 72

Una frase es un enunciado que tiene un sentido de convencin, y cada una de cuyas partes separadas significa por s algo,... ARISTTELES, Peri Hermeneias, c. 4

7 El juicio

V. 2.03

Qu es el juicio o proposicin Tipos de juicio: caractersticas del juicio categrico Propiedades de los juicios categricos Relaciones entre juicios (el cuadro de oposicin y otras relaciones). Bibliografa: en este captulo seguimos el texto de VERNEAUX, Op. Cit., 109-119, y COPI, Op. Cit., Parte II, Cap. V (Las proposiciones categricas). En Internet existen muchos materiales: Citamos como ejemplo el de : http://orbita.starmedia.com/buhofueguino/logica.htm

Qu es el juicio o proposicin Hemos indicado al inicio de nuestro estudio que el lenguaje tiene mltiples funciones, clasificables en tres grandes familias: funcin informativa funcin expresiva funcin directiva46. Solamente el lenguaje informativo puede ser evaluado lgicamente, verificando si lo que se dice corresponde a la realidad objetiva. Las unidades ms sencillas que contienen una informacin completa son las oraciones, que en lgica se designan como proposiciones o juicios47. Cada oracin est compuesta de varios trminos, relacionados entre s. La lingstica, desde tiempos de Aristteles, distingue entre Sujeto (S) y Predicado (P), que a su vez se divide en Verbo, Objeto Directo, Objeto Indirecto y Complementos Circunstanciales. Tipos de juicio: caractersticas del juicio categrico Veamos en un esquema, partiendo de lo anterior, la divisin de los juicios:

Recordemos que una frase puede tener varias funciones simultneas, y que no hay que confundir la funcin con la forma (declarativa, imperativa, interrogativa, exclamativa). Cf. Supra, cap. 3. 47 El nombre de juicio indica la forma en que se expone la informacin: como un juicio sobre la realidad, de la que se est arriesgando alguna informacin. Por ejemplo, Juan es alto o La malta se extrae de la misma cebada que la cerveza. Lo mismo proposicin, que indica el carcter de propuesta o afirmacin sobre cmo es la realidad (generalmente como parte de un argumento). Otro nombre igualmente vlido es el de enunciacin.
46

Lgica - 73

Las oraciones de lenguaje puramente expresivo o puramente directivo no nos interesan. En el grfico vemos cuatro tipos de oraciones o juicios informativos: Expresiones no evaluables (Algn da este mundo volar en pedazos con las bombas nucleares y todos moriremos), (Lo que estoy diciendo ahora es mentira). Un pequeo grupo de expresiones informativas no son evaluables y constituyen uno de los grandes problemas de los sistemas lgicos. Una oracin puede no ser evaluable porque: o Presenta problemas para que alguien la evale , como la del ejemplo (si es verdadera no habr nadie para comprobarlo ). Tambin puede ser porque en la prctica tomara demasiado tiempo o no existen las herramientas en este momento para decidir si es verdad o es mentira (las molculas de esta taza de caf estn distribuidas uniformemente y geomtricamente, los pelos de tu cabeza son ms de 300.000)48. o Porque son paradjicas. Son expresiones irresolubles, que llevan a contradicciones tanto si se asume que son verdaderas o falsas (pensar en el ejemplo, conocido como la paradoja del mentiroso)49. Juicios hipotticos (Si llueve, se nos estropear el cemento, si repruebo tres materias entonces me quitarn la beca). Son los juicios que expresan lo que pasar (o no), si se da determinada condicin (o no se da). Su forma es: si p entonces q (p y q son dos expresiones de hechos). Juicios categricos (todos los perros son mamferos, la bombilla elctrica fue inventada por Tomas Alva Edison). Son los juicios que expresan una afirmacin o negacin directa de un dato (predicado) atribuido a un sujeto. Son los juicios que usamos en la lgica clsica o aristotlica, y por tanto a los que dedicaremos ahora ms atencin. Otros tipos de juicios (que requerirn otro tipo de lgica). Algunas expresiones que indican: orden entre varios sujetos, probabilidad, relaciones espaciales, mediciones cuantitativas precisas requieren anlisis lgicos distintos a los anteriores. Para ellos existen diversas lgicas modernas muy interesantes, pero complicadas de usar. Algunos lgicos opinan que las matemticas clsicas son una lgica que opera con cantidades (nmeros). Los juicios categricos exponen una informacin en forma sencilla: dicen lo que es, lo que fue o lo que ser. Pueden ser expresiones complejas, pero reducibles a una composicin de oraciones simples. Estas oraciones simples ponen en relacin un trmino, el Sujeto, con otro trmino, el Predicado (el perro es agresivo, El Barcelona ganar la liga 2005/2006, Ningn equipo de ftbol venezolano lleg a la final).

48 Continuamente se encuentran mtodos ingeniosos para poder evaluar algunas afirmaciones, por lo que pasan a ser juicios normales. Por ejemplo, para saber el nmero de pelos en la cabeza se puede hacer: 1) examinar la densidad de pelos por cm2; 2) calcular la superficie cubierta por el cuero cabelludo; 3) multiplicar la superficie por la densidad media de cabellos por cm2; 4) dar un margen de error, que puede ser de un 5%, 1% o incluso menos. Las personas tenemos normalmente unos 300.000 cabellos en la cabeza. 49 Atencin que en el lenguaje corriente, fuera de la lgica, es suficiente con expresar trminos aparentemente contradictorios (pero reconciliables de alguna manera) para ser paradoja. Se consideran como paradoja literaria: vivo sin vivir en m, muero porque no muero , time is the best teacher; unfortunatelly, it kills all its students (El tiempo es el mejor maestro; desgraciadamente, mata a todos sus alumnos).

Lgica - 74

Propiedades de los juicios categricos Desde el punto de vista de la lgica formal, nos interesa la forma en la que est dispuesto el contenido. Examinando los diversos juicios, vemos que contienen los trminos Sujeto y Predicado junto con algunas palabras que modifican el valor lgico (verdad o mentira) del conjunto: no, todo, algn, no existe, al menos un, ningn,. Esto nos da pi para clasificar los juicios segn dos propiedades fundamentales de estos: Por la cantidad: el sujeto esta presente en grado universal (todo X) o en grado particular. Nosotros incluimos en ste ltimo grupo el singular (el perro,este carro) como un caso particular. Por la cualidad; siendo sta es la propiedad lgica de la afirmacin o la negacin. Con ello nos quedan cuatro tipos de oraciones, que reducimos siempre a su forma tpica:
oraciones Afirmativas Universales (tipo A): los S son P; todos los S son/hacen P; Todo S es P

oraciones Negativas Universales (tipo E): los S no son/hacen P; no hay S que sea P Ningn S es P oraciones Afirmativas Particulares (tipo I): El S - P; Al menos un S es P; hay Ss que Algn S es P oraciones Negativas Particulares (tipo O): Este S no es P; Hay Ss que no son P Algn S no es P

En silogstica se deben trasformar las expresiones corrientes en las cuatro formas tpicas indicadas.
Relaciones entre juicios (cuatro relaciones que permiten hacer inferencias directas) 1. El cuadro de oposiciones

Lgica - 75

Para que las proposiciones puedan considerarse subalternas, contrarias, etc, deben tener exactamente el mismo trmino como Sujeto y el mismo trmino como Predicado. La nica diferencia es la variacin en cantidad y calidad. Aristteles ya not que entre las proposiciones del cuadro de oposicin se dan unas reglas precisas que llevan directamente a saber, en algunos casos (no todos) el valor de verdad o falsedad en funcin de la otra. Hay que conocer cada relacin y la ley de verdad y falsedad que las rige.
Subalternas (A I; E O) (difieren por la cantidad): Si la Universal es verdadera, la particular tambin lo es; Si la particular es falsa, tambin lo es la Universal (pero puede ser que A-falsa e I-verdadera). Contrarias (A E) (Universales que difieren por la cualidad): No pueden ser simultneamente verdaderas (Una es verdad ==> la otra es falsa), pero pueden ser simultneamente falsas (si no hay intermediarios, se comportan como contradictorias). Subcontrarias (I O) (Particulares que difieren por la cualidad): No pueden ser simultneamente falsas (Una es falsa ==> la otra es verdadera), pero pueden ser simultneamente verdaderas (si no hay intermediarios, se comportan como contradictorias). Contradictorias (A O; I E) (difieren en cantidad y cualidad): No pueden ser simultneamente verdaderas ni simultneamente falsas. El valor de verdad de una obliga al de la otra (V F; F V).

Veamos un ejemplo de estas relaciones (ejercicio de examen): 1. Nos dicen que Ningn guanaco es portador de la gripe aviar es Verdadero. Partiendo de este juicio, elabore el resto de juicios del cuadro de oposicin y determine cules puede usted asegurar que son verdaderos y/o falsos. Haga lo mismo con Algn sherpa es A-RH+ (Verdadero)
(guanaco) A E I O (sherpa) A E I O Algn sherpa es ARH + V Juicio __ __V / F Juicio Ningn guanaco es portador de la gripe aviar V V/F

Adems de las relaciones del cuadro de oposicin hay muchas otras en las que jugamos con cantidad, calidad y con la introduccin de la negacin en el trmino Sujeto o en el trmino Predicado (mediante la sustitucin por su complementario). Veremos las tres ms importantes.

Lgica - 76

2. Conversin50 Es la operacin que consiste en invertir sujeto (S ) por predicado (P), conservando el mismo sentido. Se da slo en proposiciones de tipo E y en las de tipo I: algunas mujeres son soldados algunos soldados son mujeres (I) ningn venezolano es cobarde ningn cobarde es venezolano (E) (Falacia tpica: dar como vlida la conversin de un juicio tipo A que es cierta en el caso particular de que S y P tengan igual extensin: todo terrestre es terrcola todo terrcola es terrestre ) 3. Obversin Si no-X es la clase complementaria de X, la obversin de una proposicin (A,E,I,O) es mantener el sujeto, y cambiar simultneamente la calidad (Afirmativa a Negativa y viceversa) y el P por su complementario. (A) Todo zamuro es negro Ningn zamuro es de colores (E) (colores=todos menos negro) (O) Algunos loros no hablan Algunos loros son no-parlantes (I) El problema de la obversin no es incurrir en una falacia, sino en encontrar el complementario del trmino predicado. Como se ve en el segundo ejemplo, la manera ms sencilla es explicitar la negacin del trmino indicado, pero eso es difcilmente comprensible en castellano. El complementario es mucho ms fcil en case de que el trmino que estudiamos sea contradictorio con otro. 4. Contraposicin (ATENCIN!) Consiste en sustituir Sujeto y Predicado por sus complementarios e invertirlos, sin variar calidad ni cantidad. Es equivalente a: obversin + conversin + obversin Nota: por la razn anterior la contraposicin slo es vlida para A y O (y las E por limitacin da una O, que no es reversible a su original E ver ms en: Avi SION, Future Logics, y COPI) (A) Todo S es P Todo no-P es no-S (A); (O) Algn S no es P Algn no-P no es no-S (O);

50

Ver la simple y aguda exposicin de Aristteles: Primeros analticos, Libro I, sec. I, cap. 2

Lgica - 77

Ejercicios al Captulo 7 EL JUICIO y sus relaciones de OPOSICIN


Teora Vocabulario del juicio categrico. Defina qu es: i. cantidad de una proposicin ii. cualidad de una proposicin iii. juicios contradictorios (en el cuadro de oposicin) iv. proposiciones subcontrarias (en el cuadro de oposicin) Prctica A) Indique de qu tipo son los juicios categricos siguientes tras reducirlos a su forma tpica: 1) (porque estoy completamente seguro de que) la vaca parir maana o pasado 2) Fernando (me asegur que maana) dar todas las notas de lgica 3) Las vacas suelen parir de noche 4) La mordedura de perro debe dejarse sin cerrar para evitar infecciones 5) No hay pan totalmente duro 6) algunos alumnos repitieron el examen 7) algunos alumnos no repitieron el examen 8) No es posible que un hurn ataque al ser humano 9) Al menos uno de los machetes no se guard limpio y seco 10) La cebra no sufre alitosis 11) Juan es ms alto que Andrs B) Realice la conversin, obversin y contraposicin, cuando sea posible, de las frases anteriores reducidas a su forma tpica. C) Elabore el cuadro de oposicin y diga cul de los cuatro juicios de cada caso es verdadero o falso: 12) A E I Alguna avispa zapatera hace nidos con barro O V Juicio V/F

13) A

Juicio

V/F

E Ningn macedonio excepto Alejandro poda montar a Bucfalo I

Lgica - 78 O

14)

Juicio A Toda jalea real contiene algunos azucares quirales con propiedades astringentes E I O

V/F F

15) A E I

Juicio

V/F

O Algn maute de este rebao no fue vacunado contra la fiebre aftosa

16) A E

Juicio

V/F

I Algn animal debi daar mi cerca ayer O

17) A E I O 18)

Juicio: El zamuro real, cuando ha comido, vuela mucho mejor (F)

V/F

Entre dos juicios contradictorios, si el particular es verdadero, el Universal debe ser: :___________; Y si el Universal es falso, se sabe qu debe ser el otro? (S / No) (Debe ser: ___________).

Realice la conversin, obversin y contraposicin (siempre que sea posible) de los siguientes juicios (aada en parntesis cmo ha elegido el complementario): i. Ninguna madre es malvada ii. Todo malandro es una persona con buen corazn iii. Algn venezolano del s.XXI ser santo

Lgica - 79

Inferencia. Operacin mental por la que de una verdad conocida se pasa a otra no conocida. Esta operacin mental consiste en un proceso psicolgico segn el cual nos creemos justificados a admitir una conclusin por el mero hecho de haber aceptado una o varias premisas. Cuando la inferencia se realiza mediante reglas de lgica, recibe el nombre de razonamiento deductivo (o deduccin), o razonamiento inductivo (o induccin), ... Diccionario de filosofa Herder (CD)

8 El raciocinio I: la Argumentacin
Bibliografa: en este captulo seguimos el texto de VERNEAUX, Op. Cit., 123-130.

V. 2.03

Qu es pensar y qu es razonar? La estructura de la argumentacin Leyes de la correcta argumentacin Razonamiento inductivo y razonamiento deductivo Fundamento del razonamiento.

Qu es pensar y qu es razonar? Nuestro mundo interior es dinmico. Bullimos en percepciones, reflexiones, impulsos, deseos, sentimientos, Nosotros, en lgica, nos centramos en las operaciones del intelecto (de la inteligencia, o tambin de la mente). Es lo que llamamos pensar (que siempre es intencional: pensar en algo), y que es distinto de percibir, sentir, desear, Los elementos que conforman el pensamiento son los trminos (o conceptos o ideas) y los juicios (o proposiciones). Ya los hemos estudiado y sabemos que trminos (conceptos) y juicios dan al ser humano la capacidad de abstraer, de trabajar con universales, de aplicar leyes, previsiones, afirmaciones o negaciones de propiedades, y relaciones de todo tipo. Eso es lo que hizo que la especie humana pudiera sobrevivir en situaciones extremadamente difciles, superando a otros animales mucho ms poderosos: los homnidos no slo asociaban lugares y olores con comida o peligro: pensaban, obteniendo de su experiencia ideas que luego aplicaban a situaciones nuevas (con los juicios), y que le llevaban a hacer planes o estrategias inteligentes. Llamamos pensar a reflexionar, examinar con cuidado algo para formar dictamen 51. Es un acto que ocurre en la mente, pero sin separacin de la percepcin 52, que maneja conceptos abstractos pero desde y para la realidad concreta. Hay muchas maneras de pensar: enjuiciar (emitir un juicio sobre algo), negar, afirmar, opinar, intuir, barruntar, colegir, comparar, verificar, desconfiar, asumir, aceptar, rechazar, algunas son operaciones simples y elementales del pensamiento, otras son complejas y muy elaboradas. De todas ellas nos interesan ciertas operaciones muy importantes, las que muestran mejor el poder de la mente humana: el razonamiento o raciocinio. Cuando razonamos (argumentamos, discutimos, inferimos, deducimos,) pasamos de juicios conocidos a otros nuevos, cuya veracidad no tenemos por la experiencia, pero de los que estamos seguros porque los hemos obtenido por un razonamiento vlido. Este es el gran poder de la mente: llegar a nuevas verdades a partir de las conocidas.
Cf. DRAE, acepcin 2. Como dice el gran filsofo Xabier Zubiri, la mente es una inteligencia sentiente. Estos temas constituyen el objeto de la Filosofa del conocimiento o epistemologa.
51 52

Lgica - 80

Normalmente se llama razonamiento a este proceso mental (se puede usar el trmino especializado inferir, ver la cita al inicio del captulo), y se reserva el nombre de argumentacin a la expresin lingstica de este razonamiento (serie de frases habladas o escritas). Como en lgica trabajamos siempre con ejemplos escritos o hablados, ambas palabras son equivalentes y no haremos distincin. La estructura de la argumentacin La argumentacin tiene ante todo dos partes bien diferenciadas: Antecedente: es lo conocido, aquello que ya sabemos que es verdad. Consecuente: es a lo que llegamos con la fuerza del razonamiento. Una serie de palabra nos sirven para reconocer la estructura lgica que liga antecedente con el consecuente: Dado que porque, puesto que, luego. Cada parte, antecedente y consecuente estn compuestos de los siguientes elementos: Antecedente: est formado de juicios o proposiciones, uno detrs de otro, a veces con elementos de enlace (si Juicio1 + Juicio2 entonces). Estos juicios reciben el nombre de premisas y pueden ser dos o ms (el problema de Einstein tiene 14) Consecuente: muchas veces contiene un nico juicio, que se llama conclusin, introducido con por lo tanto, luego, y por eso. A veces hay varias proposiciones de conclusin en el consecuente (razonamientos mltiples). Leyes de la correcta argumentacin Distingamos otra vez ms: Los juicios informativos, por su adecuacin a la verdad son o bien verdaderos, o bien falsos53 (cierto o acertado equivocado, mentira, falsedad). Los razonamientos son una funcin del pensamiento. Su valor estriba en ser correcto o incorrecto (o vlido invlido). Decir en lgica que un razonamiento es *verdadero* es un error grave porque introduce un mundo de equivocaciones y confusiones. El razonamiento incorrecto muchas veces viene usado por confusin o incapacidad de mantenerse en un camino vlido, o bien por inters, para intentar convencer al otro en una discusin por otras vas que las de la verdad limpia y pura. Cuando se usa as se suele llamar falacia (y en trminos populares, engao). Las falacias obtienen su fuerza de la unin de argumentos que no tienen una verdadera relacin con la conclusin o el juego con equvocos (falacias no formales), o bien de la semejanza con argumentos verdaderos (falacias formales). Razonamiento inductivo y razonamiento deductivo De los muchos caminos que puede seguir el razonamiento, dos son las vas principales: el razonamiento inductivo y razonamiento deductivo. Por razonamiento inductivo se entiende aquel tipo de razonamiento que observa la naturaleza (la realidad), y ante la repeticin de un suficiente nmero de casos afirma la ley general

Esto en lgica clsica, en juicios categricos. En otras lgicas modernas existen otros valores de verdad, pero la cosa no responde a la apreciacin normal de lo que es verdad.
53

Lgica - 81

Por razonamiento deductivo se entiende la inferencia que partiendo de una ley general, determina la validez de un caso particular (o de otra ley de igual o menor rango). El razonamiento silogstico, por ejemplo, es de tipo deductivo. Es un razonamiento claro y elegante. El problema es quin asegura la validez de la ley general: dependen de la experiencia, se nos dice, pero para llegar a ellas se hace un razonamiento inductivo. Y el razonamiento inductivo pocas veces puede asegurar que ha llegado a examinar todos los casos posibles (ejemplo: los cuervos son negros). Fundamento del razonamiento. Porqu el razonamiento segn estas leyes, cuando parte de premisas verdaderas, nos da resultados verdaderos? Esta pregunta nos lleva de la lgica formal a la filosofa de la lgica. Se le han dado diversas explicaciones, dentro de grandes estructuras filosficas: Porque estamos aplicando las mismas categoras mentales (Kant) Por asociacin casual, que algn da fallar (Hume) Porque estamos respondiendo a las relaciones causales o concomitantes del mundo real, que nuestra inteligencia puede aprehender (percibir) (postura realista).

Lgica - 82

Ejercicios del Captulo 8


Teora 1. Haga un mapa conceptual del captulo, destacando las cuestiones prcticas (tienen aplicacin en la determinacin de la verdad o falsedad de la conclusin de un juicio) y aquellas de tipo filosfico. 2. Explique qu es un argumento deductivo. 3. Explique porqu un argumento inductivo tiene problemas para asegurar su validez de modo absoluto. 4. De cuntas premisas tiene que estar formado, como mnimo, el antecedente? 5. Cul es el nmero mximo de premisas que pueden formar un antecedente? 6. Defina e identifique las partes de un razonamiento, empleando uno o varios esquemas: i. Antecedente ii. Consecuente iii. Premisa iv. Conclusin v. Consecuencia (ojo!) Aplicaciones 7. Un argumento incorrecto, puede tener como conclusin un juicio verdadero? (S/No, y en qu situacin). 8. Puede darse que un argumento incorrecto, tenga sus premisas verdaderas y su conclusin sea tambin verdadera? (S/No y porqu).

Lgica - 83

El silogismo es una enunciacin en la que, una vez sentadas ciertas proposiciones, se concluye necesariamente en otra proposicin diferente, slo por el hecho de haber sido aquellas sentadas. ARISTTELES, Primeros Analticos L. I, cap1, 10.

9 El raciocinio II: El Silogismo

V. 2.03

Qu es un silogismo? Anlisis de la composicin de un silogismo categrico tpico Principios y reglas del silogismo Figuras y modos Otros tipos de Silogismos. Bibliografa: en este captulo seguimos el texto de VERNEAUX, Op. Cit., 131 - 145. Ver tambin COPI

Qu es un silogismo? En la cita que abre el tema tenemos la definicin que da Aristteles, que construy por completo el edificio lgico del silogismo, quizs uno de los mayores avances del pensamiento humano. El Silogismo es un tipo de razonamiento deductivo, es decir, que partiendo de conocimientos adquiridos, alguno de ellos de tipo general, puede inferir (llegar a concluir) nuevos conocimientos. Su fuerza est en que es simple y sencillo, por eso se puede saber con total seguridad si estamos ante un silogismo correcto o no54. Ejemplos de silogismos categricos tpicos55: Todos los artistas son eglatras. Algunos artistas son indigentes. Por lo tanto, algunos indigentes son eglatras. Ningn hroe es cobarde. Algunos soldados son cobardes Por lo tanto, algunos soldados no son hroes. Verneaux sintetiza: El Silogismo es una argumentacin en la que, de un antecedente que compara dos trminos con un tercero, se deduce necesariamente un consecuente que une o separa a los dos primeros trminos56 Anlisis de la composicin de un silogismo categrico tpico Existen dos tipos de Silogismos, segn el tipo de juicios de que est formado: silogismo hipottico y silogismo categrico. El Silogismo condicional y sus variantes hipotticas se analizan con ms facilidad con la lgica proposicional, por lo que los dejamos para luego. Adems, Aristteles desarroll el silogismo categrico y ha sido la herramienta fundamental en filosofa y teologa por dos mil aos.
Repasamos el tema anterior: Cuando el razonamiento es correcto, si estamos seguros de la veracidad de las permisas, podemos estar seguros de que la conclusin es verdadera. 55 Cf. COPI, Parte II, cap. VI, pg. 161s 56 VERNEAUX, p. 132.
54

Lgica - 84

Viendo los casos anteriores, podemos enumerar algunas caractersticas del silogismo: Consta de tres juicios categricos: dos premisas y una conclusin. Las premisas tienen un trmino comn, el Trmino Medio ( M), que se repite en las dos, y no aparece en la conclusin. Los dos trminos que se comparan con el comn (Trminos Extremos) son los que aparecen en la conclusin. De ellos: o El que es Predicado de la Conclusin se llama Trmino Mayor ( T), y su premisa se llama Premisa Mayor, y se pone siempre la primera. o El Sujeto de la conclusin se llama Trmino menor (t) Claramente, adems de fijarnos en los trminos que lo componen, cada uno de los tres juicios tiene sus propiedades de cantidad y calidad ( son A,E,I,O). Por tanto, la estructura ms simple de un Silogismo es parecida a esto: (Todo/ningn/algn) M es (no es) T (Todo/ ningn/algn) t es (no es) M Luego, (Todo/ ningn/algn) t es T Se ha de advertir que la disposicin de M,,t,T como sujeto y predicado en las premisas puede variar: M puede ser sujeto en la mayor y predicado en la menor; o predicado en ambas, o predicado en la mayor y sujeto en la menor. Eso da lugar a las figuras, de las que tenemos cuatro distintas, mientras que las propiedades de los juicios Mayor-menor-conclusin dan el modo del silogismo (AAI, AII, etc). Figuras:
Figura 1) MT tM tT 2) T M tM tT 3) M T Mt tT 4) T M Mt tT

Modos: (ejemplo) Mayor E, menor A, conclusin O ==> modo EAO

Principios y reglas del silogismo No todos los modos de todas las figuras son silogismo correctos. Hay unas reglas que dicen cuando el silogismo es correcto. La regla fundamental, el principio de todo razonamiento, es el principio lgico-metafsico de no contradiccin (una cosa no puede ser y no ser algo, al mismo tiempo y bajo el mismo aspecto). De l, en positivo, se deriva el principio de identidad (Si A es A, es A). El silogismo se basa en la identidad entre tres trminos (A es B, B es C, luego A es C), aplicado segn los principios de la extensin de los trminos. Las reglas clsicas (medievales), compiladas en horribles versos latinos, son: 1) Que haya tres trminos: mayor, medio, menor (que conserven el mismo significado a lo largo de todo el razonamiento). Terminus esto triples: mayor, mediusque, minorque. 2) Que los extremos no posean mayor extensin en la conclusin que en las premisas (la conclusin no debe superar las premisas). Latius hos quam praemissae conclusio non vult. 3) Que la conclusin no contenga el trmino medio. Nequaquam medium capiat conclusio fas est.

Lgica - 85

4) Que el trmino medio sea tomado siquiera una vez en toda su extensin. Aut semen aut iterum medius generaliter esto. 5) De dos premisas afirmativas no pueden engendrarse una conclusin negativa. Ambae affirmantes nequeunt generare negantem. 6) De dos premisas negativas no se sigue nada. Utraque si praemissa neget, nil inde sequetur. 7) La conclusin sigue siempre a la premisa ms dbil (ms dbil= particular frente a general y negativa frente a positiva). Peiorem semper sequitur conclusio partem. 8) De dos premisas particulares no se sigue nada. Nihil sequitur geminis ex particularibus unquam.

Figuras y modos Si combinsemos la disposicin de M en las premisas, tenemos cuatro figuras, por ejemplo:
Silogismo 1) MT t M t T (Premisa mayor) (premisa menor) (conclusin)

En este caso, estamos ante un silogismo donde el trmino medio hace esta forma: \ (sujeto en la primera, predicado en la segunda). Es un silogismo de la primera figura. Antes de clasificar un silogismo en una de las cuatro figuras, hay que cerciorarse de queel silogismo se ha reducido a su forma tpica. Para ello se procede siempre de esta forma:

primero, se verifican las reglas de los trminos (1 a 4): que sean tres, que se usen unvocamente, etc... Si cumplen, seguimos adelante. En segundo lugar, se identifica la conclusin del argumento, que en el lenguaje corriente puede ser la primera frase, la segunda o la tercera. Si nos equivocamos aqu, todo lo que sigue estar equivocado. Tercero, se identifica la premisa mayor porque es la que contiene el trmino medio M y el trmino predicado de la conclusin, T (atencin, que a pesar de llamarse mayor, no siempre es una premisa universal, p. ej IAI-3). Cuarto, se identifica la premisa menor, que es la que contiene el trmino medio y el sujeto de la conclusin.

Al poner los tres juicios en orden, Premisa mayor, premisa menor, luego conclusin, ya podemos ver cmo est situado el trmino medio, lo que nos dar una de las cuatro figuras posibles:
Figura 1) MT 2) T M 3) tM tM tT tT (sub-prae) (prae-prae) MT 4) T M Mt Mt tT tT (sub-sub) (prae-sub)

Nombres:

Cada figura representa un modo particular de razonamiento, como veremos enseguida. Una vez identificada la figura, hay que ver cual es el modo (combinacin del tipo de proposicin de cada juicio). Los modos pueden ser en teora: AAA, AAE, AAI, AAO, AEA, AEI, AEO, AIA y as hasta las 64 combinaciones posibles. Pero en la prctica slo algunos modos son vlidos en cada una de las figuras (por las reglas anteriores). Atencin:

Lgica - 86

un modo puede ser vlido en una de las figuras y no serlo en otro (esta es la causa de las falacias formales ms corrientes) Aristteles estudi solo las tres primeras; la figura 4 es la inversin de la 1) y no es una forma tpica de pensamiento humano; fue introducida ms tarde por el mdico Galeno y tiene poco uso. La figura slo se puede determinar una vez hemos reducido el silogismos su forma tpica.

Desde Aristteles, se han estudiado las variaciones posibles, y en particular los modos de cada una de las figuras. Slo esos modos dan un silogismo categrico vlido! Al ser muy pocos, se pueden y deben aprender, as uno tiene la seguridad total sobre la correccin del silogismo. Presentamos a continuacin todas los razonamientos silogsticos vlidos. FIGURA 1: Regla del razonamiento: La menor debe ser afirmativa y la mayor universal. Modos vlidos: AAA-1, EAE-1, AII-1, EIO-1 Regla nemotcnica: Barbara, Celarent, Darii, Ferio Falacia tpica: AEE* (parece una afirmacin vlida, pero es una falacia formal, no es un silogismo admisible). FIGURA 2: Regla del razonamiento: Una de las premisas debe ser negativa y la mayor universal. Modos vlidos: EAE, AEE, EIO, AOO Regla nemotcnica: Cesare, Camestres, Festino, Baroco Falacias tpica: AAA*, AII* (son vlidas por la primera) FIGURA 3: Regla del razonamiento: La menor debe ser afirmativa y la conclusin particular. Modos vlidos: AAI, EAO, IAI, AII, OAO, EIO. Regla nemotcnica: Darapti, Felapton, Disamis, Datisi, Bocardo, Ferison Falacia tpica: AEE* (por la dificultad de la figura, es una falacia potente) FIGURA 4: Regla del razonamiento: No refleja un modo uniforme de pensar, ni tiene una regla fija. Modos vlidos: El nico modo que aporta novedad respecto a las otras es EIO (Fresison), aunque otros son tambin posibles: AEE IAI AII 57. Esta figura se suele desestimar para la argumentacin y se usa muy poco. Ntese que slo en la primera figura se obtiene conclusiones de todo tipo: A, E, I, O. La primera figuras se denomina perfecta y resulta clara y convincente para una persona con capacidad de razonamiento verbal. Las reglas nemotcnicas, adems de indicar con las vocales el modo vlido, indican a cul de los modos de la figura 1 pueden reducirse mediante artificios lgicos58.

Entre la informacin que existe en Internet, lo discute claramente el libro del Dr. Sion: SION Avi, Future Logic;[], cap. 9; en Internet [Ed. Or. 21996]: http://www.thelogician.net/2_future_logic/2_chapter_09.htm 58 Para ms detalle: los modos de la 2, 3 y 4 llevan consonantes s, p, m, c; stas indican la operacin a realizar a la proposicin que les preceden. Cf. VERNEAUX, p. 143.
57

Lgica - 87

Presentamos a continuacin algunos ejemplos de silogismos vlidos (empleamos trminos elementales por claridad)59. Hay que leerlos, visualizar su estructura lgica, asimilar su valor como razonamiento tpicamente vlido (y verificar identificando trminos, figura y modo). Brbara (AAA 1) Toda virtud es buena Toda justicia es virtud Toda justicia es buena Ferison (EIO 3) Ningn sofista es creble Algn sofista es abogado Algn abogado no es creble Cesare (EAE 2) Ninguna utopa es realidad Toda verdad es realidad Ninguna verdad es utopa Fresison (EIO 4) Ningn depresivo es violento Algn violento es paranoico Algn paranoico no es depresivo

Figuras:

SILOGISMOS CATEGRICOS: SNTESIS 1) M T 2) T M 3) M T 4) T M tM tM Mt Mt tT tT tT tT (sub-prae) (prae-prae) (sub-sub) (prae-sub)

FIGURA 1: La menor debe ser afirmativa y la mayor universal. (Falacia tpica: AEE*) Modos: Barbara, Celarent, Darii, Ferio (AAA, EAE, AII, EIO). FIGURA 2: Una de las premisas debe ser negativa y la mayor universal. (Falacias tpica: AAA*, AII*) Modos: Cesare, Camestres, Festino, Baroco (EAE, AEE, EIO, AOO) FIGURA 3: La menor debe ser afirmativa y la conclusin particular. (Falacia tpica: AEE*) Modos: Darapti, Felapton, Disamis, Datisi, Bocardo, Ferison (AAI, EAO, IAI, AII, OAO, EIO) FIGURA 4: No refleja un modo uniforme de pensar, ni tiene una regla fija; Modos: El nico modo que aporta novedad respecto a las otras es EIO (Fresison), aunque otros son tambin posibles: AEE IAI AII EAO. Esta figura se suele desestimar para la argumentacin.

Silogismos en la vida diaria Con mucha frecuencia los silogismos que se utilizan en debates o en investigaciones no se expresan de manera tan sencilla. Por ejemplo, el argumento Ningn submarino es un barco de recreo, pues todos los submarinos son barcos de guerra y ningn barco de recreo es un barco de guerra.. Este razonamiento es un silogismo vlido (No slo se puede tratar de un silogismo desordenado, con frecuencia nos encontramos ante silogismos de los que no se explicita la premisa mayor porque se da por supuesta. Todo ello nos lleva a indicar algunas dificultades y el modo de trabajo para poder juzgar finalmente el argumento con las tcnicas de los silogismos categricos60.

59 60

Tomados los tres primeros de SANGUINETTI Juan Jos, Lgica, Eunsa, Pamplona 1982, p. 120. COPI tiene un largo captulo, el VII, dedicado a clarificar estos casos, con muchos ejemplos. Es nuestra

gua.

Lgica - 88

Reduccin de nmero de trminos. Es el primer caso, muy sencillo, de razonamiento que no entra en la forma tpica. Veamos un ejemplo: (1) Todos los mamferos son de sangre caliente (2) Ningn lagarto es de sangre caliente (3) Luego, todos los lagartos son reptiles-peces-o-aves Si lo examinamos bien, hay cuatro trminos. Pero, con los conocimientos de relaciones de oposicin, podemos concluir que reptiles, peces o aves es una expresin complementaria de mamferos. Eso nos permite realizar la obversin de la conclusin, quedando (3') Luego, Ningn lagarto es mamfero No nos queda sino reescribir las premisas en singular con (3'), y tenemos ya la forma tpica sobre la que ejercitar nuestros instrumentos de anlisis. Muchas veces podremos uniformizar trminos mediante las inferencias inmediatas (conversin, obversin, contraposicin,...) Traduccin de las proposiciones a forma tpica . Es un trabajo delicado a realizar sobre los trminos, cuando son adjetivales, sobre el orden de sujeto y predicado, el tratamiento de verbos distintos del verbo ser, y la traduccin de frases con artculo el/la a clase A o I (preferir la A siempre que se se pueda). Entimemas. Es otro tipo de problema. En un entimema, se da slo una premisa y la conclusin. Ello se debe a que los participantes en el debate asumen una premisa escondida, que se supone obvia y aceptada por todos. Ejemplo: Juan es venezolano, porque naci en Venezuela La premisa que falta es aceptada por todos: todo nacido en Venezuela es (automticamente)venezolano. Cuando el entimema se combina con alguno de los problemas anteriores, es ms difcil cuadrar el silogismo tpico. Sorites (polisilogismo encadenado). El sorites es una forma tpica de razonar. En l, nos encontramos con varios silogismos, en los que la conclusin del primero es premisa del segundo (y sucesivamente). No es un silogismo tpico, sino varios, y ser vlido si y solo si cada uno de los silogismo que lo forman son vlidos. Si uno no lo es, se rompe la cadena de inferencias. El silogismo sorites tpico tiene adems una zona entimemtica (no formulada): se dan lasprimeras premisas y la conclusin final. El trmino sorites viene de la paradoja del montn de granos que propusieron algunos sofistas, intentando probar que un montn es (eliminando cada vez un grano), igual a uno, o incluso a nada. Epiquerema. El epiquerema es un silogismo cuyas premisas van demostradas al tiempo que se enuncian. La confusin es slo de orden, basta examinar la demostracin de cada premisa, y pasar luego a construir el silogismo tpico dejando fuera las pruebas.

Lgica - 89

Otros tipos de Silogismos. Durante muchos siglos, los silogismos fueron el formato de razonamiento mejor estudiado, el que se poda analizar con unas leyes simples y efectivas. El modelo del silogismo categrico sirvi pues para afrontar otros tipos de razonamientos que no son de tipo categrico. Vamos a indicarlos, aunque no vamos a usar estas tcnicas, puesto que hay lgicas modernas que nos sern ms sencillas de usar. Silogismo de relacin. El juicio categrico predica una propiedad de un sujeto (Juan es alto, la mapanare es venenosa). Existe otro tipo de juicios, en los que se establece una relacin entre dos sujetos: Pablo es padre de Jorge, la anaconda es ms grande que la mapanare)61. En estos casos, hay que establecer la naturaleza de las relaciones, si son convertibles (simtricas o conmutativas) o no convertibles, si son opuestas (ser ms grande que, llegar despus de...), si tienen transitividad,... Las relaciones de parentesco siguen estas formas (A padre de B <--> B hijo de A). Los signos matemticos = y <, > pueden ser aplicados a ciertos casos, dejando patente la solucin con unos pocos y elegantes clculos. Silogismos hipotticos. Son aquellos que incorporan un juicio hipottico, del tipo: Si viene tu madre, tendremos problemas. Si es verdad que el toro tiene fiebre aftosa, lo tendremos que sacrificar. El funcionamiento de estos juicios es sencillo: tienen una condicin o prtasis, que es un supuesto no realizado o no conocido todava (la conjuncin gramatical que la identifica es el si..., a la que se le une una consecuencia o apdosis. El juicio afirma que esta consecuencia se dar en el caso de que se de la condicin: si p , entonces q. Un razonamiento hipottico usa uno de estos juicios, junto a otro categrico que afirma un hecho real: se dio p, tu madre no vino, no lo tuvimos que sacrificar. Eso da lugar a la inferencia de uno u otro trmino de la condicin. Los do silogismos ms sencillos de esta categora es el Modus Ponens y el Modus Tollens . Vemoslos en notacin proposicional: Modus ponens: pq p entonces, q Modus tollens.
61

[p(pq)]q (forma de la ley)

(pq)q']p'

Tomamos la informacin de SANGUINETI, Op. Cit., 120-121.

Lgica - 90

pq no-q entonces, no-p Veremos su utilidad en la lgica proposicional. Dilemas. El dilema se construye a partir de varios condicionales, que presentan alternativas a una decisin que se va a tomar. Incluyen una clusula que se expresa en un juicio propio: puedes elegir seguir adelante o volverte atrs; a este juicio se le aaden las condiciones sobre el resultado: si sigues adelante te perders y tardarn en encontrarte; si vuelves atrs, nunca sabrs lo que se esconda en el lago encantado. Nuevamente, las herramientas de la lgica proposicional nos permiten establecer la forma del razonamiento y verificar que sean lgicamente correctos.

Lgica - 91

Ejercicios resueltos
Determinar los componentes, el tipo (figura-modo) de cada Silogismo, e indicar si es vlido. 1.Ningn ave amamanta sus criaturas Algn ave camina por tierra Luego, alguno que camina por tierra no amamanta sus criaturas. Respuesta Argumentacin. Es un razonamiento de tres juicios, la conclusin es claramente el de abajo (luego,), dos premisas. Consecuencia de tipo silogstico. Uniformidad y orden. El razonamiento consta de dos premisas con trmino medio (ave). Las premisas estn en forma tpica, no hace falta uniformizar trminos ni cuantificadores. El orden de Premisa Mayor y Menor se cumple tal como est. Estructura. M= Ave T= amamanta a sus criaturas t= (algo) que camina Premisa Mayor: E; Menor: I; Conclusin: O Figura 3) MT

Mt tT

EIO-3 (Ferison)

Validez. Es un modo vlido de la tercera figura, luego es un razonamiento silogstico vlido (correcto). --------------------------------------------------------------------------------------------------------------------------------------2.No hay persona sin Dios que d la vida por sus amigos Alguna persona actualmente da la vida por sus amigos Luego alguna persona actualmente no est sin Dios. Respuesta a) Argumentacin. Conclusin bien identificada, argumentacin de tipo silogstico, todo aparece claro. b) Uniformidad y orden. Dos premisas con tres trminos usados en sentido nico en el argumento. La conclusin emplea est si Dios, pero es una diferencia aceptable que no cambia la argumentacin. c) Estructura M= da/d la vida por sus amigos T= persona sin Dios t= persona (que) actualmente No hay persona sin Dios que d la vida por sus amigos E Figura: 2) T M Alguna persona actualmente da la vida por sus amigos I t

M
Luego alguna persona actualmente no est sin Dios. O

T
d) Validez Es un modo vlido de la segunda figura (Festino), luego es un razonamiento silogstico vlido. ---------------------------------------------------------------------------------------------------------------------------------------3.Todas las personas exitosas son personas profundamente interesadas en su trabajo Ninguna persona que est profundamente interesada en su trabajo se distrae fcilmente cuando est trabajando Por lo tanto, ninguna persona que se distrae fcilmente cuando est trabajando es una persona exitosa. Respuesta a) Argumentacin. Conclusin bien identificada, premisas en su sitio, todo claro,

Lgica - 92

b) Uniformidad y orden. Las premisas tienen tres trminos con significado uniforme. Estn
ordenadas, pero quedar un silogismo de la 4. c) Estructura M= profundamente interesadas en su trabajo T= persona exitosa que) se distrae fclimente Todas las personas exitosas son personas profun t= A E (persona Figura:

4)

TM
Ninguna persona que est profun se distrae fcilmente

Mt
Ninguna persona que se distrae fcilmente es una persona exitosa E

tT

d) Validez. Es un modo vlido de la cuarta figura, luego es un razonamiento silogstico vlido


(correcto) --------------------------------------------------------------------------------------------------------------------------------------4.Ningn submarino es un barco de recreo, pues todos los submarinos son barcos de guerra y ningn barco de recreo es un barco de guerra . Respuesta a) Argumentacin. La conclusin es Ningn submarino es un barco de recreo , lo que viene tras el pues son premisas. b) Uniformidad y orden. La premisa que contiene submarino (t) debe ser la menor; la que contiene barco de recreo debe ser la mayor. c) Estructura. El silogismo queda: ningn barco de recreo es un barco de guerra.E Figura: 2) T M todos los submarinos son barcos de guerra. A tM Ningn submarino es un barco de recreo E. tT

d) Validez. Es un modo propio de la segunda figura, el argumento es vlido. ---------------------------------------------------------------------------------------------------------------------------------------5.Algunos metodistas no son catlicos, pues ningn protestante es catlico y todos los metodistas son protestantes. Respuesta a) Argumentacin. Todo lo que viene tras el pues son argumentos o premisas, luego Algunos metodistas no son catlicos es la conclusin. b) Uniformidad y orden. A partir de la conclusin, observamos los tres trminos: son uniformes y bien situados. La premisa mator debe ser la que contenga catlicos ( ningn protestante es catlico), la menor la otra. c) Estructura M= protestante T= catlico t= metodista Ningn protestante es catlico. E Figura: 1) M T Todos los metodistas son protestantes. A tM Algunos metodistas no son catlicos. O tT Validez. EAO-1 no es un modo vlido (se salta la regla de que la conclusin sigue a la premisa ms dbil en cantidad y calidad. Sin embargo, el mismo razonamiento con la conclusin en E (luego, ningn metodista es catlico) sera vlido, y por el cuadro de oposicin, si una premisa en E es Verdadera, su subalterna en O es tambin verdad, luego, con un paso intermedio se puede probar que el razonamiento es vlido (aunque como una combinacin de 1 silogismo + 1 inferencia). (Problema del examen diferido del Segundo Parcial, febrero 2006)

d)

e) 6.-

Lgica - 93 Ningn culpable se acerca a charlar con la polica y, se ha comprobado que mi cliente jugaba a las bolas criollas con los guardias nacionales del pueblo todos los das, luego esto demuestra que mi cliente es inocente Respuesta a) Argumentacin. Claramente estamos ante un argumento, pronunciado en tono de alegato del defensor, pero con muchos trminos. La conclusin es: (luego esto demuestra que ) mi cliente es inocente (tipo I singular) b) Uniformidad y orden. Debemos tener apenas tres trminos para poder aplicar el anlisis silogstico. Una posibilidad es descubrir un encadenamiento de silogismos; el otro es la simplificacin de trminos. Intentemos ste ltimo. Siendo la conclusin mi cliente es inocente se observa una relacin con la premisa: Ningn culpable se acerca a charlar con la polica (tipo E). La relacin entre culpableinocente es de complementarios, luego se puede obvertir una de ellas (la conclusin) mi cliente es inocente(I) mi cliente no es culpable (O) (por obversin). En segundo lugar, hay que uniformizar el tema de la charla con la polica. Podemos asumir que el cumplimiento de una condicin ms estricta hace veraz el cumplimiento de una condicin menos exigente. Por ello: mi cliente jugaba a las bolas criollas con los guardias nacionales del pueblo todos los das mi cliente se acerca a charlar con la polica (todos los das) c) Estructura. El silogismo queda: Ningn culpable se acerca a charlar con la polica. E Figura:

2) tM tT

M
Mi cliente se acerca(ba) a charlar con la polica mi cliente no es culpable I O

d) Validez. EIO-2 es un modo vlido y propio de razonamiento elemental. Es un silogismo vlido.

Este tipo de ejercicio se puede realizar en internet, con la ayuda de la pgina web
www.duniho.com/fergus/sillysyllogisms.html Le aconsejo que pruebe, an si no sabe ingls. El esquema de anlisis es siempre el mismo, aunque no se pueda decir el significado de lo que se examina (slo estudiamos la forma del razonamiento).

Lgica - 94

Ejercicios del captulo 9 SILOGISMOS CATEGRICOS


Tras haber estudiado el Silogismo, habiendo comprendido y memorizado la estructura (figura y modos) de los silogismos categricos de forma tpica, identificar Trmino mayor, trmino Medio, trmino menor Tipo de la proposicin mayor-menor y conclusin (si no estn en orden, ordenar as) Figura Concluir si es silogismo vlido o no. Teora 1. Haga un mapa conceptual del captulo, situando los conceptos claves y la relacin entre ellos. 2. Explique con sus propias palabras qu es un silogismo. Aplicaciones SILOGISMOS INVLIDOS 1. Al mirar la conclusin, sospechamos que el Silogismo no es vlido (son una falacia formal, independientemente de que la conclusin sea verdadera a veces). Qu reglas violan los siguientes silogismos? (tomado de COPI y de Internet) Algn perro es lindo. Algn perro es feo. Algn feo es lindo. Ningn venezolano es ingls. Ningn tachirense es ingls. Ningn tachirense es venezolano. Todo pez es acutico. Algn mamfero es acutico Algn mamfero es pez Todo burro es equino. Narciso es un burro. Narciso es un equino. Toda oveja bala. Toda oveja es animal Todo animal bala. Todo perro es can. Algn perro es simptico. Todo can es simptico. Todo indio es asitico. Todo vietnamita es asitico. Todo vietnamita es indio. Algn tachirense no es patriota Algn colombiano no es tachirense Algn colombiano no es patriota. Mi vecino es pobre. Mi vecino es un diablo. Mi vecino es un pobre diablo

Lgica - 95 2. Determinar los componentes, el tipo (figura-modo) de cada Silogismo, e indicar si es vlido. Ningn ave amamanta sus criaturas Algn ave camina por tierra Luego, alguno que camina por tierra no amamanta sus criaturas

No hay persona sin Dios que d la vida por sus amigos Alguna persona actualmente da la vida por sus amigos Luego alguna persona actualmente no est sin Dios

Todas las personas exitosas son personas profundamente interesadas en su trabajo Ninguna persona que est profundamente interesada en su trabajo se distrae fcilmente cuando est trabajando Por lo tanto, ninguna persona que se distrae fcilmente cuando est trabajando es una persona exitosa. -

Lgica - 96

SILOGISMOS CATEGRICOS
Tras haber estudiado el Silogismo, habiendo comprendido y memorizado la estructura (figura y modos) de los silogismos categricos de forma tpica, identificar Trmino mayor, trmino Medio, trmino menor Tipo de la proposicin mayor-menor y conclusin (si no estn en orden, ordenar as) Figura Concluir si es silogismo vlido o no. NOTAS: La primera parte es pura mecnica elemental. La segunda requerir ms trabajo. Piense bien!

1. Identifique trminos figura modo, y diga si el silogismo es correcto. Si no lo es, qu regla infringe? _____(Marcar trminos)_______________________ Juicios Figura Modo: Vlido? Ningn perrito faldero es agresivo; Ningn gato capado es agresivo ; Luego ningn gato capado es perrito faldero ..... - .. ..... - .. ..... - ..

2. _____(Marcar trminos)_______________________ Juicios Figura Modo: Vlido? Todos los tachirenses son considerados gochos. ..... - .. Todos los nacidos en Gusimos son tachirenses. Luego, los nacidos en Gusimos son considerados gochos. 3. _____(Marcar trminos)_______________________ Todo cristiano cuenta con la gracia de Dios.
Todo estudiante catlico del IUESTA es cristiano Todo estudiante catlico del IUESTA cuenta con la gracia de Dios Juicios Figura Modo: ..... - .. ...... - .. ..... - ..

..... - .. ..... - ..
Vlido?

4. _____(Marcar trminos)_______________________ Ningn nazi merece compasin. Algn soldado alemn de la IIWW era nazi. Algn soldado alemn de la IIWW no merece compasin

Juicios Figura Modo:

Vlido? ..... - .. ..... - .. ..... - ..

Identificar antecedente y consecuente, determinar los componentes, el tipo (figura-modo) de cada Silogismo, e indicar si es vlido. 5. Todos los revolucionarios son personas socialmente comprometidas. Por eso creo que todos los comunistas son personas socialmente comprometidas, porque todos los comunistas son revolucionarios. 6. Dado que algunos polticos mienten siempre que les conviene, es seguro que algunos polticos no son cristianos, porque ningn cristiano miente siempre que le conviene. 7. (He llegado a la conviccin de que) algunos indigentes son eglatras, porque algunos artistas son indigentes y todos los artistas son eglatras.

Lgica - 97

EVALUACIN DE SILOGISMOS
Pasamos a silogismos ms complejos, y que no parten de la forma tpica.

1. Convierta el siguiente razonamiento a un silogismo de forma tpica e identifique trminos, figura y modo: Sorprende ver cmo con ningn formador, una vez han pasado los primeros aos, es fantico (ni del ftbol ni de nada); debe ser que ningn formador desprecia los hechos, y todos los fanticos desprecian los hechos. (nota: algunos elementos explicativos pueden ser dejados fuera del silogismo, quedando como contexto). 2. Este razonamiento necesita de algunas adaptaciones para poder mostrar el silogismo que lo sostiene: (1) Ningn culpable colabora como voluntario en tareas sociales y, se ha comprobado que (2) mi cliente iba todos los das a dirigir cursos de danza para nios discapacitados, luego esto demuestra que (3) mi cliente es inocente (los nmeros se han aadido para dar indicaciones sobre cada juicio) Nota: Para tener tres trminos uniformes, es necesario a) realizar la obversin de la conclusin (3) para concordar con (1); b) El trmino medio entre (1) y (2) debe ser uniformizado, lo cual se suele hacer en la premisa menor. 3. Complete el entimema, uniformice el silogismo e identifique trminos, figura y modo: Debe haber llovido ltimamente, porque los peces no pican. (ste es un razonamiento vlido) 4. Es conocido que todas las guerrillas latinoamericanas son de inspiracin izquierdistamarxista (en el siglo XX y XXI, dejando aparte los casos muy particulares de Centroamrica). Construya el silogismo que le permita probar lgicamente que: el ELN (liderado por Camilo Torres y otros ex-sacerdotes izquierdistas) es una guerrilla; otro silogismo para demostrar que las AUC (autodefensas Unidas de Colombia, de inspiracin liberalcapitalista) no es una guerrilla. (nota: seguramente necesitar usar 2 modos distintos) 5. El inspector Rodrguez tranquiliz a la seora: Lo que vio usted no es un fantasma, porque lo que vio usted se estaba comiendo la torta que sobr, y ningn fantasma come ni torta ni nada (me temo que se le col el novio de su hija en casa, seora).
Ayude al inspector a convencer a la seora exponiendo el silogismo empleado, analizndolo y determinando si es correcto o no.

Lgica - 98

El caso del turista incomprendido


Tras haber estudiado el Silogismo, este ejercicio pretende ayudar a aplicar los silogismos a la vida ordinaria. Repasar antes qu son los entimemas y el Sorites. El objetivo es resolver el caso con silogismos categricos, y cada uno de ellos debe estar analizado (identificar trminos, figura y modo, y asegurar su validez).

El inspector Rodrguez sacudi la cabeza. Aquel caso le iba a complicar el viernes, y seguramente todo el fin de semana. Demasiados intereses para otro caso de asesinato en esta ciudad bendita. La vctima, un hombre grande y catire, estaba tendido boca abajo en el centro de la habitacin del hotel, en medio de un charco de sangre, con el revolver al lado. Mr. Smith, se llamaba. Segn el tcnico de la cientfica, aquello estaba recientito recientito, de apenas una o dos horas, hacia la 1pm. Qu haca un turista norteamericano en un hotelucho de baja categora del Oeste de Caracas?. En cualquier caso ya haba visto, adems de los periodistas de sucesos que iban llegando, a dos o tres de la polica secreta adems de un yankee que deba ser un alto funcionario de la embajada norteamericana. Todos con sus propios intereses. Y el suyo era aclarar el caso e irse a casa cuanto antes. Antes de pasar a interrogar a los testigos, se fij en los detalles de la escena del crimen. A primera vista pareca un suicido, el tiro en la sien, el revolver al lado, la habitacin con ropas desordenadas. Sin embargo, haba algunos detalles que no pasaron desapercibidos para una persona metdica y de frrea lgica como el inspector Rodrguez: el revolver, como comprob, haba sido disparado dos veces (busc por la habitacin, hasta encontrar que) el otro tiro se haba alojado en en techo, casi oculto por la mancha de humedad. El turista llevaba ropas casuales pero de marca (casi todo Nike, sobre todo las Air Jordan del 2006) Por la marca en la piel, se notaba que le haban quitado el anillo de casado, probablemente un anillo de oro bien grueso. Haba dos maletas en la habitacin, una era la de l y la otra la de la esposa, que le haban dicho que estaba en estado de shoc en la habitacin de al lado, sin soltar ni prenda (haba contado nicamente que ella estaba en el bao cuando sucedi todo). Entre sus libros haba varias novelas y diversos libros de la Fundacin Libreras del Sur sobre socialismo del siglo XXI y gestin participativa. Todo estaba revuelto, como si hubiera habido una discusin o un registro En el cenicero haba muchas colillas de tabaco. Tras examinarlas, comprob que casi todas eran de Marlboro americano, de clase, excepto dos de tabaco negro local. Segn los testigos, se haban odo gritos de discusin, pero no saban con quin hablaba Mr. Smith. Tambin haban visto entrar a la habitacin de la pareja a una sola persona a las 12, sin decir quien era (nadie quiere ser sapo en Catia, eso ya lo saba) pero no vieron si sali antes de la muerte de Mr. Smith o despus.

Bueno, era tiempo de ir a ver a los sospechosos, adems del casi evidente suicidio (1): el fundamentalista islmico (2), el choro local (3), el activista bolivariano (4). Y adems estaba su mujer, una buena moza de buen ver, aparentemente desecha por el horror del asesinato (5). Pero la cosa no estaba tan fcil. Le esperaban fuera de la escena del crimen varios personajes. Su jefe, de la polica cientfica, le susurr: a) Mira, chamo, esto es claramente un suicidio (no nos interesa que salte a la prensa otro caso de turista extranjero balaceado).

Lgica - 99

El alto funcionario de la embajada norteamericana le dijo: b) Mira, los indicios sealan que el asesino es Mr. Ahmed, un fundamentalista Islmico que hemos agarrado aqu al lado cuando se iba a la mezquita. Debe ser de 'Al Quaedda'. El polica local le dijo satisfecho: c) Fue el Chorvo, un conocido choro local, que encontramos cerca del hotel con 200 dlares en el bolsillo. Y el politiquillo de la oposicin estaba gritando a los periodistas: d) Aqu tenemos otro caso de violencia institucional. El asesino es el Checito, conocido activista bolivariano que vino con la tonta excusa de charlar con l . Qu ladillas!pens el inspector. Entr en la sala donde los tenan a todos detenidos, encendi su cigarrillo, y tras exhalar una bocanada de humo, mir a los que estaban a su alrededor, y dijo tranquilamente: Pueden soltarlos a todos. No fue un suicido, ni le robaron, ni es un atentado islmico ni un crimen poltico; lo mat su mujer. Indique con uno o ms silogismos cada vez cmo dedujo lgicamente el inspector que no era ni 1. ni 2. ni 3. ni 4. , sabiendo como sabe el inspector (Premisas mayores) el comportamiento de cada tipo de personas 1 ayuda: algunas Premisas mayores

todo fundamentalista islmico respeta el Viernes ningn choro de Caracas pela unas Nike originales el suicida dispara una sola vez ningn activista bolivariano mata a un simpatizante de la revolucin

2 ayuda: Algunas Premisas menores que se pueden formalizar de distintas maneras: Mr. Smith simpatizaba con la revolucin popular (porque compraba libros en Libreras del Sur) (tipo I) El revolver no se dispar una sola vez (tipo O)

Lgica - 100

LGICA - PARTE III: LGICA


10 LGICA PROPOSICIONAL 11 LGEBRA DE CONECTIVAS 12 OTRAS LGICAS: LGICA
BORROSAS

MODERNA Y APLICACIONES

DE PREDICADOS, DE PROBABILIDAD Y LGICAS

13 ARGUMENTACIN E INVESTIGACIONES LGICAS

Lgica - 101

La proposicin muestra la forma lgica de la realidad Ludwig WITTGENSTEIN, Tractatus Logico-Philosophicus,4.121

PARTE III: LGICA MODERNA Y APLICACIONES

10 Lgica proposicional - 11 Algebra de conectivas

V. 2.01

Lgica proposicional: Origen histrico de la lgica proposicional Proposiciones simples y complejas Conectivas: estudio de las seis conectivas Tablas de verdad Analogas elctricas y electrnicas - Tautologas y Contradicciones Listado de tautologas como teoremas Algebra de conectivas: Principios del clculo proposicional Verificacin de casos con tablas de verdad Uso simple de las tautologas Uso deductivo del clculo Uso demostrativo del clculo Aplicaciones: buscadores en Internet, sistemas electrnicos, Sudoku. Bibliografa especfica: BURGOS Alfonso, Iniciacin a la lgica matemticaEdiciones Vega, Caracas 101983, pp. 91; CIIDET, Lgica Matemtica, en Internet [2006]: http://www.divulcat.com/monografias/matematicas/logica_matematica.html

Nota: en Internet se encuentran muy buenos textos de resumen de lgica proposicional (ya que es la lgica ms estudiada en las universidades de ciencias). Por ello he desistido de crear mis propios apuntes, y en su lugar inserto completamente un trabajo que cito a continuacin, con algn retoque ortogrfico (los autores vienen al final del mismo).
Fuente: CIIDET, Lgica Matemtica, en Internet [2006]: http://www.divulcat.com/monografias/matematicas/logica_matematica.html

------------------- INICIO DEL TEXTO INSERTADO DE INTERNET -----------------Centro Interdisciplinario de Investigacin y Docencia en Educacin Tcnica (CIIDET)

LGICA MATEMTICA
Introduccin. Aprender matemticas, fsica y qumica es muy difcil; as se expresan la mayora de estudiantes de todos los niveles, sin embargo pocas veces se busca una explicacin del porqu no aprenden las ciencias exactas los alumnos. Nuestra teora es la siguiente: Los alumnos no aprenden ciencias exactas, porque no saben relacionar las conocimientos que se proporcionan en la escuela (leyes, teoremas, formulas) con los problemas que se le presentan en la vida real . Otro problema grave es que el aprendizaje no es significativo. El presente trabajo pretende motivar a los estudiantes para que con ayuda de la lgica matemtica, l sea capaz de encontrar estos relacionamientos entre los diferentes esquemas de aprendizaje, para que de esta manera tenga una buena estructura cognitiva. Consideramos que si el alumno sabe lgica matemtica puede relacionar estos conocimientos, con los de otras reas para de esta manera crear conocimiento. La lgica estudia la forma del razonamiento, es una disciplina que por medio de reglas y tcnicas determina si un argumento es vlido. La lgica es ampliamente aplicada en la filosofa, matemticas, computacin, fsica. En la filosofa para determinar si un razonamiento es vlido o no, ya que una frase puede tener diferentes interpretaciones, sin embargo la lgica permite saber el significado correcto. En las matemticos para demostrar teoremas e inferir resultados matemticas que puedan ser aplicados en investigaciones. En la computacin para revisar programas. En general la lgica se aplica en la tarea diaria, ya que cualquier trabajo que se realiza tiene un procedimiento lgico, por el ejemplo; para ir de compras al supermercado una ama de casa tiene que realizar cierto procedimiento lgico que permita realizar dicha tarea. Si una persona desea pintar una pared, este trabajo tiene un procedimiento lgico, ya que no puede pintar si antes no prepara la pintura, o no debe pintar la parte baja de la pared si antes no pint la parte alta porque se manchara lo que ya tiene pintado, tambin dependiendo si es zurdo o

Lgica - 102 derecho, l puede pintar de izquierda a derecha o de derecha a izquierda segn el caso, todo esto es la aplicacin de la lgica. La lgica es pues muy importante; ya que permite resolver incluso problemas a los que nunca se ha enfrentado el ser humano utilizando solamente su inteligencia y apoyndose de algunos conocimientos acumulados, se pueden obtener nuevos inventos innovaciones a los ya existentes o simplemente utilizacin de los mismos. El orden en que se presenta el documento es el siguiente: Primeramente se establece la importancia de la lgica matemtica, despus definimos el concepto de proposicin. Se establece el significado y utilidad de conectivos lgicos para formar proposiciones compuestas. Ms tarde abordamos las proposiciones condicionales y bicondicionales. Definimos tautologa, contradiccin y contingente, y proporcionamos una lista de las tautologas ms importantes, as mismo explicamos a que se le llama proposiciones lgicamente equivalente apoyndonos de tablas de verdad. Para finalizar; abordamos los mtodos de demostracin: directo y por contradiccin, en donde incluye reglas de inferencia. En este trabajo se trata adems de presentar las explicaciones con ejemplos que le sean familiares. Nuestro objetivo es que el alumno aprenda a realizar demostraciones formales por el mtodo directo y el mtodo por contradiccin. Ya que la mayora de los libros comerciales nicamente se quedan en explicacin y demostracin de reglas de inferencia. Consideramos que s el alumno aprende lgica matemtica no tendr problemas para aprender ciencias exacta y ser capaz de programar computadoras, ya que un programa de computadora no es otra cosa que una secuencia de pasos lgicos, que la persona establece para resolver n problema determinado. Es importante mencionar que en las demostraciones no hay un solo camino para llegar al resultado. El camino puede ser mas largo o ms corto dependiendo de las reglas de inferencia y tautologas que el alumno seleccione, pero definitivamente deber llegar al resultado. Puede haber tantas soluciones como alumnos se tenga en clase y todas estar bien. Esto permite que el estudiante tenga confianza en la aplicacin de reglas y frmulas. De tal manera que cuando llegue a poner en practica esto, el sea capaz de inventar su propia solucin, porque en la vida cada quien resuelve sus problemas aplicando las reglas de inferencia para relacionar los conocimientos y obtener el resultado. Desarrollo. La lgica matemtica es la disciplina que trata de mtodos de razonamiento. En un nivel elemental, la lgica proporciona reglas y tcnicas para determinar si es o no valido un argumento dado. El razonamiento lgico se emplea en matemticas para demostrar teoremas; en ciencias de la computacin para verificar si son o no correctos los programas; en las ciencias fsica y naturales, para sacar conclusiones de experimentos; y en las ciencias sociales y en la vida cotidiana, para resolver una multitud de problemas. Ciertamente se usa en forma constante el razonamiento lgico para realizar cualquier actividad. Proposiciones y operaciones lgicas. Una proposicin o enunciado es una oracin que puede ser falsa o verdadera pero no ambas a la vez. La proposicin es un elemento fundamental de la lgica matemtica. A continuacin se tienen algunos ejemplos de proposiciones vlidas y no vlidas, y se explica el porqu algunos enunciados no son proposiciones. Las proposiciones se indican por medio de una letra minscula, dos puntos y la proposicin propiamente dicha. Ejemplo. p: q: r: s: t: w: La tierra es plana. -17 + 38 = 21 x > y-9 El Morelia ser campen en la presente temporada de Fut-Bol. Hola como estas? Lava el coche por favor.

Lgica - 103 Los incisos p y q sabemos que pueden tomar un valor de falso o verdadero; por lo tanto son proposiciones validas. El inciso r tambin es una proposicin valida, aunque el valor de falso o verdadero depende del valor asignado a las variables x y y en determinado momento. La proposicin del inciso s tambin esta perfectamente expresada aunque para decir si es falsa o verdadera se tendra que esperar a que terminara la temporada de fut-boll. Sin embargo los enunciados t y w no son vlidos, ya que no pueden tomar un valor de falso o verdadero, uno de ellos es un saludo y el otro es una orden. Conectivos lgicos y proposiciones compuestas. Existen conectores u operadores lgicas que permiten formar proposiciones compuestas (formadas por varias proposiciones). Los operadores o conectores bsicos son: 1. Operador and (y) (AND) Se utiliza para conectar dos proposiciones que se deben cumplir para que se pueda obtener un resultado verdadero. Si smbolo es: {, un punto (.), un parntesis}. Se le conoce como la multiplicacin lgica: Ejemplo. Sea el siguiente enunciado El coche enciende cuando tiene gasolina en el tanque y tiene corriente la batera Sean: p: El coche enciende. q: Tiene gasolina el tanque. r: Tiene corriente la batera. De tal manera que la representacin del enunciado anterior usando simbologa lgica es como sigue: p= qr Su tabla de verdad es como sigue: q r 1 1 1 0 0 1 0 0 Donde. 1 = verdadero 0 = falso En la tabla anterior el valor de q=1 significa que el tanque tiene gasolina, r=1 significa que la batera tiene corriente y p = q r=1 significa que el coche puede encender. Se puede notar que si q o r valen cero implica que el auto no tiene gasolina y que por lo tanto no puede encender. p=qr 1 0 0 0 NOTA: La proposicin resultante p = q r tiene un solo caso verdadero: aquel en el que las proposiciones simples son todas verdaderas. Conmutativa: la operacin Y es reversible: p = q r = p = r q. Asociativa y tranzsitiva: la operacin Ygoza de ambas propiedades, de manera que: p [( q r ) (s t)] = p [ q r s t] = p qr st y tendr un nico valor verdadero: cuando todas las variables sean verdadera.

2. Operador Or (o) (OR) Con este operador se obtiene un resultado verdadero cuando alguna de las proposiciones es verdadera. Se eindica por medio de los siguientes smbolos: { ,+, }. Se conoce como las suma lgica. Ejemplo. Sea el siguiente enunciado Una persona puede entrar al cine si compra su boleto u obtiene un pase. Donde.

Lgica - 104 p: Entra al cine. q: Compra su boleto. r: Obtiene un pase. q 1 1 0 0 r 1 0 1 0 p =q r 1 1 1 0

La nica manera en la que no puede ingresar al cine (p=0), es que no compre su boleto (q=0) y que no obtenga un pase (r=0). Atencin, en la lgica informal, a veces se plantean opciones que no tiene esta forma, sino que exigen que se cumpla una sola opcin (ver ms adelante el operador derivado O-exclusiva). 3. Operador Not (no) (NOT)

NOTA: La proposicin resultante p = q r tiene un solo caso falso: aquel en el que las proposiciones simples son todas falsas. Conmutativa: la operacin O es reversible: p = q r = p = r q. Asociativa y tranzsitiva: la operacin Ygoza de ambas propiedades, de manera que: p [( q r ) (s t)] = p [ q r s t] = p q r s t y tendr un nico valor falso: cuando todas las variables sean falsas.

Su funcin es negar la proposicin. Esto significa que s alguna proposicin es verdadera y se le aplica el operador not se obtendr su complemento o negacin (falso). Este operador se indica por medio de los siguientes smbolos: {, ,}. Ejemplo: p 1 0 p 0 1 La negacin de est lloviendo en este momento (p=1), es no est lloviendo en este momento (p=0)

Adems de los operadores bsicos (and, or y not) existe el operador xor, cuyo funcionamiento es semejante al operador or con la diferencia en que su resultado es verdadero solamente si una de las proposiciones es cierta, cuando ambas con verdad el resultado es falso. En este momento ya se pueden representar con notacin lgica enunciados ms complejos. Ejemplo Sean las proposiciones: p: Hoy es domingo. q: Tengo que estudiar teoras del aprendizaje. r: Aprobar el curso. El enunciado: Hoy es domingo y tengo que estudiar teoras de aprendizaje o no aprobar el curso. Se puede representar simblicamente de la siguiente manera: p q r Por otro lado con ayuda de estos operadores bsicos se pueden formar los operadores compuestos Nand (combinacin de los operadores Not y And), Nor (combina operadores Not y Or) y Xnor (resultado de Xor y Not)62. 4. Proposiciones condicionales. Conectiva condicional (si entonces) (IF) Una proposicin condicional, es aquella que est formada por dos proposiciones simples (o compuesta) p y q. La cual se indica de la siguiente manera: pq
62

Se lee Si p entonces q

Los operadores negados, NAND, NOR y XNOR son importantes porque es la forma ms sencilla de traducir a circuitos electrnicos de silicio las operaciones lgicas. Por eso se estudian especialmente en informtica. Sobre el XOR (O-exclusivo) ver ms adelante.

Lgica - 105 Ejemplo. El candidato del PRI dice Si salgo electo presidente de la Repblica recibirn un 50% de aumento en su sueldo el prximo ao. Una declaracin como esta se conoce como condicional. Su tabla de verdad es la siguiente: Sean p: Sali electo Presidente de la Repblica. q: Recibirn un 50% de aumento en su sueldo el prximo ao. De tal manera que el enunciado se puede expresar de las siguiente manera. pq Su tabla de verdad queda de la siguiente manera: p 1 1 0 0 q 1 0 1 0 pq 1 0 1 1

La interpretacin de los resultados de la tabla es la siguiente: Considere que se desea analizar si el candidato presidencial minti con la afirmacin del enunciado anterior. Cuando p=1; significa que sali electo, q=1 y recibieron un aumento de 50% en su sueldo, por lo tanto p q =1; significa que el candidato dijo la verdad en su campaa. Cuando p=1 y q=0 significa que p q =0; el candidato minti, ya que sali electo y no se incrementaron los salarios. Cuando p=0 y q=1 significa que aunque no sali electo hubo un aumento del 50% en su salario, que posiblemente fue ajeno al candidato presidencial y por lo tanto; tampoco minti de tal forma que p q =1. 5. Proposicin bicondicional. Conectiva bicondicional (si y solo si) (IFF) Sean p y q dos proposiciones entonces se puede indicar la proposicin bicondicional de la siguiente manera: pq Se lee p si solo si q Esto significa que p es verdadera si y solo si q es tambin verdadera. O bien p es falsa si y solo si q tambin lo es. Ejemplo; el enunciado siguiente es una proposicin bicondicional Es buen estudiante, si y solo si; tiene promedio de diez Donde: p: Es buen estudiante. q: Tiene promedio de diez. por lo tanto su tabla de verdad es. p 1 1 0 0 q 1 0 1 0 pq 1 0 0 1 La proposicin condicional solamente es verdadera si tanto p como q son falsas o bien ambas verdaderas

A partir de este momento, ya se est en condiciones de representar cualquier enunciado con conectores lgicos. Ejemplo.

Lgica - 106 Sea el siguiente enunciado Si no pago la luz, entonces me cortarn la corriente elctrica. Y Si pago la luz, entonces me quedar sin dinero o pedir prestado. Y Si me quedo sin dinero y pido prestado, entonces no podr pagar la deuda, si solo si soy desorganizado Donde: p: Pago la luz. q: Me cortarn la corriente elctrica. r: Me quedar sin dinero. s: Pedir prestado. t: Pagar la deuda. w: soy desorganizado. (p q) [p (rs) ] [(r s) t ] w 6. Operador O-Exclusiva (XOR) El operador O-exclusiva no es uno de los operadores bsicos, pero como se da con cierta frecuencia en el lenguaje natural, en lo que se conoce como dilema aadimos aqu su definicin y su tabla de verdad. Supongamos que una persona tiene dos opciones: o continuar sus estudios de medicina, o ponerse a trabajar de transportista. Ambas opciones son claramente incompatibles: no se puede aceptar la posibilidad de que se cumplan las dos a la vez. q = sigo estudiando r = me pongo a trabajar la expresin es la siguiente: q 1 1 0 0 r 1 0 1 0 p =q r 0 1 1 0

Su smbolo vara segn las ediciones, al no ser un operador fundamental. Aqu proponemos (o bien , ambos son del juego de caracteres 'Symbol'). Se lee as: o bien q, o bien r, y como se ve, tiene dos valores veritativos: cuando una sola de las proposiciones es verdadera

Tablas de verdad. En estos momentos ya se est en condiciones de elaborar cualquier tabla de verdad. A continuacin se presenta un ejemplo para la proposicin [(pq) (qr) ] (rq). p 0 0 0 0 1 1 1 1 q 0 0 1 1 0 0 1 1 r 0 1 0 1 0 1 0 1 q pq (qr) (pq) (qr) 1 1 0 1 1 1 1 1 0 1 0 1 0 1 0 1 1 0 0 0 1 0 1 1 0 1 0 1 0 1 0 1 rq 1 0 1 1 1 0 1 1 [(pq) (qr) ] (rq) 1 0 1 1 0 0 1 1

El nmero de lneas de la tabla de verdad depende del nmero de variables de la expresin y se puede calcular por medio de la siguiente formula. No de lneas = 2n Donde n = nmero de variables distintas.

Lgica - 107 Es importante destacar a medida que se avanza en el contenido del material el alumno deber participar activamente. Estos significa que cuando se esta definiendo proposiciones y caractersticas propias de ellas, adems de los ejemplos que el maestro explique, el alumno deber citar proposiciones diferentes, deber entender el porque un enunciado no es vlido. Cuando se ven conectores lgicos, los alumnos debern saber emplearlos en la representacin de proposiciones ms complejas. Pero algo muy importante, es que los ejemplo que el maestro y los alumnos encuentren en la clase, deben ser de inters para el estudiante. Cuando se ven tablas de verdad el alumno deber saber perfectamente bien el porque de cada uno de los resultados. En pocas palabras el conocimiento deber ser significativo. Tautologa y contradiccin. Tautologa, es aquella proposicin (compuesta) que es cierta para todos los valores de verdad de sus variables. Un ejemplo tpico es la contrapositiva cuya tabla de verdad se indica a continuacin. p 0 0 1 1 q 0 1 0 1 p 1 1 0 0 q 1 0 1 0 pq 1 1 0 1 qp 1 1 0 1 (pq)(qp) 1 1 1 1

Note que en las tautologas para todos los valores de verdad el resultado de la proposicin es siempre 1. Las tautologas son muy importantes en lgica matemtica ya que se consideran leyes en las cuales nos podemos apoyar para realizar demostraciones. A continuacin me permito citar una lista de las tautologas ms conocidas y reglas de inferencia de mayor uso en las demostraciones formales que obviamente el autor no consider.

1.- Doble negacin. 2.- Leyes conmutativas. a). p''p a). (pq)(qp) b). (pq)(qp) c). (pq)(qp) a). [(pq)r][p(qr)] b. [(pq)r][p(qr)] a). [p(qr)][(pq)(pr)] b. [p(qr)][(pq)(pr)] a). (pp)p b). (pp)p a). (pq)'(p'q') b). (pq)'(p'q') c). (pq)(p'q')' b). (pq)(p'q')' a). (pq)(q'p')

3.- Leyes asociativas.

4.- Leyes distributivas.

5.- Leyes de idempotencia.

6.- Leyes de Morgan

7.- Contrapositiva.

Lgica - 108

8.- Implicacin. a). (pq)(p'q) b). (pq)(pq')' c). (pq)(p'q) d). (pq)(pq')' e). [(pr)(qr)][(pq)r] f). [(pq)(pr)][p(qr)] 9.- Equivalencia 10.- Adicin. 11.- Simplificacin. 12.- Absurdo 13.- Modus ponens. 14.- Modus tollens. 15.- Transitividad del 16.- Transitividad del 17.- Mas implicaciones lgicas. a). [p(pq)]q a). [(pq)q']p' a). [(pq)(qr)](pr) a). [(pq)(qr)](pr) a). (pq)[(pr)(qs)] b). (pq)[(pr)(qs)] c). (pq)[(qr)(pr)] a). [(pq)(rs)][(pr)(qs)] b). [(pq)(rs)][(pr)(qs)] a). (pq)[(pq)(qp)] a). p(pq) a). (pq)p a). (p0)p'

18.- Dilemas constructivos.

Contradiccin es aquella proposicin que siempre es falsa para todos los valores de verdad, una de las mas usadas y mas sencilla es p p . Como lo muestra su correspondiente tabla de verdad: p 0 1 Si en el ejemplo anterior p: La puerta es verde. La proposicin p p equivale a decir que La puerta es verde y la puerta no es verde. Por lo tanto se esta contradiciendo o se dice que es una falacia. p 1 0 p p 0 0

Lgica - 109 Una proposicin compuesta cuyos resultados en sus deferentes lneas de la tabla de verdad, dan como resultado 1s y 0s se le llama contingente. Equivalencia lgica. Se dice que dos proposiciones son lgicamente equivalentes, o simplemente equivalentes. Si coinciden sus resultados para los mismo valores de verdad. Se indican como p q. Considero que un buen ejemplo es el que se estableci para ilustrar la tautologa en donde se puede observar que las columnas de (p q) y (qp) para los mismo valores de verdad, por lo tanto se puede establecer que (pq) (qp).

Lgica - 110

Ejercicios del tema 10


Presentamos algunos ejercicios de representacin formal de proposiciones de la vida corriente. Es muy importante practicarlos y dominar la traduccin al lenguaje formal de la lgica proposicional. Identifique las proposiciones atmicas (usando las variables p,q,r,s,t,...) y los conectores lgicos de las siguientes frases, dando finalmente su formalizacin. 1. Empez el curso y t no has hecho ni un solo trabajo! 2. Si no viene maana al entrenamiento no jugar el Domingo y el club lo sancionar. 3. El petroleo subir de precio si las naciones desarrolladas aumentan su demanda y los pases productores no incrementan su produccin. 4. Aprobars el curso si, y slo s, superas todos los exmenes parciales, superas el examen final, o bien superas el examen de reparacin. 5. No aprob las materias, por lo tanto lo botaron. 6. Este verano, si tengo suficiente dinero me compro la playstation. Si no, me pongo a trabajar y la compro, o bien me contento con la vieja gameboy. 7. El criminal salt por la ventana y se alej corriendo o en un carro que le esperaba, o bien no salt, y est todava aqu en la casa!. 8. Si el jurado lo declara culpable, el juez lo puede sentenciar a dos aos de crcel o a cinco de trabajos comunitarios. 9. Cobrar la nmina por el banco es lo mismo que tener que ir todos los primeros de mes a hacer cola y estar en peligro de que te atraquen a la salida, aunque puedes cobrar por banco y mover tus fondos por internet y por la tarjata de crdito. 10. Las condiciones para tener los dolares por CADIVI son: ser venezolano o extranjero con visado, tener una cuenta de ahorros en un banco venezolano y presentar documento de viaje si es por viaje al extranjero, o solicitud justificativa de compras por internet si es que quiere sus dlares para comprar as, o bien documentos de importacin si es usted un pequeo empresario. 11. Iremos de turismo a Mrida si y solo s me pagan el suledo el Lunes y se confirman la vacaciones en la empresa. 12. Esta alarma antiincendios se activa solamente cuando se dispara el detector de humos de la cocina, de las habitaciones (cinco habitaciones), o cuando se presiona el botn de alarma del pasillo o del saln. 13. Lo mismo que la anterior, con la consecuencia: Esta alarma ... del saln, y entonces dispara los aspersores de agua y activa las sirena y las luces de emergencia. 14. Lo mismo que la anterior, con el aadido: Esta alarma ... de emergencia, y si compra el mdulo de conexin telefnica, har una llamada al centro de bomberos que usted programe. Exprese en lenguaje escrito el modo en el que se deben leer las expresiones siguientes:

REPRESENTACIN DE PROPOSICIONES

15. (pq)[(pr)(qs)] 16. [(pr)(qr)][(pq)r] 17. [(pq) (qr) ] (rq).

Lgica - 111

TABLAS DE VERDAD
1. Estudie el valor lgico de la proposicin [(p q) (qr) ] (rq). p 0 0 0 0 1 1 1 1 q 0 0 1 1 0 0 1 1 r 0 1 0 1 0 1 0 1 q pq (qr) (pq) (qr) rq [(pq) (qr) ] (rq)

2. La Central de alarma de incendios STA-3 tiene tres tipos de detectores o entradas lgicas: p = humo (activada en 1); q = calor (activada en 1); r = botn de alarma (activada en 1). El diseador prepara la funcin de seguridad con la siguiente proposicin compuesta: S: [(p q) r) ] r.

S est conectada a una luz verde, indicando con 1 que Todo OK. Cuando pasa a 0 se activan los programas de sirena, llamada a bomberos, etc. Tras hacer la tabla de verdad, diga, en qu condiciones la Central indica Todo OK? p 0 0 0 0 1 1 1 1 q 0 0 1 1 0 0 1 1 r 0 1 0 1 0 1 0 1

Lgica - 112

11 Algebra de conectivas

V. 0.9

Algebra de conectivas: Reglas de inferencia Mtodos de demostracin: por el mtodo directo o por contradiccin (reduccin al absurdo) Verificacin de casos con tablas de verdad Uso simple de las tautologas Uso deductivo del clculo Uso demostrativo del clculo Aplicaciones: buscadores en Internet, sistemas electrnicos, Sudoku. Bibliografa especfica: BURGOS Alfonso, Iniciacin a la lgica matemticaEdiciones Vega, Caracas 101983, pp. 91; CIIDET, Lgica Matemtica, en Internet [2006]: http://www.divulcat.com/monografias/matematicas/logica_matematica.html

----------------- SEGUNDA PARTE DE LOS APUNTES DEL CIIDET--------------Fuente: CIIDET, Lgica Matemtica, en Internet [2006]: http://www.divulcat.com/monografias/matematicas/logica_matematica.html

------------------- SIGUE EL TEXTO INSERTADO DE INTERNET -----------------Reglas de inferencia Los argumentos basados en tautologas representan mtodos de razonamiento universalmente correctos. Su validez depende solamente de la forma de las proposiciones que intervienen y no de los valores de verdad de las variables que contienen. A esos argumentos se les llama reglas de inferencia. Las reglas de inferencia permiten relacionar dos o ms tautologas o hiptesis en una demostracin. Ejemplo 1 Es valido el siguiente argumento?. Si usted invierte en el mercado de valores, entonces se har rico. Si se hace usted rico, entonces ser feliz. ____________________________________________________ Si usted invierte en el mercado de valores, entonces ser feliz. Sea: p: Usted invierte en el mercado de valores. q: Se har rico. r: Ser feliz De tal manera que el enunciado anterior se puede representar con notacin lgica de la siguiente manera: pq qr ______ pr Ejemplo 2. Es valido el siguiente argumento?. Si bajan los impuestos, entonces se eleva el ingreso El ingreso se eleva.

Lgica - 113 _________________________________________ Los impuestos bajan Solucin: Sea p: Los impuestos bajan. q: El ingreso se eleva. pq q _____ p El aplicar la regla de inferencia es lo que le cuesta ms al alumno y se deber poner mucha atencin para que el alumno aprenda a aplicar dicha regla. En una demostracin no solamente hay tautologas e hiptesis, tambin existen reglas de inferencia que permiten obtener nuevas lneas vlidas, esta es la parte en donde la mayora de alumnos tienen problemas y en donde no sabe que regla aplicar para resolver un determinado problema. A continuacin se cita una lista de las principales reglas de inferencia que se pueden aplicar en una demostracin. 19.- Adicin p _______ pq 20.- Simplificacin pq ____________ p 21.- Silogismo disyuntivo pq p _________ q 22.- Silogismo hipottico pq qr ________ pr Mtodos de demostracin. Demostracin por el mtodo directo. Supngase que pq es una tautologa, en donde p y q pueden ser proposiciones compuestas, en las que intervengan cualquier nmero de variables propositvas, se dice que q se desprende lgicamente de p. Supngase una implicacin de la forma. (p1 p2 ....... pn) q 23.Conjuncin p q _________ pq

24.- Modus pones p pq _________ q 25.- Modus tollens pq q ___________ p

Lgica - 114

Es una tautologa. Entonces est implicacin es verdadera sin importar los valores de verdad de cualquiera de sus componentes. En este caso, se dice que q se desprende lgicamente de p1,p2,......,pn. Se escribe. p1 p2 . . pn ___ q

Realmente el camino que se debe seguir para llevar a cabo una demostracin formal usando el mtodo directo. Significa que s se sabe que p1 es verdadera, p2 es verdadera,...... y pn tambin es verdadera, entonces se sabe que q es verdadera. Prcticamente todos los teoremas matemticos estn compuestos por implicaciones de este tipo. (p1 p2 ....... pn) q Donde la pi son llamadas hiptesis o premisas, y q es llamada conclusin. Demostrar el teorema, es demostrar que la implicacin es una tautologa. Note que no estamos tratando de demostrar que q (la conclusin) es verdadera, sino solamente que q es verdadera si todas las pi son verdaderas. Toda demostracin debe comenzar con las hiptesis, seguidas de las tautologas y reglas de inferencia necesarias, hasta llegar a la conclusin. A continuacin se prueba un enunciado en donde se puede apreciar el uso tanto de las tautologas como de las reglas de inferencia. Sean p: Trabajo. q: Ahorro. r: Comprar una casa. s: Podr guardar el coche en mi casa. Analizar el siguiente argumento: "Si trabajo o ahorro, entonces comprar una casa. Si compro una casa, entonces podr guardar el coche en mi casa. Por consiguiente, si no puedo guardar el coche en mi casa, entonces no ahorro". El enunciado anterior se puede representar como: Equivale tambin a probar el siguiente teorema:

p q r; y r s; entonces s' q' [(p q) r] [r s] [s' q'] .

Como se trata de probar un teorema de la forma general: p1 p2 ...... pn q Se aplica el procedimiento general para demostracin de enunciados vlidos. A continuacin se demuestra el teorema respaldando cada uno de sus pasos en tautologas o reglas de inferencia ya conocidas.

1.- (p q) r

Hiptesis

Lgica - 115

2.- r s 3.- q (q p) 4.- q (p q) 5.- q r 6.- q s 7.- s' q'

Hiptesis Adicin tautologa 10 3; ley conmutativa, regla 2 4,1; silogismo hipottico, regla 22 5,2; regla 22 6; contrapositiva, regla 7.

El enunciado es vlido aunque la conclusin puede ser falsa o verdadera. Es recomendable numerar cada uno de los pasos. Se puede notar que las primeras lneas son hiptesis, la lnea 3 es una tautologa conocida y de la lnea 4 a 7 se obtuvieron aplicando reglas de inferencia. Se indica la regla de inferencia aplicada por medio del nmero de la derecha, y las lneas a las cuales se les aplic dicha regla de inferencia por medio de los nmeros de la izquierda. El ejemplo anterior es una demostracin sencilla, pero puede ser tan complicada como sea necesario y el mtodo debe funcionar. Demostracin por contradiccin. El procedimiento de la demostracin por contradiccin es semejante a la que se realiz por el mtodo directo con la diferencia de que las lneas iniciales de dicha demostracin no son nicamente las hiptesis, sino adems se incluye en la demostracin una lnea con la negacin de la conclusin. Por otro lado el objetivo de la demostracin es llegar a una contradiccin. La demostracin del siguiente teorema por el mtodo de contradiccin es como se indica [ p (p r) ] [ (q s) t ] (p s) t Demostracin

1.- p (p r) 2.- (q s) t 3.- p s 4.- t 5.- (q s) 6.- q s 7.- q 8.- s q 9.- s 10.- s p 11.- p 12.- q r 13.- q 14.- q q 15.- Contradiccin.

Hiptesis Hiptesis Hiptesis Negacin de la conclusin 2,4; Modus tollens, regla 25 5; Ley de Morgan, 6 6; Simplificacin, regla 20 6; Ley conmutativa, 2b 8; Simplificacin, regla 20 3; Ley conmutativa, 2 10,9; Silogismo disyuntivo, regla 21 11,1; Modus ponens, regla 24 12; Simplificacin, regla 29 13,7; Conjuncin, regla 23

Note que juntamente con las premisas se debe incluir la negacin de la conclusin. En este momento el alumno ya tiene los elementos para llevar a cabo demostraciones con el apoyo del maestro. Es conveniente plantear varios enunciados, para que el alumno los represente con simbologa lgica en

Lgica - 116 forma de teorema. Que ese mismo teorema lo represente con su tabla de verdad y haga la correspondiente demostracin por los dos mtodos antes mencionados. La forma en que el aprende a aplicar reglas de inferencia es semejante a la manera en que deber realizar una factorizacin o una aplicacin de una frmula en clculo diferencial o integral o la formula que debe aplicar para resolver un problema en fsica. Lo que debe aprender es a relacionar los distintos conocimientos para poder llegar a la solucin. Es importante mencionar que el camino que debe seguir el alumno no es el mismo que el maestro sigui sino uno distinto pero que ambos llegan al resultado.

Conclusiones. La idea principal de este trabajo es que el alumno aprenda el concepto de proposicin, la forma en que se pueden formar proposiciones compuestas usando los conectores lgicos, representar enunciados por medio de simbologa lgica, conocer los conceptos de tautologa, equivalencia lgica, regla de inferencia. Realizar demostraciones de teoremas por medio del mtodo directo y contradiccin. Pero con problemas que le sean familiares e interesantes. Se trata de que en cada uno de los subtemas participe proponiendo sus propios ejemplo y que sobre todo al final de la unidad l tenga la habilidad, confianza e iniciativa para inferir posibles soluciones. Todo enunciado puede ser planteado en trminos de teoremas. Un teorema por lo general es resultado de un planteamiento de un problema, este planteamiento debe tener el siguiente formato. (p1 p2 ....... pn) q Como se establece p1, p2 ,......,p n son hiptesis (o premisas) derivadas del mismo problema y que se consideran vlidas. Pero adems debern conectarse con el operador And ( ), lo cual implica que p1 es cierta y () p2 es verdad y ()...... y pn tambin es cierta entonces ( ) la conclusin (q) es cierta. Para realizar la demostracin formal del teorema se deber partir de las hiptesis, y despus obtener una serie de pasos que tambin deben ser vlidos, ya que son producto de reglas de inferencia. Sin embargo no solamente las hiptesis y reglas de inferencia pueden aparecer en una demostracin formal, sino tambin tautologas conocidas. En el teorema anterior cada uno de los pasos p1, p2,...pn son escalones que debern alcanzarse hasta llegar a la solucin. Lo mismo ocurre con todo tipo de problemas que se nos presentan en la vida, antes de llegar a la solucin debemos alcanzar ciertas metas (p1,p2,....p n) hasta llegar al objetivo o conclusin (q). Pero una vez que logramos el objetivo debemos plantearnos nuevos objetivos que nos permitirn superarnos. Dependiendo del rea de inters al estudiante puede transportad dichos conocimientos, de tal manera que le auxilien para entender y resolver otro tipo de problemas. En el caso de computacin cada lnea de un programa se obtiene inconcientemente aplicando una regla de inferencia y por lo tanto cada instruccin tiene su orden en que debe de ir colocada, si se cambia esa lnea seguramente el resultado ya no ser igual. Pero hay tantas formas de resolver un problema por medio de un programa como alumnos distintos tenga un maestro. Una demostracin formal equivale a relacionar esquemas para formar estructuras cognitivas. S el alumno sabe inferir soluciones lgicas, estar en condiciones de resolver todo tipo de problemas. Uno de los objetivos principales del constructivismo, es la construccin del conocimiento. El tema de lgica matemtica, se presta para que el alumno pueda realizar las relaciones entre las distintas proposiciones, esto permite crear nuevas formas de resolver problemas en distintas ramas: matemticas, fsica, qumica pero tambin en las ciencias sociales y por su puesto cualquier problema de la vida real. Porque cada vez que nos enfrentamos a un problema, manipulamos la informacin por medio de reglas de inferencia que aunque no estn escritas debemos respetar. Cada vez que

Lgica - 117 realizamos una actividad empleamos la lgica para realizarla, quiz algunos realicen dicha actividad por caminos ms corto, otros realizan recorridos ms largos, pero al fin de cuentas lo que importa es llegar al resultado. Si se le da la confianza al alumno para que cree e innove, su estructura cognitiva seguramente va a crecer.

Bibliografa. Libro Autor Bernard Kolman, Robert C. Bisby, Estructuras de Matemticas Discretas Sharon Ross C.L.Liu Elements of Discrete Mathematics Ralph P. Grimaldi Matemticas Discreta y Combinatoria Matemticas Discretas con aplicacin a Jean Paul Tremblay, Ram Manohar las ciencias de la computacin Kenneth A. Ross, Charles R.B. Wright Matemticas Discretas Winfried Karl, Jean Paul Tremblay Matemtica Discreta y Lgica Richard Johnsonbaugh Matemticas Discretas Editorial Prentice Hall Mc graw Hill Addiso Wesley CECSA Prentice Hall Prentice Hall Gpo. Editorial Iberoamerica

Trabajo enviado por: Jos Alfredo Jimnez Murillo. e-mail: ppalf@yahoo.com Ma. Aleida Hernndez Ynez. e-mail: aleidahy@yahoo.com Alumnos del Centro Interdisciplinario de Investigacin y Docencia en Educacin Tcnica (CIIDET) Quertaro Qro. Mxico.

Lgica - 118

Ejercicios del tema 11

Lgica - 119

Despus de lo dicho, es preciso tratar del orden que debe guardarse en los argumentos, y decir cmo se debe interrogar. La primera cosa que importa, cuando se debe hablar de una pregunta, es encontrar el lugar que deba emplearse; luego, interrogarse a s mismo, prepararlo todo convenientemente; y en tercer lugar, exponer todo esto a la persona a quien nos dirigimos ARISTTELES, Tpicos, Libro VIII, c. 1

13 Investigaciones lgicas

V. 2.12

Dialctica clsica Retrica clsica El dilogo como bsqueda de verdad y de acuerdo Aplicacin de la lgica a las decisiones de la vida diaria La opcin: racionalidad o irracionalidad Investigacin de la lgica-logos subyacente a un problema Palabras de despedida. Bibliografa: ROMERA ngel, Retrica. Manual de retrica y recursos estilsticos. Disponible on-line en Internet[2007]: Libro de notas, http://retorica.librodenotas.com/ ; GARCA DAMBORENEA Ricardo, Uso de razn. El arte de razonar, persuadir, refutar , en Internet [2007]: http://www.usoderazon.com/ ; ESPASA-CALPE, art. Retrica, tomo 50, 1414-1418; Id., art. Dialctica, .Dilogo democrtico, sitio web: http://www.democraticdialoguenetwork.org/index.pl? lang=es

Dialctica clsica Por dialectica entendemos arte de dialogar, argumentar y discutir , pero significa tambin Mtodo de razonamiento desarrollado a partir de principios,Serie ordenada de verdades o teoremas que se desarrolla en la ciencia o en la sucesin y encadenamiento de los hechos63 Como llamaremos al que sabe preguntar y responder sino dialctico? (Platn, Cratilo, 390, c). Numerosos fueron los que estudiaron y codificaron sus leyes, cada uno desde su particular experiencia y formacin filosica. A partir de Hegel, el trmino se usa tambin para referirse a la particular dinmica del Espritu absoluto y, ms tarde, de toda la realidad. Eso convierte la dialctica, con sus tres fases clsicas de tesis, anttesis, sntesis (y paso a una nueva etapa en un proceso casi indefinido). Sin embargo, nosotros no usaremos esta acepcin filosfica y asumiremos su uso ms sencillo: es una aplicacin de la Lgica a problemas reales de argmentacin, en un contexto de debate. Naturalmente, la dialctica se expresa en el debate pblico, por medio de la palabra oral. Pero sus leyes pueden aplicarse a otros tipos de discurso hablado y escrito. Durante la antigedad y la edad media las referencias bsicas en el arte del debate fueron los dilogos de Platn, como ejemplo claro del proceso de justificacin razonablede las propias convicciones, y Aristteles, con sus ocho libros de los Tpicos sobre la dialctica (ms el primer tratado de falacias en Refutaciones sofsticas).
Retrica clsica

Por retrica entendemos arte del bien decir, de dar al lenguaje escrito o hablado eficacia bastante para deleitar, persuadir o conmover64. Ciertamente el arte de la retrica ha sido empleado para convencer en situaciones falaces, y el diccionario recoge trazas de esa

63 64

DRAE, ad. loc. cit. DRAE, ad. loc. cit.

Lgica - 120

valoracin despectiva65. Tiene conexiones con la teora del buen hacer en literatura: la potica, la preceptiva literaria y, sobre todo, la oratoria. Los primeros tratadistas del arte de convencer con la palabra fueron Platn ( Gorgias), Aristteles, Cicern (De la invencin Retrica, La retrica a Herennio; De Oratore), Marco Sneca y Qintiliano (Institutiones, libro II). El arte sigui prosperando en la edad media y en el renacimiento66. En la retrica clsica se consideraban las siguientes fases de elaboracin de un discurso retrico: invencin, o reflexin sobre los argumentos y creacin de los mismos; disposicin, modo y forma de distribuirlos; elocucin, puesta en obra del discurso con uso de la palabra y estilo67. Invencin San Agustn dice que los argumentos se deben crear para ut veritas pateat, placeat et flectet (que la verdad aparezca, se haga deleitable e incline los nimos de los oyentes). La regla fundamental es que, excepto en la irona, toda afirmacin debe ser verdadera, y haber sido suficientemente verificada. Para estar seguro de la veracidad los lugares comunes para tomar nuestra informacin son: la definicin, la enumeracin de partes, la etimologa, el gnero, la especie y la forma, la semejanza y desemejanza, los contrarios, los adjunta, el antecedente y el consecuente, la causa, el efecto y la comparacin. Otros argumentos eran propios para deleitar y para mover los nimos. Disposicin En la disposicin se estudiaban las reglas propias de cada gnero, como pueden sen la novela, el discurso poltico, la oratoria del abogado, etc. Cada uno tiene su propias reglas, ritmo y modos tpicos de proceder. Elocucin En la elocucin se estudiaban con detenimiento las formas que podan adoptarse para elaborar un discurso bello y convincente. Cada una de estas formas (repeticin, gradacin, sinonimia, anttesis, licencia, prolepsis, paradoja, imprecacin, etc). Se estudiaban tambin las figuras literarias: metfora, metonimia, irona, hiprbole,etc. Adems, en los manuales ms modernos se aadan otras partes, como la actio, que se ocupa de la entonacin y la correcta exposicin (hoy en da se ensea en la locucin), y la memoria, o el arte de recordar ordenadamente todo el material preparado. La enseanza de la retrica se haca mediante el estudio y memorizacin de buenos ejemplos de retrica, el conocimiento de las figuras y topos posibles para el debate, y la prctica continuada del debate en pblico y de las otras formas de retrica.
El dilogo como bsqueda de verdad y acuerdo.

En la actualidad no se encuentran muchos tratados de dialctica y retrica. Sin embargo, el dilogo ha saltado al primer puesto entre las aplicaciones prcticas de la razn. Como les sucediera a los griegos en el siglo V, al asumir la democracia como forma de autogobierno, y
Cf. DRAE, ad. loc. cit., acepcin 4. El artculo de Wikipedia sobre el tema aporta una enorme cantidad de informacin, recomendamos su lectura: Wikipedia, art. Retrica, en Internet [nov 2007]: http://es.wikipedia.org/wiki/Retorica 67 Para ampliar estos datos recomendamos el manual de retrica de Angel Romera.
65 66

Lgica - 121

la negociacin como camino de resolucin de conflictos de toda ndole (familiar, laboral, local, internacional), el dilogo aparece como el instrumento y el mbito fundamental en el que se puede lograr este acuerdo. Las implicaciones filosficas son muy importantes y estamos apremiados por la necesidad de formarnos para el dilogo en todos los rdenes de la vida. Una de las corrientes ms fecundas en filosofa ha sido la teora de la accin comunicativa de Jurgen Habermas68. Segn este autor, relacionado con al escuela de Franckfurt, la verdad slo se percibe en un marco de dilogo que asegura la necesaria objetividad de los puntos de vista, y esta verdad se inscribe en una accin comunicativa en la que los interlocutores se reconocen como seres capaces de razonar y dispuestos a la bsqueda de la realidad. Pasando al dilogo, destacamos sus elementos esenciales, dejando para el lector su evidente y urgente desarrollo:
Hay que procurar que quienes estn en formacin aprendan el difcil arte del dilogo. Sobre todo en la convivencia diaria, tanto las conversaciones ordinarias como en el trabajo, por lo tanto insistir en los cinco pasos del dilogo: 1. Escuchar al otro 2. Tratar sinceramente de entender su punto de vista 3. Reconocer con humildad y sencillez lo que tenga de verdad 4. Exponer con seguridad su visin personal de las cosas 5. Estar dispuesto a matizarla o cambiarla si fuera necesario. Que siempre se muestren con humildad abiertos y dispuestos a buscar en comn la verdad. Dialogar es dar y recibir, ensear y aprender. 69

El elemento previo a esta cita es el reconocimiento del otro como interlocutor. Y eso significa asumir que el otro es una persona que puede razonar, cuya intencin, aunque no sea totalmente asumible desde mi actual punto de vista, es aceptable como comienzo, y sus puntos de vista son respetables. Eso conlleva un principio de respeto que debe extenderse incluso al que no respeta, para que el dilogo sea posible (ver el ejercicio sobre el debate ChavezZapatero-Rey de Espaa en la XVII Cumbre Iberoamericana, el 10/11/2007). A partir de esta asuncin, el segundo elemento para garantizar el dilogo es asumir el riesgo a cambiar de opinin. Asumo que no tengo el monopolio de la verdad, y que el otro, mediante sus argumentos, puede llegar a convencerme de otros puntos de vista, o a desvelar mis errores. Eso supone arriesgarse a estropear o perder los actuales puntos de vista, y eso requiere valor. Por ello es muy difcil pero no imposible dialogar con fundamentalistas. Los elementos siguientes son los que indicaba la cita de una conferencia sobre formacin para el apostolado cristiano: escuchar, entender, reconocer, exponer, y retroalimentar. Y dedicarle, por supuesto, toda la atencin necesaria y el tiempo que requiera llegar a asumir

68 Cf. FERNNDEZ Sergio Pablo, Habermas y la Teora Crtica de la Sociedad . Legado y Diferencias en Teora de la Comunicacin , en Cinta de Moebio N1 (Septiembre de 1997) Facultad de Ciencias Sociales. Universidad de Chile; en Internet [2007]: http://rehue.csociales.uchile.cl/publicaciones/moebio/01/frames30.htm 69 Cf. Capacidad de dilogo y escucha, , en Mercaba.org, en Internet [2007]: http://www.mercaba.org/FICHAS/Vida_consagrada/capacidad_de_dialogo_y_escucha.htm .

Lgica - 122

posturas comunes, o acuerdos pragmticos, o al menos una comprensin emptica del punto de vista del otro70.

Recomendamos como material el libro Dilogo democrtico; manual para practicantes : PNUD-DRALIDEA, Democratic Dialogue A Handbook for Practitioners , UNDP, Washigton 2007 . Disponible en Internet con muchos materiales para el trabajo de campo, en medio de conflictos sociales y militares, sobre todo en Amrica latina: http://www.democraticdialoguenetwork.org/index.pl?lang=es .
70

Lgica - 123

Apndice al Tema 13
Lenguaje y argumentacin71 Las falacias no formales [...] Discursos Argumentativos. Los discursos argumentativos son construcciones verbales que agrupan y ordenan opiniones, descripciones. relatos, (incluso gritos, llantos, etc.) con el objeto de persuadir a otros. Los argumentos estn determinados socialmente por las situaciones y los hechos que rodean al objeto y al sujeto de la argumentacin. Aquello sobre lo que se argumento tiene que ver en general con cuestiones que circulan en la superficie de la sociedad y que provocan controversia o son susceptibles de generar diversas posiciones. Argumentos y Opiniones. En primer lugar, para argumentar es necesario estar al tanto del asunto sobre el cual se plantear la discusin. A los tenas a tratar se los denomina objetos de acuerdo. Nexos Lgicos de los Argumentos. El enlace entre opiniones para construir los razonamientos argumentativos se lleva a cabo mediante nexos lgicos. Los nexos lgicos asocian o disocian las opiniones marcando una cierta "lgica" en la argumentacin ("lgica informal"). Los nexos son varios, pero podemos agruparlos en tres:

conjuncin (el SIDA y la corrupcin son los males de nuestro tiempo) disyuncin (seguimos en este rumbo econmico o perdemos el tren de la historia) condicional (si Palermo sigue errando penales entonces ser el fin de la seleccin)

En los argumentos encontraremos estos nexos en muy diversas formas (equivalencias. incompatibilidades, jerarqua- transitividad), Formas de Razonamiento Argumentativo. Este conjunto de nexos se utilizan para construir dos formas de razonamiento argumentativo. 1. El ejemplo: funciona articulando distintas opiniones a modo de ejemplos en forma inductiva.. 2. El entimema: toma la forma de razonamiento deductivo. las opiniones se articulan como premisas que se organizan haciendo aparecer la conclusin como surgiendo de aquellas. Toda argumentacin, no importa el tipo o nmero de nexos que utilice, termina construyndose por ejemplo o entimema. Las Tcnicas Argumentativas. Entre las tcnicas argumentativas ms utilizadas encontramos las siguientes: 1. La tcnica de la reciprocidad: Con esta tcnica se busca aplicar el mismo tratamiento a dos situaciones, hechos o personas que no son idnticas. El recurso que suele utilizarse es la simetra resaltando las similitudes entre las partes y ocultando las caractersticas que las diferencian ("Muchos jvenes de su edad han cometido el mismo error. Por qu lo vamos a juzgar de manera diferente?"). Este texto es la segunda parte sobre dialctica y retrica de la excelente pgina de lgica de El buho fueguino, del Colegio diocesano M.A. Alemn de Ushuaia, Tierra de Fuego (extremo sur de Argentina). Desgraciadamente, a partir del 2007 la pgina est fuera de circulacin. Su direccin original era [2006]: http://www.orbita.starmedia.com/~buhofueguino
71

Lgica - 124 2. La tcnica de la comparacin: En esta tcnica se confrontan dos partes en controversia pero, a diferencia de la anterior, se suele apelar a los lugares comunes de la cantidad ("muchos jvenes a su edad ya han dejado de lado esas conductas y l todava insiste en mantenerlas"); de la cualidad (se utilizan para cuestionar los argumentos cuyas principales razones son cuantitativas: "su madurez y nivel cultural lo hacen singularmente diferente de la gran mayora de jvenes de su edad") o de la ordenacin (ser ms antiguo que, estar antes que, etc). 3. La tcnica del argumento pragmtico: Se la detecta en aquellos argumentos que se esfuerzan por mostrar las consecuencias o los efectos (favorables o desfavorables) de aquello que est puesto en discusin. Esto significa que para justipreciar un hecho, una poltica o una circunstancia, slo vale juzgarla por los resultados que ha arrojado ("no s si vale la pena tener en cuenta otras particularidades poco relevantes: a juzgar por los resultados obtenidos por el seor presidente, no hay otro que est en mejores condiciones que l de ocupar el cargo"). 4. La tcnica del despilfarro: Son los argumentos que sostienen que "ya que estamos en el baile, sigamos bailando". Con este argumento se suele intentar minimizar los esfuerzos que se han realizado hasta ese momento y se procura que se liga en la misma direccin trazada ("Si se cambia el rumbo, todo lo hecho hasta aqu habr sido intil"). 5. La tcnica de la direccin: Con esta tcnica se pretende dividir el objetivo trazado en etapas. Esto se hace ante la imposibilidad o la inconveniencia de presentarlo en su conjunto, o en su totalidad. como la mejor solucin al problema planteado. (Esta tcnica es frecuentemente utilizada para justificar la conveniencia de lograr pequeos objetivos para acercarse a la meta final frente al riesgo que supone intentar un cambio brusco de la situacin. Pensemos en los argumentos esgrimidos por los reformistas" frente a los "revolucionarios"). 6. La tcnica de la superacin: Con esta tcnica se intenta, generalmente, obtener ms de lo que se ha alcanzado hasta ese momento. Es el argumento que se suele utilizar para justificar las utopas, el sostn de grandes o desmedidas posiciones o aspiraciones o, simplemente, la tentativa de que los otros sigan apoyando la propuesta expuesta ("Estamos aqu para no conformarnos con los logros obtenidos. Nuestras conquistas han de ser siempre punto de partida para alcanzar el futuro que todos nos merecemos"). 7. La tcnica del recurso a la autoridad: En los argumentos que se utiliza el recurso a la autoridad se recurre a las actitudes, saberes u opiniones de personas o grupos con alguna reputacin o prestigio, para sostener la posicin que se expone. La autoridad a la que se apela puede ser una Persona, un grupo, una organizacin, una institucin, un documento, una ciencia, etc. El modo ms frecuente de utilizar este recurso es la cita de autoridad: "como dijo el filsofo Kant... de acuerdo con la opinin de la Iglesia... y esto consta expresamente en nuestra doctrina, que sostiene que... 8. La tcnica del ejemplo: Son argumentos que sostienen todo su peso en la exposicin de casos particulares. Esos casos particulares se los suele usar de tres maneras diferentes: a) como ejemplo, para buscar la generalizacin ("El hijo de la seora X adquiri el vicio de las drogas por estar todo el da en los videojuegos. Sin duda, los jvenes que frecuentan esos lugares corren el riesgo serio de ser vctimas del flagelo. Evitemos que nuestros hijos vayan a esos lugares para que no les pase algo parecido"); b) como ilustracin, para sostener una regularidad ya establecida ("las modificaciones introducidas en la poltica educativa son un buen indicio de que estamos en el buen camino"); c) como modelo, para alentar la imitacin ("Sed perfectos como el Padre Celestial es perfecto"). La puesta en escena argumentativa El discurso argumentativo siempre se nos presenta como si estuviramos frente a un espectculo. Ya lo sabemos, estamos dentro de una sociedad que sobrevalora la categora del espectculo. Tambin sabemos que todo espectculo es, en principio, un proyecto, un plan para ser presentado. En efecto, con frecuencia observamos que cuando un sujeto quiere persuadir a otro o a otros, en general "acta", es decir, representa un papel. Advertimos que gesticula, acenta frases, pone nfasis en algunas partes de su alocucin, presenta ejemplos, busca pruebas, si es ingenioso hace algn chiste o alguna broma en el medio de su exposicin; si observa que el otro o los otros no estn muy

Lgica - 125 convencidos, entonces cita a otros como autoridad; en fin, monta un espectculo a los ojos de los dems. La puesta en escena del discurso argumentativo tiene que ver con la disposicin del texto dentro de un mbito (una escenografa) que busca mantener un orden, una coherencia entre todos los elementos que la componen y el texto. Esa bsqueda de coherencia es, de algn modo, la parte lgica del proceso argumentativo, y no puede estar ausente si se pretende ser eficaz. Decimos entonces que la lgica constituye la puesta en escena de ese espectculo y de esa estrategia. En esta puesta en escena se trata de ordenar tanto los recursos que servirn a la construccin del argumento como las acciones del argumentador dentro de la escena. Desde la antigedad se suele dividir el discurso argumentativo en tres grandes partes bien diferenciadas: la inventio, la disposito y la elocutio. La inventio es la instancia de rescate del conjunto de materiales que posibilitarn la ulterior construccin de la argumentacin (ya hemos visto, es el momento de bsqueda y recoleccin de pruebas). La elocutio atraviesa toda la argumentacin, en tanto que expresividad de la misma (porque justamente constituye la exposicin en s misma con las particularidades que la conforman). La dispositio puede entenderse como la parte lgica del argumento, en tanto, que ordenadora y posibilitadora de acceso al sentido de la argumentacin. A este solo efecto es til resear las partes que la componen: a) El exordio, cmo la puesta en marcha de la argumentacin busca captar la atencin del receptor mediante mecanismos de seduccin y puntualizando las divisiones que conforman el resto de la exposicin. De l dice Barthes que es (junto con el eplogo) el momento pasional de la dispositio, porque se priorizan los recursos persuasivos tendientes a movilizar lo irracional del receptor. b) La narratio y la confirmatio son las partes que pretenden mostrarse como ms convincentes (ms racionales, menos pasionales) de la dispositio. De la primera, para ser eficaz se requiere que sea clara, creble y breve en la manifestacin de los hechos y descripciones que contiene; de la segunda se pretende bsicamente: 1. Que defina el objeto que motiva la argumentacin; 2. Que las razones que expone para convencer sean efectivamente convincentes y no dispersas o dbiles. c) El eplogo, pasional como el exordio, presentar los rasgos que le permitan al receptor advertir que se llega al final de la .argumentacin y al mismo tiempo se expondr con nfasis, acudiendo a recursos si se quiere histrinicos, que permitan pegar el salto del carcter sobrio y demostrativo de las dos instancias anteriores a la bsqueda de la persuasin mediante la utilizacin de efectos ampulosos.

Lgica - 126

Ejercicios del Tema 13


Teora 1. Explique qu es dialctica y qu es retrica. 2. Haga un esquema de las partes de un discurso demostrativo. Aplicaciones 7. Presentamos la transcripcin del famoso debate entre el presidente del gobierno espaol y el presidente venezolano en la XVII Cumbre Iberoamericana (10/11/2007), con la famosa intervencin del Rey Juan Carlos I. Para analizar el debate se le propone: 1. Identifique los objetivos de cada uno de los agentes en el debate (qu idea quieren argumentar) 2. Identifique el modo en el que se comportan, y jzguelo segn los cinco pasos del dilogo propuestos en el tema. 3. Haga un ensayo breve sobre la tesis de Zapatero: respetar aunque se discrepe como medio para poder trabajar en el entendimiento.

Transcripcin del tenso enfrentamiento entre el Rey de Espaa, el presidente venezolano, Hugo Chvez, y el presidente del gobierno espaol, Jos Luis Rodrguez Zapatero, que ha tenido lugar durante la clausura de la XVII Cumbre Iberoamericana, en Santiago. -Zapatero: Estamos en una mesa donde hay gobiernos democrticos, que representan a sus ciudadanos en una comunidad iberoamericana que tienen como principios esenciales el respeto. Se puede estar en las antpodas de una posicin ideolgica, no ser yo el que est cerca del ex presidente Aznar, pero el ex presidente Aznar fue elegido por los espaoles, y exijo, exijo -Chvez: Dgale a l que respete. -Zapatero: Exijo que t, un momentn -Chvez: Dgale lo mismo a l. -Zapatero: Exijo ese respeto, por una razn, adems -Chvez: Dgale lo mismo a l, presidente. -Zapatero: Por supuesto -Chvez: Dgale lo mismo a l -Rey: Por qu no te callas? -Bachelet: Por favor, no hagamos dilogo, han tenido tiempo para plantear su posicin, presidente, termine. -Chvez: Podr ser espaol el presidente Aznar, pero es un fascista y es un Zapatero: Presidente Hugo Chvez, creo que hay una esencia y es que para respetar y para ser respetado, debemos procurar no caer en la descalificacin. Se puede discrepar radicalmente de las ideas, denunciar los comportamientos, sin caer en la descalificacin. Lo que quiero expresar es que es una buena forma de poder trabajar entendernos en favor de nuestros pueblos, que nos respetemos, a los representantes democrticos, y pido presidenta Bachelet que sa sea una norma de conducta en un foro que representa a los ciudadanos, que respetemos a todos nuestros dirigentes, a todos los gobernantes y ex gobernantes de los pases que formamos esta comunidad. Creo que es un buen principio y deseo fervientemente que se sea un cdigo de conducta, porque las formas dan el ser a las cosas, y se puede discrepar radicalmente de todo respetando a las personas, se es el principio para que uno luego pueda ser respetado. Estoy seguro que toda esta mesa y todos los latinoamericanos quieren que todos los gobernantes democrticos () seamos respetados, hoy [alza la voz] y maana, aunque discrepemos profundamente de las ideas que tengamos. [aplausos]

Lgica - 127

-Chvez: El gobierno de Venezuela se reserva el derecho a responder cualquier agresin en cualquier lugar, en cualquier espacio y en cualquier tono. Extrado de: http://www.elperiodico.com

8. Elaboren un debate sobre un tema controvertido, usando las notas del apndice y los recursos de retrica que se indican en la bibliografa del captulo. Se har en dos grupos, sobre un tema controvertido de suficiente inters, y acordando previamente las dos posturas que se debaten. Un tercer grupo har de espectador y jurado para asumir la postura que le haya convencido.

Lgica - 128

ANEXO 1: P R O B L E M A S D E L G I C A
En Internet [2006]: http://platea.pntic.mec.es/~jescuder/logica.htm
1. SILENCIO. Si ngela habla ms bajo que Rosa y Celia habla ms alto que Rosa, habla ngela ms alto o ms bajo que Celia? 2. LA NOTA MEDIA. La nota media conseguida en una clase de 20 alumnos ha sido de 6. Ocho alumnos han suspendido con un 3 y el resto super el 5. Cul es la nota media de los alumnos aprobados? 3. LOS CUATRO ATLETAS. De cuatro corredores de atletismo se sabe que C ha llegado inmediatamente detrs de B, y D ha llegado en medio de A y C. Podra Vd. calcular el orden de llegada? 4. SEIS AMIGOS DE VACACIONES. Seis amigos desean pasar sus vacaciones juntos y deciden, cada dos, utilizar diferentes medios de transporte; sabemos que Alejandro no utiliza el coche ya que ste acompaa a Benito que no va en avin. Andrs viaja en avin. Si Carlos no va acompaado de Daro ni hace uso del avin, podra Vd. decirnos en qu medio de transporte llega a su destino Toms. 5. LOS CUATRO PERROS. Tenemos cuatro perros: un galgo, un dogo, un alano y un podenco. ste ltimo come ms que el galgo; el alano come ms que el galgo y menos que el dogo, pero ste come ms que el podenco. Cul de los cuatro ser ms barato de mantener? 6. TENIS DE CATEGORA. En un partido del prestigioso torneo de tenis de Roland Garros se enfrentaron Agasy y Becker. El triunfo correspondi al primero por 6-3 y 7-5. Comenz sacando Agasy y no perdi nunca su saque. Becker perdi su servicio dos veces. Agasy rompi el servicio de su rival en el segundo juego del primer set y, en qu juego del segundo set? 10. CABALLOS. El caballo de Mac es ms oscuro que el de Smith, pero ms rpido y ms viejo que el de Jack, que es an ms lento que el de Willy, que es ms joven que el de Mac, que es ms viejo que el de Smith, que es ms claro que el de Willy, aunque el de Jack es ms lento y ms oscuro que el de Smith. Cul es el ms viejo, cul el ms lento y cul el ms claro? En ocasiones, ciertas personas se encuentran en una situacin crtica, y slo por su agudeza e inteligencia pueden salir de ella. 11. EL EXPLORADOR CONDENADO. Un explorador cay en manos de una tribu de indgenas, se le propuso la eleccin entre morir en la hoguera o envenenado. Para ello, el condenado deba pronunciar una frase tal que, si era cierta, morira envenenado, y si era falsa, morira en la hoguera. Cmo escap el condenado a su funesta suerte? 19. LAS DEPORTISTAS. Ana, Beatriz y Carmen. Una es tenista, otra gimnasta y otra nadadora. La gimnasta, la ms baja de las tres, es soltera. Ana, que es suegra de Beatriz, es ms alta que la tenista. Qu deporte practica cada una? 20. SILOGISMOS. Ejemplo que est en todos los manuales de lgica elemental. El silogismo: Los hombres son mortales, Scrates es hombre. Luego, Scrates es mortal. Es indudablemente conocido e inevitablemente vlido. Qu ocurre con el siguiente?: Los chinos son numerosos, Confucio es chino. Luego, Confucio es numeroso.

Lgica - 129 24. BLANCO, RUBIO Y CASTAO. Tres personas, de apellidos Blanco, Rubio y Castao, se conocen en una reunin. Poco despus de hacerse las presentaciones, la dama hace notar: "Es muy curioso que nuestros apellidos sean Blanco Rubio y Castao, y que nos hayamos reunido aqu tres personas con ese color de cabello" "S que lo es -dijo la persona que tena el pelo rubio-, pero habrs observado que nadie tiene el color de pelo que corresponde a su apellido." "Es verdad!" -exclam quien se apellidaba Blanco. Si la dama no tiene el pelo castao, de qu color es el cabello de Rubio? 25. LOS CIEN POLTICOS. Cierta convencin reuna a cien polticos. Cada poltico era o bien deshonesto o bien honesto. Se dan los datos: a) Al menos uno de los polticos era honesto. b) Dado cualquier par de polticos, al menos uno de los dos era deshonesto. Puede determinarse partiendo de estos dos datos cuntos polticos eran honestos y cuntos deshonestos? 26. COMIENDO EN EL RESTAURANTE. Armando, Basilio, Carlos y Dionisio fueron, con sus mujeres, a comer. En el restaurante, se sentaron en una mesa redonda, de forma que: - Ninguna mujer se sentaba al lado de su marido. - Enfrente de Basilio se sentaba Dionisio. - A la derecha de la mujer de Basilio se sentaba Carlos. - No haba dos mujeres juntas. Quin se sentaba entre Basilio y Armando? 27. SELLOS DE COLORES. Tres sujetos A, B y C eran lgicos perfectos. Cada uno poda deducir instantneamente todas las conclusiones de cualquier conjunto de premisas. Cada uno era consciente, adems, de que cada uno de los otros era un lgico perfecto. A los tres se les mostraron siete sellos: dos rojos, dos amarillos y tres verdes. A continuacin, se les taparon los ojos y a cada uno le fue pegado un sello en la frente; los cuatro sellos restantes se guardaron en un cajn. Cuando se les destaparon los ojos se le pregunt a A: -Sabe un color que con seguridad usted no tenga? A, respondi: -No. A la misma pregunta respondi B: -No. Es posible, a partir de esta informacin, deducir el color del sello de A, o del de B, o del de C? 28. LA LGICA DE EINSTEIN. Problema propuesto por Einstein y traducido a varios idiomas conservando su lgica. Einstein aseguraba que el 98% de la poblacin mundial sera incapaz de resolverlo. Yo creo que Vd. es del 2% restante. Intntelo y ver como tengo razn. Condiciones iniciales: - Tenemos cinco casas, cada una de un color. - Cada casa tiene un dueo de nacionalidad diferente. - Los 5 dueos beben una bebida diferente, fuman marca diferente y tienen mascota diferente. - Ningn dueo tiene la misma mascota, fuma la misma marca o bebe el mismo tipo de bebida que otro. Datos: 1. El noruego vive en la primera casa, junto a la casa azul. 2. El que vive en la casa del centro toma leche. 3. El ingls vive en la casa roja. 4. La mascota del Sueco es un perro. 5. El Dans bebe t. 6. La casa verde es la inmediata de la izquierda de la casa blanca. 7. El de la casa verde toma caf. 8. El que fuma PallMall cra pjaros. 9. El de la casa amarilla fuma Dunhill. 10. El que fuma Blend vive junto al que tiene gatos. 11. El que tiene caballos vive junto al que fuma Dunhill. 12. El que fuma BlueMaster bebe cerveza. 13. El alemn fuma Prince. 14. El que fuma Blend tiene un vecino que bebe agua. Quin tiene peces por mascota?

Lgica - 130 29. COLOCANDO NMEROS (1). Colocar un nmero en cada cuadro, teniendo en cuenta que:

a) b) c) d)

3, 6, 8, estn en la horizontal superior. 5, 7, 9, estn en la horizontal inferior. 1, 2, 3, 6, 7, 9, no estn en la vertical izquierda. 1, 3, 4, 5, 8, 9, no estn en la vertical derecha.

30. COLOCANDO NMEROS (2). Colocar un nmero en cada cuadro, teniendo en cuenta que:

a) b) c) d)

3, 5, 9, estn en la horizontal superior. 2, 6, 7, estn en la horizontal inferior. 1, 2, 3, 4, 5, 6, no estn en la vertical izquierda. 1, 2, 5, 7, 8, 9, no estn en la vertical derecha.

31. LA BARAJA ESPAOLA. En una mesa hay cuatro cartas en fila: 1. El caballo esta a la derecha de los bastos. 2. Las copas estn mas lejos de las espadas que las espadas de los bastos. 3. El rey esta mas cerca del as que el caballo del rey. 4. Las espadas, mas cerca de las copas que los oros de las espadas. 5. El as esta mas lejos del rey que el rey de la sota. Cules son los cuatro naipes y en qu orden se encuentran? 32. COLOCANDO NMEROS (3). Colocar un nmero en cada cuadro, teniendo en cuenta que:

a) b) c) d)

4, 5, 6, estn en la horizontal superior. 7, 8, estn en la horizontal inferior. 2, 3, 4, 5, 8, 9, no estn en la vertical izquierda. 1, 5, 6, 7, 8, 9, no estn en la vertical derecha.

33. EN EL ASCENSOR. Cuatro jugadores de rugby entran en un ascensor que puede trasportar un mximo de 380 kilos. Para que no suene una alarma, que detendra al elevador por exceso de carga, tiene usted que calcular su peso total con gran rapidez. Pero, cuanto pesa cada jugador? He aqu los datos: Pablo es quien pesa ms: si cada uno de los otros pesara tanto como el, la alarma detendra el ascensor. Carlos es el mas ligero: el ascensor podra subir a cinco como el Renato pesa 14 kilos menos que Pablo, y solo seis menos que Jess. Jess pesa 17 kilos mas que Carlos. Los pesos de Pablo y de Carlos son mltiplos de cinco. 34. COLOCANDO NMEROS (4). Colocar un nmero en cada cuadro, teniendo en cuenta que:

Lgica - 131 a) b) c) d) 2, 5, 6, estn en la horizontal superior. 4, 7, 8, estn en la horizontal inferior. 2, 3, 4, 6, 7, 9, no estn en la vertical izquierda. 1, 2, 4, 5, 8, 9, no estn en la vertical derecha.

35. LA ORUGA Y EL LAGARTO. La oruga piensa que tanto ella como el lagarto estn locos. Si lo que cree el cuerdo es siempre cierto y lo que cree el loco es siempre falso, el lagarto est cuerdo? (Original de Lewis Carroll) 36. LOS TRES DADOS. Tengo tres dados con letras diferentes. Al tirar los dados puedo formar palabras como: OSA, ESA, ATE, CAE, SOL, GOL, REY, SUR, MIA, PIO, FIN, VID, pero no puedo formar palabras tales como DIA, VOY, RIN. Cules son las letras de cada dado? 37. SON MENTIROSOS? Andrs: Cuando yo digo la verdad, t tambin. Pablo: Cuando yo miento, tu tambin. Es posible que en esta ocasin uno mienta y el otro no? 38. PASTELES PARA NIOS. Un nio y medio se comen un pastel y medio en un minuto y medio. Cuntos nios hacen falta para comer 60 pasteles en media hora? 39. LA BODA. Cuando Mara pregunt a Mario si quera casarse con ella, este contest: "No estara mintiendo si te dijera que no puedo no decirte que es imposible negarte que si creo que es verdadero que no deja de ser falso que no vayamos a casarnos". Mara se mare. Puede ayudarla dicindola si Mario quiere o no quiere casarse? 40. EL ENCUENTRO. ngel, Boris, Csar y Diego se sentaron a beber. El que se sent a la izquierda de Boris, bebi agua. ngel estaba frente al que beba vino. Quien se sentaba a la derecha de Diego beba ans. El del caf y el del ans estaban frente a frente. Cul era la bebida de cada hombre? 41. EL NMERO. Buscamos un nmero de seis cifras con las siguientes condiciones. - Ninguna cifra es impar. - La primera es un tercio de la quinta y la mitad de la tercera. - La segunda es la menor de todas. - La ltima es la diferencia entre la cuarta y la quinta. 42. LA HILERA DE CASAS. En una hilera de cuatro casas, los Brown viven al lado de los Smith pero no al lado de los Bruce. Si los Bruce no viven al lado de los Jones, quines son los vecinos inmediatos de los Jones? 43. COMPLETANDO. Completar la oracin siguiente colocando palabras en los espacios: Ningn pobre es emperador, y algunos avaros son pobres: luego: algunos (.........) no son (.........). 44. EXAMEN DE HISTORIA. De las siguientes afirmaciones. cules son las dos que. tomadas conjuntamente, prueban en forma concluyente que una o ms nias aprobaron el examen de historia? a) Algunas nias son casi tan competentes en historia como los nios. b) Las nias que hicieron el examen de historia eran ms que los nios. c) Ms de la mitad de los nios aprobaron el examen. d) Menos de la mitad de todos los alumnos fueron suspendidos. 45. CONDUCTORES Y SU SEXO. Las estadsticas indican que los conductores del sexo masculino sufren ms accidentes de automvil que las conductoras. La conclusin es que: a) Como siempre, los hombres, tpicos machistas, se equivocan en lo que respecta a la pericia de la mujer conductora. b) Los hombres conducen mejor, pero lo hacen con ms frecuencia.

Lgica - 132 c) Los hombres y mujeres conducen igualmente bien, pero los hombres hacen ms kilometraje. d) La mayora de los camioneros son hombres. e) No hay suficientes datos para justificar una conclusin. 46. GASOLINA. Si al llegar a la esquina Jim dobla a la derecha o a la izquierda puede quedarse sin gasolina antes de encontrar una estacin de servicio. Ha dejado una atrs, pero sabe que, si vuelve, se le acabar la gasolina antes de llegar. En la direccin que lleva no ve ningn surtidor. Por tanto: a) Puede que se quede sin gasolina. b) Se quedar sin gasolina. c) No debi seguir. d) Se ha perdido. e) Debera girar a la derecha. f) Debera girar a la izquierda. 47. NEUMTICOS. Todos los neumticos son de goma. Todo lo de goma es flexible. Alguna goma es negra. Segn esto, cul o cules de las siguientes afirmaciones son ciertas? a) Todos los neumticos son flexibles y negros. b) Todos los neumticos son negros. c) Slo algunos neumticos son de goma. d) Todos los neumticos son flexibles. e) Todos los neumticos son flexibles y algunos negros. 48. OSTRAS. Todas las ostras son conchas y todos los conchas son azules; adems algunas conchas son la morada de animalitos pequeos. Segn los datos suministrados, cul de las siguientes afirmaciones es cierta? a) Todas las ostras son azules. b) Todas las moradas de animalitos pequeos son ostras. c) a) y b) no son ciertas. d) a) y b) son ciertas las dos. 49. PUEBLOS. A lo largo de una carretera hay cuatro pueblos seguidos: los Rojos viven al lado de los Verdes pero no de los Grises; los Azules no viven al lado de los Grises. Quines son pues los vecinos de los Grises? 50. EL TEST. Toms, Pedro, Jaime, Susana y Julia realizaron un test. Julia obtuvo mayor puntuacin que Toms, Jaime puntu ms bajo que Pedro pero ms alto que Susana, y Pedro logr menos puntos que Toms. Quin obtuvo la puntuacin ms alta? ----------------- ----------------

De http://www.xtec.es/~jjover/page3.htm Dos mineros salen de la mina. Uno tiene la cara manchada de carbon y el otro no. Coinciden en el ascensor, se saludan y cuando van cada uno para su casa (cada uno en una direccin) el que tiene la cara limpia se la limpia y el que tiene la cara sucia no se la limpia. Porqu? Despus del Concilio Vaticano segundo, y segn la doctrina catlica... Puede un hombre casarse con la hermana de su viuda? Yendo hacia Palo Santo me cruc con una vieja. La vieja tenia siete sacos, en cada saco siete gatos. Entre yo, la vieja, sacos y gatos Cuantos bamos hacia Palo Santo?

Lgica - 133

Resulta que iban a matar a un prisionero, pero deciden darle una oportunidad y le dicen lo siguiente: Te vamos a dar 12 pastillas, de las cuales una esta envenenada, esta tiene una diferencia de peso con respecto a las dems, pero no se sabe si pesa mas o menos. Aqu tienes esta balanza y en tres pesadas tienes que decir cual es la pastilla que esta envenenada. Nota: el prisionero queda en libertad Un arquelogo, despus de mucho tiempo y esfuerzo encuentra dos sarcfagos en lo mas profundo de una extraa pirmide. Los abre y encuentra en su interior dos momias magnficamente conservadas. Las desenvuelve cuidadosamente y al momento de terminar, su cara palidece y mientras retrocede, exclama: "Dios mo, son Adn y Eva !". Cmo lo ha averiguado?

Potrebbero piacerti anche